PATHOLOGY Superexam TOPNOTCH

Réussis tes devoirs et examens dès maintenant avec Quizwiz!

Cell-derived mediators of inflammation, in contrast with plasma-derived mediators A. Are normally sequestered in granule and can be rapidly secreted by granule exocytosis in response to stimulus. B. Are produced mainly in the liver. C. Are inactive precursors that must be activated, usually by a series of proteolytic cleavages to acquire their biologic properties. D. Act only in one or few target cell types. E. None of the above.

122 A. Options B and C are properties of plasma-derived mediators of inflammation. Option D is incorrect. Both types of mediators can act on one or few target cells. SIMILAR TO PREVIOUS BOARD EXAM CONCEPT.

The following presents with granulomatous inflammation EXCEPT: A. Histoplasmosis B. Tuberculosis C. Sarcoidosis D. Schistomiasis E. Molluscum contagiosum

123 E. Histoplasma mimics tuberculosis. All except E would show granulomatous type of inflammation in the affected organs. SIMILAR TO PREVIOUS BOARD EXAM CONCEPT.

Which of the following lesions is more prone to develop breast CA A. Proliferative atypia B. Proliferation without atypia C. Carcinoma in situ D. Fibroadenoma E. None of the above

153 C. Lobular carcinoma in situ is a fertile ground for breast cancer, proliferation without atypia and proliferation with atypia is hyperplasia, fibroadenoma is a benign condition.

Which among the following is involved in staging a particular malignancy except A. Presence of metastasis B. Lymph node involvement C. Degree of differentiation D. Size of the mass E. None of the above

154 C. Degree of differentiation or tissue grading is not needed in the staging in most malignancies

What is the most common type of gastric malignancy? A. Lymphoma B. Gastric adenocarcinoma C. Carcinoid tumor D. Gastric adenoma E. Pleomorphic adenoma

160 B. Gastric adenoCA is the most common type of gastric malignancy

This can be a pathologic or physiologic process which is described as the increase in the number of cells in an organ. A. Hypertrophy B. Hyperplasia C. Atrophy D. Metaplasia E. Anaplasia

161 B. hypertrophy is increase in size of cells. Atrophy is decrease in number and size of the cell. Metaplasia is replacement of one differentiated type to another. Anaplasia happens when cells lose their morphologic characteristic of a mature cells.

In Rheumatic Heart Disease, at which layer/layers of the heart can you find Aschoff bodies? A. pericardium B. myocardium C. endocardium D. B and C E. All layers

162 E. Aschoff bodies can be found in all layers of the heart in RF and RHD. SIMILAR TO PREVIOUS BOARD EXAM CONCEPT/PRINCIPLE

In Libman-Sacks Disease, which of the following valve is usually affected? A. Tricuspid B. Pulmonary C. Mitral D. Aortic E. All valves are equally involve

164 C. In Robbins, Mitral and Tricuspid. In Medscape, Mitral and Aortic.

Wire-loop capillaries are seen in which type of kidney disease? A. Lupus nephritis B. RPGN C. Diabetic nephropathy D. Polycystic kidney disease E. Acute tubular necrosis

166 A. SIMILAR TO PREVIOUS BOARD EXAM CONCEPT/PRINCIPLE

Reiter syndrome is an autoimmune disease which is usually triggered by infections caused by Shigella, Salmonella, Yersinia, and Chlamydia. Which among the following is/are component of the disease? A. urethritis B. arthritis C. uveitis D. A and B E. All of the above

177 E. these three are the triad of Reiter syndrome

In infective endocarditis, what is the most organism involved in patients with native but previously damaged heart valves? A. Strep viridans B. Staph epidermidis C. Strep bovis D. Staph aureus E. Enterococcus

178 A.

Which of the following is the most common cause of chronic pancreatitis? A. Gallstones B. Alcoholism C. Hypercholesterolemia D. Smoking E. Hereditary predisposition

18 B. Most common cause Acute pancreatitis â€" gallstones Chronic pancreatitis â€" alcoholism Source: Robbins and Cotran Pathologic Basis of Disease 8th ed p. 896

Which of the following diseases is an X-linked dominant disorder? A. G6PD deficiency B. Hemophilia C. Ehlers Danlos syndrome D. Marfan syndrome E. Alport syndrome

180 E. Alport syndrome and vitamin D resistant rickets are the only most commonly associated with X linked dominant genetic transmission.

What is the valve most commonly affected in Libman-Sacks endocarditis? A. Mitral B. Tricuspid C. Aortic D. Pulmonic E. A and B

185 A. SIMILAR TO PREVIOUS BOARD EXAM CONCEPT/PRINCIPLE.. In SLE, mitral and tricuspid valvulitis with small, sterile vegetations, called Libman-Sacks endocarditis is ocassionally encountered. according to harrison, "The characteristic endocardial lesions of SLE are verrucous valvular abnormalities known as Libman- Sacks endocarditis . They most often are located on the left-sided cardiac valves, particularly on the ventricular surface of the posterior mitral leaflet, and are made up almost entirely of fibrin. " So yes, the correct answer is mitral valve.

8 year old male patient abruptly develops malaise, fever, nausea, oliguria, and hematuria 2 weeks after recovery from a sore throat. The patient has red cell casts in the urine, mild proteinuria, periorbital edema, and hypertension. What is the expected electron microscopy finding in this case? A. Loss of foot processes B. Subendothelial deposits C. Mesangial and paramesangial dense deposits D. Subepithelial deposits E. Subepithelial humps

198 E. Diagnosis is PSGN. The electron microscopy finding in this condition is subepithelial humps. A- minimal change disease; B- MPGN; C- IgA nephropathy; D- Membranous GN

In the heart, grossly apparent bands of yellowed myocardium alternating with bands of darker, red-brown, uninvolved myocardium is known as tigered effect. This is due to accumulation of what material in the cardiac cells? A. calcium B. glycogen C. triglyceride D. cholesterol E. lipofuscin

199 C. The terms steatosis and fatty change describe abnormal accumulations of triglycerides within parenchymal cells. This is most often seen in the liver and heart. In the heart, fat deposits create grossly apparent bands of yellowed myocardium alternating with bands of darker, red-brown, uninvolved myocardium (tigered effect)

The gene involved in Williams syndrome is: A. Fibrillin B. Collagen C. Elastin D. Reticulin E. Chondroitin

201 C. Williams syndrome: supravalvular aortic stenosis, hypercalcemia, cognitive abnormalities, and hallmark facial anomalies.

Nocturnal pain relieved by aspirin intake is characteristic of: A. Osteomyelitis B. Osteoid osteoma C. Chondroma D. Osteosarcoma E. Ossifying fibroma

204 B. The pain is probably caused by excessive prostaglandin E2 (PGE2) production by the proliferating osteoblasts.

What is the most important factor in the prognosis of GI carcinoid? A. Size B. Nuclear grade C. Location D. Metastasis E. Presence of other tumors

205 C. Location in the jejunum or ileum carries the worst prognosis.

Which condition is most likely to give rise to hepatocellular carcinoma? A. Hereditary tyrosinemia B. Chronic Hepatitis B C. Chronic Hepatitis C D. Chronic Alcoholism E. Autoimmune hepatitis

206 A. In hereditary tyrosinemia, almost 40% of patients develop the tumor despite adequate dietary control. However, this condition is extremely rare.

Mucocutaneous lymph node syndrome preferentially affects the: A. Temporal artery B. Aortic arch C. Renal arteries D. Coronary arteries E. Tibial and radial arteries

207 D. AKA Kawasaki disease because it presents with conjunctival and oral erythema and erosion, edema of the hands and feet, erythema of the palms and soles, a desquamative rash, and cervical lymph node involvement. Approximately 20% of untreated patients develop cardiovascular sequela involving the coronary arteries.

A person with hypothyroidism and sensorineural deafness most likely has an abnormality of the: A. H-P-O axis B. Thyroglobulin C. Anion transporter D. Iodide channel E. Peripheral deiodinases

208 C. Pendred syndrome is caused by a mutation in the SLC26A4 gene whose product, pendrin, is an anion transporter expressed on the apical surface of thyrocytes and in the inner ear.

What is the most common clinical manifestation among patients with Sipple Syndrome? A. Pheochromocytoma B. Medullary carcinoma of the thyroid C. Parathyroid adenoma D. Neuroganglioma E. Pituitary adenoma

209 B. Primary hyperparathyroidism is the most common manifestation in MEN 1 (Wermer syndrome).

40 year old male patient presented with repeated episodes of bloody stools. Workup and evaluation revealed multiple superficial broad based ulcers, with diffuse inflammation limited to the colon, biopsy revealed inflammation limited only to the submucosal area, which of the following is the most likely condition? A. Crohn's disease B. bacillary dysentery C. amoebic colitis D. ulcerative colitis E. none of the above

245 D. SIMILAR TO PREVIOUS BOARD EXAM CONCEPT/PRINCIPLE, no granulomas, no strictures, limited to the colon, submucosal inflammation, no skip lesions

50 year old male chronic smoker presents with chronic cough dyspnea and sputum production. He also has bouts of easy fatigability. Patient also presents with increased anteroposterior diameter of the chest wall. Patient most likely has this type of emphysema A. panacinar B. centrilobular C. paraseptal D. alveolar E. none of the above

246 B. SIMILAR TO PREVIOUS BOARD EXAM CONCEPT/PRINCIPLE, panacinar is with alpha 1 antitrypsin deficiency

A 50 year old male alcoholic presents with liver failure. He has spider angiomata and testicular atrophy. A liver biopsy would reveal which of the following? A. Ballooning degeneration of hepatocytes B. Disorganized liver cells with Councilman bodies C. Regenerating hepatic nodules surrounded by extensive fibrous tissue D. Severe congestion with centrilobular atrophy

342 C. This patient presents with signs of chronic liver disease. The pathologic hallmark of chronic liver disease is the presence of fibrosis.

A 59 year old woman had loss of consciousness that persisted for over an hour. When she became arousable, she cannot speak nor move her right arm or leg. A cerebral angiogram revealed an occlusion to her left middle cerebral artery. Months later, a CT scan shows a large 5 cm cystic area in her left parietal lobe cortex. This CT finding is most likely the consequence of resolution from which of the following cellular events? A. Apoptosis B. Atrophy C. Coagulation necrosis D. Liquefactive necrosis

344 D. This patient had an ischemic stroke. For unknown reasons, hypoxic death of cells within the CNS often manifests as liquefactive necrosis. Reference: Robbins, Pathologic Basis of Disease, 8th ed. p. 15

Classic polyarteritis nodosa spares the blood vessels in which of the following organs? A. Spleen B. Kidneys C. Heart D. Lungs

346 D. Reference: Topnotch Pathology Handouts

A 12 year old girl was brought to a paediatrician because of joint pains. This was accompanied by moderate grade fever. On PE, the paediatrician noted diastolic murmurs and friction rub. The patient had a history of pharyngitis 10 weeks prior to consultation. The major criterion seen in this patient that would help in your diagnosis is: A. Carditis B. Fever C. Joint pains D. Previous history of pharyngitis

347 A. Diagnosis is Rheumatic Fever. Based on the Jones Criteria, fever and joint pains are only minor criteria (migratory polyarthritis is a major criterion), and evidence of prior Strep infection, though a required criterion, is not considered as one of the five major criteria.

As the pathologist tasked to autopsy a 10/M who recently died due to rabies, you know that Negri bodies can be found in the? A. Cerebral cortex B. Cerebellum C. Pons D. Medulla E. None of the above

362 B. There is widespread neuronal degeneration and an inflammatory reaction that is most severe in the brainstem. The basal ganglia, spinal cord, and dorsal root ganglia may also be involved. Negri bodies are cytoplasmic, round to oval, eosinophilic inclusions found in the pyramidal neurons of the hippocampus and Purkinje cells of the cerebellum. Robbins 9th pg 1277

Which of the following is TRUE regarding apoptosis? A. The intrinsic pathway is initiated by the release of cytochrome c into the cytoplasm leading to activation of caspase 9 B. Apoptosis is inhibited by anti-apoptotic proteins such as BAX and BAK C. Triggering of the death receptor pathway leads to the activation of caspase 8 and 10 D. A and C E. All of the above

363 D. Anti-apoptotic proteins include BCL-2 BCL-xL and MCL1 while pro-apoptotic proteins include BAX and BAK. Sensors of cellular stress and damage which are the regulators between the two groups include BAD BIM BID Puma and Noxa Robbins 9th pg 53-56

A 34/F with lupus successfully gave birth to a live baby girl. On PE, the neonate was bradycardic but otherwise asymptomatic with no signs of distress. On ECG, there was prolongation of the PR interval followed by a dropped beat. What marker/s is/are associated with this condition? A. Anti-Ro B. Anti-CCP C. Anti-dsDNA D. Anti-Sm E. C and D

372 A. Anti-Ro/SS-A and Anti-La/SS-B are associated with congenital heart block and neonatal lupus Robbins 9th pg 219

C3 convertase splits C3 into two distinct fragments, C3a and C3b. C3 convertase is formed through which pathway/s? A. Classical B. Alternative C. Lectin D. A and B E. All of the above

373 E. All three pathways of complement activation lead to the formation of an active enzyme called the C3 conver- tase, which splits C3 into two functionally distinct frag- ments, C3a and C3b. Robbins 9th pg 88

Pneumoconiosis refers to nonneoplastic lung reaction to inhalation of mineral dusts encountered in the work place. Which pneumoconiosis is characterized by discrete pale to blackened nodules in the upper zones of the lungs. Radiographically it presents as eggshell calcifications. This disease may be progressive even if patient is no longer exposed and it has also been associated with increased susceptibility to PTB. A. Coal workers’ pneumoconiosis B. Silicosis C. Asbestosis D. Byssinosis E. Sarcoidosis

10 B. Silicosis â€" upper lobes, eggshell calcification, PTB Source: Robbins and Cotran Pathologic Basis of Disease 8th ed p. 699

20. A 14 year old female presented with a breast mass. Biopsy was done which showed dense collagenous connective tissue and marked micropapillary epithelial hyperplasia of the duct lining. Which of the following is the most likely diagnosis? a. fibroadenoma b. fibrocystic disease c. gynecomastia d. phylloides tumor e. Papilloma

100 C. Yes during our exam, it was female and not male but the description was really referring to gynecomastia. A - delicate cellular, and often myxoid stroma which resembles normal intralobular stroma. B - nonproliferative changes which includes cysts, fibrosis, and adenosis. D - bulbous protrusions due to presence of nodules of proliferating stroma covered by epithelium. E - multiple branching fibrovascular cores each having a connective tissue axis lined by luminal and myoepithelial cells Robbins 8th edition p. 1093

Characteristic cytologic feature of medullary thyroid cancer: A. Orphan annie B. Vacuolated nucleus C. Amyloid stroma D. Ground glass E. Pleomorphic giant cells

101 C. Page 73 of Topnotch Handout. Medullary thyroid carcinoma has polygonal to spindle shaped cells, acellular amyloid deposits, and C-cell hyperplasia.

Which of the following is a calcitonin secreting tumor of the thyroid gland? A. Papillary CA B. Follicular CA C. Medullary CA D. Hurthle cell CA E. Anaplastic CA

102 C. Page 73 of Topnotch Handout. Medully thyroid carcinoma is a neuroendocrine neoplasma derived from the parafollicular cells or c-cells which secrete calcitonin and is important in diagnosis.

23y/o female consults because of bipedal edema & passing out of tea colored urine. On PE, her BP is 160/100. she has +2 bipedal edema. The urinalysis showed +4 protein, 50- 60RBC/hpf, 0-1 WBC/hpf and occasional red cell casts. Serum creatinine is elevated 2mg/dL from a previously normal level of 0.9 mg/dL 1 week ago. If this is a case of SLE, which of the following is the most common and severe histopathologic type? A. Mesangial GN B. Focal proliferative GN C. Membranous GN D. Difffuse proliferative GN E. Cresenteric GN

103 D. Page 125 of Topnotch Handout. The most common and severe type of Lupus nephritis is Type IV: Diffuse proliferative GN which has a wire-loop capillary appearance.

Early chronic myeloid leukemia (CML) and leukemoid reaction (LR) sometimes show similar histological pictures. To differentiate between a leukemoid reaction and CML the following should be requested: A. Neutrophil alkaline phosphatase B. C Reactive Protein C. Cytogenetic testing D. A and B only E. All of the above

114 E. Page 30 of Topnotch Handout. CML must be differentiated from leukemoid reactions in which there is a marked increase in myeloid elements secondary to infection, chronic inflammation and other causes. Both present with extreme leukocytosis however CML has low NAP and CRP. Cytogenetic testing will determine the presence of the BCR-ABL gene.

This type of Hodgkin's lymphoma which has an intermediate prognosis, has a highly significant association with EBV infection and numerous R-S cells in a mixed inflammatory background that obliterates the normal architecture: A. Lymphocyte depleted B. Lymphocyte predominant C. Lymphocyte rich D. Mixed cellularity E. Nodular sclerosis

115 D. Page 32 of Topnotch Handout. Refer to the table on types of hodgkin's lymphoma. Mixed cellularity involved lymph nodes are diffusely effaced by a heterogenous cellular infiltrate. Plentiful RS cells admixed with lymphocytes. It has an intermediate prognosis and EBV-infected in 70% of cases.

Penile carcinoma in situ has a strong association with: A. HPV 6 B. HPV 11 C. HPV 16 D. HPV 18 E. HPV 31

119 C. SIMILAR TO PREVIOUS BOARD EXAM CONCEPT/PRINCIPLE. Page 52 of Topnotch Handout. Penile Carcinoma in situ is strongly associated with HPV 16 infection.

Psammoma bodies are frequently encountered in the following conditions except? A. Papillary thyroid CA B. Serous cystadeno CA C. Meningioma D. Mesothelioma E. Medullary thyroid CA

120 E. Page 59 of Topnotch Handout. PSaMMoma bodies are encountered in A-D.

Which of the following is an effect of PAF? A. Vasodilation B. Bronchoconstriction C. Decreased vascular permeability D. Decreased platelet aggregation E. Decreased leukocyte adhesion

121 A. or B. Platelet activating factor is a phospholipid-dervided mediator that has multiple inflammatory effects. It cause platelet aggregation, vasoconstriction, bronchoconstriction, increased venular permeability, increased leukocyte adhesion to endothelium and chemotaxis. (Robbin's) PAF causes bronchoconstriction and also vasodilation in low doses. SIMILAR TO PREVIOUS BOARD EXAM CONCEPT.

Achalasia is characterized by: A. Increased resting tone of LES B. Lack of esophageal peristalsis C. Incomplete LES relaxation during deglutition D. All of the above

423 D.

What is the most frequent type of hyperfunctioning pituitary adenoma? A. Gonadotroph adenoma B. Somatotroph adenoma C. Lactotroph adenoma D. Corticotroph adenoma

424 C.

Which of the following is incorrectly matched? A. Bronchopneumonia: Patchy multifocal consolidation B. Lobar pneumonia: Lobar consolidation C. Atypical pneumonia: Inflammation within alveolar interstitium D. AOTA E. NOTA

44 E. All of them are correct

A male infant was born at term. No congenital anomalies were noted at birth. About 6 months later, he was noted of failure to thrive and has been getting bacterial pneumonia with Hemophilus influenzae and Streptococcus pneumoniae cultured from his sputum. You strongly suspect that he has: A. Selective IgA deficiency B. Severe Combined Immunodeficiency C. EBV Infection D. X-linked Hypommaglobulinemia E. Di-George Syndrome

445 D. X-linked hypogammaglobulinemia or Bruton's agammaglobulinemia occurs due to low levels of all immunoglobulins, a virtual absence of B cells due to tyrosine kinase mutation. Male infants at about 6 months of age present with recurrent pyogenic bacterial infections caused by Strep pneumoniae and Haemophilus influenzae.

A pigeon raiser suddenly developed stiff neck and deteriorating sensorium was rushed to the ER and was admitted. Work-up was done for probable diagnosis. Lumbar puncture was done and CSF was obtained and should be stained with: A. Acid fast stain B. Methylene blue C. India ink stain D. Gram Stain E. H&E Stain

446 C. Cryptococcus neoformans is a type of fungus that is found in the soil worldwide, usually in association with bird droppings. Cryptococcal meningitis is believed to result from dissemination of the fungus from either an observed or unappreciated pulmonary infection. India ink of the CSF is a traditional microscopic method of diagnosis.

A 5-year old was brought in at the out-patient department because of generalized edema accompanied by easily pluckable hair, listlessness and loss of appetite. The child is most likely suffering from: A. Anorexia B. Marasmus C. Kwashiorkor D. Bulimia E. Vitamin Deficiency

447 C. Kwashiorkor is caused by protein deprivation greater than reduction in caloric intake. It is the most common form of protein-energy malnutrition which is associated with severe loss of the visceral protein compartment. Clinical findings include generalized or dependent edema, skin lesions, hair changes and enlarged, fatty liver.

Leading cause of blood transfusion related mortality A. Transfusion related acute lung injury B. Graft versus host reaction C. Hemolytic transfusion reaction D. Sepsis E. Anaphylaxis

45 A. TRALI is currently the leading cause of transfusion mortality. Donor antibodies to recipient leukocyte antigens activate complement and encourage granulocytes to aggregate within the pulmonary microvasculature. The diagnosis is based on clinical findings that are similar to ARDS but arise within 6 hours after transfusion. TRALI has a 5â€"10% mortality rate, but with interim ventilatory support, most symptoms resolve within 96 hours.

A 55 year-old man was suffering from recurrent attacks of persistent cough. He was rushed to the Emergency Room due to severe defficulty of breathing. PE revealed wheezes over lung fields. Mucous plugs were collected and histologically examined revealing collections of cystalloid made up of eosinophil membrane protein. These are: A. Charcot-Leyden Crystals B. Reinke Crystals C. Curschmann spirals D. Councilman Bodies E. Psammoma Bodies

450 A. Charcot-Leyden crystals are collections of crystalloid made up of eosinophil lysophospholipase binding protein called galectin-10. Curshmann spirals are mucus plugs containing whorls of shed epithelium. These result from mucus plugging in subepithelial mucus gland ducts or bronchioles which later become extruded.

Compromise of heart function because the pericardium is stiff is: A. Hypertrophic cardiomyopathy B. Restrictive cardiomyopathy C. Cardiac tamponade D. Constrictive pericarditis E. Fibrinous pericarditis

455 D. Constrictive pericarditis is diagnosed when heart is completely encased by a dense fibrosis that it cannot expand normally during diastole. Fibrinous pericarditis is seen in patients with uremia and or viral infection. The exudate imparts an irregular appearance to the pericardial surface (bread and butter pericarditis).

A 2-month old infant, first born male was brought to the Emergency Room for persistent nonbilious projectile vomiting. Physical examination revealed a palpable olive-shaped mass on the abdomen. The most probable diagnosis is: A. Duodenal atresia B. Intussusception C. Volvulus D. Hypertrophic pyloric stenosis E. Hirschprung's disease

458 D. Congenital hypertrophic pyloric stenosis is a condition which causes projectile non-bilious vomiting. It most often occurs in the first 2 to 6 weeks of life. The pyloric hypertrophy is felt classically as an olive- shaped mass in the middle upper part or right upper quadrant of the infant's abdomen.

A 21 year old male noticed a painless testicular mass. If it is a testicular tumor, what lymph node is directly involved in its lymphatic spread? A. Para-aortic nodes B. Inguinal nodes C. Obturator nodes D. Iliac nodes E. All of the choices

472 A. The route of lymphatic spread of testicular tumor is as follows: para-aortic nodes > mediastinal nodes > supraclavicular nodes

An ICU patient has "muddy brown" casts on urinalysis. This finding is pathognomonic for: A. Acute tubular necrosis B. Nephrotic syndrome C. Acute pyelonephritis D. Acute glomerulonephritis

498 A. AGN presents with RBC casts, nephrotic syndrome with Fatty casts and Acute pyelonephritis with WBC casts.

The leather-bottle appearance of the gastric adenocarcinoma is characteristic of this histologic type. A. Intestinal B. Diffuse C. Nodular D. Esophageal E. Squamous

50 B. Linitis plastica refers to a thickened “leather-bottle†appearance of the stomach caused by diffuse infiltration of the gastric wall by a poorly differentiated signet-ring type of adenocarcinoma.

A 45.F presents with a 4-day history of an acutely painful right sided goiter. She has no previous history of thyroid disease; and clinically appears euthyroid. Thyroid gland biopsy would reveal disruption of thyroid follicles with extravasation of colloid leading to a polymorphonuclear infiltrate. A granulomatous reaction may be seen. What is the most likely diagnosis? A. Subacute lymphocytic thyroiditis B. Hashimoto's thyroiditis C. DeQuervain's thyroiditis D. Grave's disease E. Colloid adenomatous goiter

549 C. Remember the classic cases! Middle-aged woman with PAINLESS goiter and hypothyroid sx = Hashimoto's. Postpartal woman with PAINLESS goiter and signs of thyrotoxicosis = Subacute lymphocytic thyroiditis.

What is the average weight of the bone marrow of an adult individual? a. 1000 grams b. 1500 grams c.2000 grams d. 1750 grams

570 B. 1200-1500 grams as per Henry’s diagnostics

Caspases are noted for their cysteine content and the ability to cleave aspartic acid residues. They are hence involve in programmed cell death. Which caspase acts as the initiator caspase in humans under the extrinsic pathway? a. Caspase 8 b. Caspase 9 c. Caspase 10 d. Smac/DIABLO

571 C. caspase 8 in the nematode C. elegans but Caspase 10 in humans p30 robbins

Most common site of AV malformation in the brain? a. ACA b. MCA c. PCA d. Basilar artery

575 B. MCA as per robbins p1299

Flexner-wintersteiner rosettes is associated with a. Neuroblastoma b. Retinoblastoma c. Medulloblastoma d. Glioblastoma

576 B.

Cardiac hypertrophy is noted with ventricular wall thickness of about: a. 0.5cm b. 1cm c. 1.5cmcm d. 2cm

579 D. 2cm; normal thickness is 1-1.5cm

What cytokine mediates septic shock and plays a role in cancer cachexia? A. TNF alpha B. IL-1 C. Inteferon alpha D. Inferterferon gamma E. Lipopolysaccharide

62 A. SIMILAR TO PREVIOUS BOARD EXAM CONCEPT/PRINCIPLE/ TNF alpha is believed to be the cytokine that mediates septic shock and is implicated in cachexia of malignancy.

In lung carcinoma, distant metastasis is most commonly found in: A. Liver B. Brain C. Bone D. Adrenals E. Kidney

614 D. Adrenals is the most common organ of distant metastasis of a lung carcinoma (50% of the time).

The major risk factor/s for developing breast carcinoma is/are: A. Age B. Family history C. Hormonal D. B and C E. All of the above

617D.The major risk factors for the development of breast cancer are hormonal and genetic (family history)

During your patho rotation, the 3rd year resident showed you a slide containing large cells with multiple nuclei, some have single nucleus with multiple nuclear lobes (each half is a mirror image of the other). With this very limited information, it is safe to say that the patient whom this specimen belongs to is suffering from? A. Chronic Lymphocytic Leukemia B. Hairy Cell Leukemia C. Non-hodgkin's lymphoma D. Hodgkin's lymphoma E. Leukemoid reaction

646 D. the description given points out to the REED- STERNBERG cells which are the pathognomonic for Hodgkin's lymphoma

Which of the following is most likely associated with the Philadelphia chromosome? A. Found in AML B. t (9;23) C. BCR-ABL gene D. Good prognosis

683 C. Answer: C. BCR-ABL gene

10. A previously healthy neonate presented with tachypnea, retractions, and cyanosis. CRP was elevated. What is the most likely diagnosis? a. RDS type I b. RDS type II c. ARDS d. BPD e. none of the above

90 A. Robbins 8th edition p.456

14. One week prior to consult, an 18 year old nursing student came in for complaints of impacted wisdom tooth on the right side of the mandible. After 24 hours, a painful neck mass was noted. Which of the following morphology of the neck mass may undergo necrosis and pus formation? a. follicular hyperplasia b. paracortical hyperplasia c. acute lymphadenitis d. reticular hyperplasia e. none of the above

94 C. Nodes involved in acute lymphadenitis are enlarged and painful. When pyogenic organisms are the cause, the centers of the follicles may undergo necrosis. Choices A, B, and D refer to chronic nonspecific lymphadenitis. (robbins 8th edition p.595)

15. What stage of inflammatory response of the lung is described when the lung is heavy, boggy, and red? a. congestion b. red hepatization c. gray hepatization d. resolution e. none of the above

95 A. It is characterized by vascular engorgement, intra- alveolar fluid with few neutrophils and often the presence of numerous bacteria. In red hepatization, the lobe now appears red, firm, and airless with a liver - like consistency. (robbins 8th edition p. 713)

What is the most common cause of nephritic syndrome in children? A. Hepatitis B B. Intake of toxic substances C. Antecedent GABHS infection D. Viral infections E. None of the above

152 C.

Which of the following is the most important prognosticating factor in cases of breast carcinoma? A. HER-2/neu status B. Family history C. Tumor histology D. Age E. Axillary node status

395 E. SIMILAR TO PREVIOUS BOARD EXAM CONCEPT/PRINCIPLE

Which of the following is the most common benign neoplasm of the liver? A. Hepatic adenoma B. Cavernous hemangioma C. Focal nodular hyperplasia D. Lipoma E. Biliary cystadenoma

396 B.

A presence of necrosis with nuclear changes involving basophilia of the chromatin which later on fades or dissolves is called? A) Karyolysis B) Pyknosis C) Karyorrhexis D) Histiocytosis

539 A. Karyolysis>> basophilia of the chromatin fades or dissolves

Which of the following is least likely to facilitate chemotaxis: A. LTB4 B. IL8 C. C5a D. TGF E. N-formylmethionine

1 D. TGF - transforming growth factor. Questions in the boards can sometimes be phrased as “least likely or most likelyâ€, so try to accustom yourself to choosing the best answer in such circumstances. Source: Robbins and Cotran Pathologic Basis of Disease 8th ed p. 50

A 40 y/o female was found to have a diastolic rumble at the apex. The chest x-ray showed uplifting of the left main stem bronchus, retrosternal fullness, and dilated main pulmonary artery. What is the most likely diagnosis? A. Mitral Stenosis B. Mitral Regurgitation C. Aortic Stenosis D. Pulmonic Stenosis E. Aortic Regurgitation

104 A. Page 112 of hand out. Mitral stenosis causes an increase in LA size due to restriction of blood flow from the left atrium to the left ventricle as a result of a narrowed mitral passage. Murmurs associated with MS are opening snap, accentuated S1, and diastolic rumble. Plain film chest xray findings are cardiomegaly, double right heart border (enlarged left atrium and normal right atrium), prominent left atrial appendage and splaying of the subcarinal angle (>120 degrees).

What is the expected thyroid function test in primary hyperthyroidism? A. decreased TSH, increased T4 B. decreased TSH, decreased T4 C. increased TSH, increased T4 D. increased TSH, decreased T4 E. None of the above

105 A. Page 71 of Topnotch Handout. Primary hyperthyroidism is the term used when the pathology is within the thyroid gland. Secondary hyperthyroidism is the term used when the thyroid gland is stimulated by excessive thyroid-stimulating hormone (TSH) in the circulation.The diagnosis of hyperthyroidism is confirmed by blood tests that show a decreased thyroid-stimulating hormone (TSH) level and elevated T4 and T3 levels. A low TSH level typically indicates that the pituitary gland is being inhibited or "instructed" by the brain to cut back on stimulating the thyroid gland, having sensed increased levels of T4 and/or T3 in the blood.

What can generally differentiate between asthma and COPD in spirometry? a. FEV1 b. reversibility c. PEF d. TLC E. FVC

106 B. Page 115of Topnotch Handout. Asthma and COPD are both obstructive lung diseases. On spirometry FEV1, FVC and FEV1/FVC are decreased in both how ever asthma is a reversible condition.

Ranson’s criteria is used in acute panceatitis for: A. Diagnosis B. Monitor abscess formation C. Predict possible etiology D. Decide surgical intervention E. Prognostication

107 E. Page 101 of Topnotch Handout. Ranson's criteria is a criteria for prognostication on admission and for the first 48 hours.

Which collagen type is commonly found in the dermoepidermal junction and is usually defective in Epidermolysis bullosa: A. Type I B. Type 3 C. Type 5 D. Type 7 E. Type 9

108 D. Page 8 of Topnotch Handout. Epidermolysis bullosa is caused by genetic defects (or mutations) within the human COL7A1 gene encoding the protein type VII collagen (collagen VII).

In a patient with pulmonary embolism the most common ECG manifestation that should be expected is? A. Incomplete right bundle branch block B. Sinus tachycardia C. Right ventricular strain pattern D. S1Q3T3 E. Junctional rhythm

109 B. Page 11 of Topnotch Handout. The most common ECG manifestation is a sinus tachycardia. Right ventricular strain pattern or S1Q3T3 is found in only 6% of patients.

A 40 yr old male presents with fever and cough of 5 days duration. PE revealed increased bronchial breath sounds over the lower segment of the right lobe posteriorly. Chest xray showed lobar consolidation on the right lower lobe and culture was positive for pneumonococcus. Which is the prominent inflammatory cells of this exudate? A. Platelets B. Basophils C. Eosinophils D. Neutrophils E. Macrophage

11 D. Patient has bacterial pneumonia. Streptococcus is a popular agent for causing lobar pneumonia. Stages of inflammatory response in lobar pneumonia: - Congestion: red, heavy, boggy lung - Red hepatization: massive, confluent exudation with neutrophils, red cells and fibrin - Gray hepatization - Resolution Source: Robbins and Cotran Pathologic Basis of Disease 8th ed p.712

A patient diagnosed with drug-induced lupus would most likely be positive for which autoantibody? A. Anti-SS-A B. Anti-centromere C. Anti-histone D. Anti-dsDNA E. Anti-mitochondrial

110 C. Page 20 of Topnotch Handout. A patient with drug induced lupus will be positive to anti-histone. A- Sjorgren syndrome, B-CREST syndrome, D-SLE, E- Primary biliary cirrhosis

A 59 y/o male, received a blood group identical living unrelated kidney graft. During surgery no abnormalities occurred. Four hours after the transplantation, it was noted that diuresis suddenly decreased. Upon repeat laparotomy the transplanted kidney showed signs of hyperacute rejection and had to be removed. Which pathological examination findings are consistent with hyperacute rejection? A. Thrombosis of capillaries and fibrinoid necrosis in arterial walls B. Presence of interstitial mononuclear cell infiltration and edema C. Necrotizing vasculitis withendothelial cell necrosis D. Interstitial fibrosis and tubular atrophy with loss of renal parenchyma E. Neutrophilic infiltration, deposition of Ig, complement and fibrin

111 A. Page 23 of Topnotch Handout. Hyperacute rejection occurs within minutes to hours due to preformed antibodies. It is a Type II hypersensitibity reaction. Morphological features: thrombotic occlusion of capillaries and fibrinoid necrosis occurs in arterial walls.

A 32-day-old female infant was admitted due to delayed umbilical cord detachment and omphalitis. After admission, CBC revealed severe leukocytosis, and there was poor clinical response to several kinds of antibiotics. She is diagnosed to have aan immunodefiency disease specifically a phagocyte disorder. What is the most probable pathophysiologic mechanism of this disease? A. Mutation in WASP gene for actin filament assembly B. Mutation in DNA repair enzymes C. Lack of NADPH oxidase activity D. Failure of phagolysosomal fusion E. Defective LFA-1 proteins

112 E. Page 25 of Topnotch Handout. This is a case of Leukocyte adhesion deficiency syndrome it is an autosomal recessive disease involving a mutation in integrins hence defective adhesion LFA-1 proteins on the surface of phagocytes. It is commonly manifested bu severe pyogenic infections in infancy and delayed cord separation.

Which electrolyte abnormality is not usually found in tumor lysis syndrome? A. Hypocalcemia B. Hypercalcemia C. Hyperphosphatemia D. Hyperuricemia E. Hyperkalemia

113 B. Page 29 of Topnotch Handout. Tumor lysis syndrome is an oncologic emergency that is caused by massive tumor cell lysis with the release of large amounts of potassium, phosphate, and nucleic acids into the systemic circulation.

Cold agglutinin disease is a form of autoimmune hemolytic anemia caused by cold- reacting autoantibodies. Autoantibodies bind to the erythrocyte membrane leading to premature erythrocyte destruction. Which antibody is commonly involved? A. IgM B. IgG C. IgA D. IgD E. IgE

116 A. Page 38 of Topnotch Handout. IgM antibodies generally cause cold agglutinin disease. M-Malamig!

Incomplete excision of a dentigerous cyst may result in a neoplastic transformation, specifically into a/an: A. Odontoma B. Ameloblastoma C. Basal cell carcinoma D. Cholesteatoma E. Paraganglioma

117 B. Page 42 of Topnotch Handout. Dentigerous cyst originates around the crown of an unerupted tooth, often associated with an impacted third molar. Complete excision is curative, however incomplete excision may result to recurrence or neoplastic transformation into an ameloblastoma or a squamous cell carcinoma.

A primary melanoma located in which part of the body has the worst prognosis? A. Sole B. Palm C. Scalp D. Chest E. Back

118 C. SIMILAR TO PREVIOUS BOARD EXAM CONCEPT/PRINCIPLE. Page 44 of Topnotch Handout. Anatomic location of the primary melanoma is an important independent predictor of SLN status and prognosis. Patients with primary melanomas of the head/neck and trunk have a worse prognosis than primary melanomas of other anatomic locations.

19 year old male presents with acute onset hematuria, oliguria and periorbital edema which is worse in the morning and gradually improves through the day. On PE, blood pressure is elevated and urinalysis shows trace proteinuria, and many RBCs. ASOT is negative and DNAse is positive. Which is the expected finding in light microscopy for this patient? A. Diffuse endocapillary proliferation B. Subepithelial humps C. Normal appearing D. Dense deposits E. Extracapillary proliferation of crescents

12 A. When you study for patho and all basic sciences, try to make sure you know how the disease will present clinically cause they like to correlate things. The hardcore micro and biochem or patho knowledge will only get you halfway cause sometimes they will not give you the diagnosis. This is a case of PSGN. ASOT may be negative if the nephritic strain comes from a skin infection because skin lipids bind to streptolysin O. DNAse is the most sensitive test for skin infection with Group A streptococcus. The patient in this case presented with the classic nephritic syndrome of hypertension, hematuria and oliguria. A and B are both PSGN but B is an electron microscopy finding. Source: Robbins and Cotran Pathologic Basis of Disease 8th ed p.918

What is the mechanism of edema in nephrotic syndrome? A. Glomerular injury B. Increased capillary permeability to proteins C. Decreased protein absorption D. Tubulointerstitial disorder E. Increased hydrostatic pressure

124 B. Increased permeability to plasma proteins resulting from either structural or physicochemical alteration allows protein to escape from the plasma into the urinary space. Massive proteinuria depletes serum albumin resulting in hypoalbuminemia, and thus decreased colloid osmotic pressure of the blood with subsequent accumulation of fluid in the interstitial tissues. Sodium and water retention due to compensatory secretion of aldosterone and stimulation of sympathetic system also contributes and aggravates the edema. SIMILAR TO PREVIOUS BOARD EXAM CONCEPT.

A 24-year old male patient presented with cherry-red spots in the macula. Morphology of the brain shows neurons ballooned with cytoplasmic vacuoles. What enzyme is deficient in this condition? A. Alpha 1,4-glucosidase B. Sphingomyelinase C. Arylsulfatase D. Iduronidase E. Hexosaminidase

125 E. This is a morphologic description of Tay-Sachs Disease, a deficiency of hexosaminidase. SIMILAR TO PREVIOUS BOARD EXAM CONCEPT.

A 48-year old male patient who underwent kidney transplant suddenly developed bloody urine few hours after the procedure. Morphologic changes in this pattern of rejection will reveal: A. Extensive interstitial mononuclear cell infiltration and edema B. Thrombotic occlusion of capillaries and fibrinoid necrosis C. Necrotizing vasculitis with endothelial cell necrosis D. Neutrophilic infiltration and deposition of immunoglobulin, complement, and fibrin E. Interstitial fibrosis and tubular atrophy with loss of renal parenchyma.

126 B. Hyperacute rejection occurs few minutes to few hours after transplant. Option A describes Acute cellular rejection. Options C and D are mophologic findings found in acute humoral rejection. Option E describes the morphology of kidney in chronic rejection.

What is the hallmark of tissue repair? A. Vasoconstriction B. Blood clot formation C. Granulation tissue D. Tissue remodeling E. Wound contraction

127 C. SIMILAR TO PREVIOUS BOARD EXAM CONCEPT.

The first step in phagocytosis is: A. Formation of phagocytic vacuole B. Degradation of ingested material C. Recognition of particle D. Attachment E. Engulfment

128 C. Steps in phagocytosis: 1. Recognition and attachment; 2. Engulfment and formation of phagocytic vacuole, 3. Killing and degradation of ingested material. SIMILAR TO PREVIOUS BOARD EXAM CONCEPT.

An 18-year old male patient presents with easy fatigability, fever, and cutaneous bleeding. Bone marrow biopsy showed 40% myeloblast. What is the most likely diagnosis? A. ALL B. AML C. CML D. Burkitt's lymphoma E. Adult T cell lymphoma

129 B. The diagnosis of AML is based on the presence of at least 20% myeloid blasts in the bone marrow. Robbins 9th ed., p. 613 . The most common manifestation of AML include fever, easy fatigability and bleeding. SIMILAR TO PREVIOUS BOARD EXAM CONCEPT.

Patient sought consult for chest pain and hemoptysis associated with hematuria, and signs and symptoms of uremia. His blood pressure is elevated and he has grade III bipedal edema. As the clinician in charge, you suspect that he is suffering from an autoimmune disease in which antibodies against type IV collagen attack the basement membrane of the lungs and kidneys. On immunofluorescence renal biopsy would most probably reveal: A. Granular IgG and C3 in GBM and mesangium B. Linear IgG and C3 C. Negative D. Focal IgM or C3 E. IgA in the mesangium

13 B. Don’t just focus on the electron and LM findings and sacrifice immunofluorescence. SIMILAR TO PREVIOUS BOARD EXAM CONCEPT/PRINCIPLE. A rule of thumb is if the pathophysiology involves immune complex deposition, it would usually present with a granular pattern. If the pathophysiology involves antibodies against basement membranes, the pattern is usually linear. This is actually a case of Goodpasture syndrome which involves antibodies against the basement membrane of the lungs and the kidneys and the findings on immunofluorescent studies show linear IgG and C3. Source: Robbins and Cotran Pathologic Basis of Disease 8th ed p. 709, 918

Morphologic finding/s in alcoholic hepatitis include: A. Councilman bodies B. Hepatocyte swelling and necrosis C. Lymphoid aggregates within portal tracts D. Hepatocyte apoptosis E. All of the above

130 B. The rest are features of viral hepatitis. SIMILAR TO PREVIOUS BOARD EXAM CONCEPT.

The most common cause of sudden cardiac death in Myocardial infaction is A. Congestive heart failure B. Ventricular fibrillation C. Pulmonary edema D. Acute pericarditis E. Ventricular rupture

131 B. SIMILAR TO PREVIOUS BOARD EXAM CONCEPT.

Juxta-articular osteopenia is characteristic of A. Systemic lupus erythematosus B. Rheumatoid arthritis C. Osteoarthritis D. Ankylosing spondylitis E. Gouty arthritis

132 B. SIMILAR TO PREVIOUS BOARD EXAM CONCEPT.

True of female hemophilia carrier A. Both X chromosomes are defective B. Decrease Factor VIII C. 25% of her offspring are affected D. One of the X chromosome shows abnormality E. Transmits disease to half her sons and half her daughters

133 D. Hemophilia is an X-linked recessive disorders wherein heterozygous female or female carriers does not express full phenotypic change because of paired normal allele, with random inactivation of one of the X chromosome leading to variability. Only 1 of the X chromosome is abnormal. Decrease in Factor VIII is a manifestation of affected male offspring. Option C is characteristic of AR while Option E is a property of X- Linked Dominant disorders.

An 8-year old male presented with sunburn- like rash that spread over the entire body and evolves into fragile bullae. Desquamation of epidermis follows at the level of granulosa layer. This is most likely caused by: A. Staphylococcus aureus B. Streptococcus pyogenes C. Viridans streptococcus D. Drug hypersensitivity E. Pseudomonas aeruginosa

134 A. This is a case of Staphylococcal scalded-skin syndrome or Ritter disease caused by S. aureus. It is distinguished from toxic epidermal necrolysis or Lyell’s disease which is secondary to drug hypersensitivity and causes desquamation at the level of epidermal-dermal junction.

A 62-year old female presented with fever and cough for the past few days. She also had necrotizing oval lesion on her extremities. The most likely etiology is: A. Staphylococcus aureus B. Streptococcus pyogenes C. Clostridium perfringens D. Bacillus anthracis E. Pseudomonas aeruginosa

135 E. Pseudomonas causes necrotizing pneumonia, vasculitis accompanied by thrombosis and hemorrhage. It proliferates widely, penetrating deeply into the veins and spreads hematogenously. Ecthyma gangrenosum, well-demarcated necrotic and hemorrhagic oval skin lesion, often appear.

The most likely renal pathology in multiple myeloma is: A. Tubulo-interstitial nephritis B. Membranoproliferative glomerulonephritis C. Acute glomerulonephritis D. Focal segmental glomerulosclerosis E. Tubular necrosis

136 A. Renal insufficiency in multiple myeloma can be due to Bence Jones proteins which are directly toxic to epithelial cell, and accumulation of light chains which results to tubulo-interstitial nephritis. Other causes include hypercalcemia nad hyperuricemia.

A 45-year old male, smoker, presented with cough, copious sputum, and progressive dyspnea on exertion for the last two years. On physical examination, he has wheezes all over his lung fields, and cyanotic lips. Morphologic findings of the lungs in this condition will most likely show a/an: A. Abnormally large alveoli separated by thin septa and deformed respiratory bronchioles. B. Enlargement of the mucus-secreting glands of the bronchi and hyperemia and edema of the mucus membranes. C. Dilatation of bronchi and bronchioles caused by destruction of the muscle and elastic tissue. D. Thickening of the basement membrane, edema and inflammatory infiltrate in the bronchial walls. E. Cobblestone pleural surfaces and patchy interstitial fibrosis varying in intensity.

137 B. Chronic cough, copious sputum in a smoker points to chronic bronchitis as the diagnosis. Option A - emphysema. Option C - Bronchiectasis. Option D - Asthma; Option E - Idiopathic interstitial fibrosis. SIMILAR TO PREVIOUS BOARD EXAM CONCEPT.

The most common malignancy of the stomach: A. Carcinoid B. Lymphoma C. Adenocarcinoma D. Squamous cell carcinoma E. Gastrointestinal stromal tumor

138 C. SIMILAR TO PREVIOUS BOARD EXAM CONCEPT.

A 57-year old, obese male was brought to the hospital because of sudden onset of chest pain, characterized as squeezing, accompanied by diaphoresis and nausea. Neutrophils were noted to be elevated. What is the pathophysiologic mechanism of his condition? A. Tissue necrosis B. Alveolar edema C. Infection D. Inflammation of airways E. B and C

139 A. This is a case of acute myocardial infarction. Myocardial necrosis begins at approximately 30 minutes after coronary occlusion. SIMILAR TO PREVIOUS BOARD EXAM CONCEPT.

Patent urachus is a risk factor for which type of carcinoma of the bladder? A. Transitional cell carcinoma B. Squamous cell carcinoma C. Adenocarcinoma D. Clear cell carcinoma E. Sarcoma

14 C. Transitional cell carcinoma is associated with smoking. Squamous cell carcinoma is associated with chronic irritation and S. haematobium infection. Source: Robbins and Cotran Pathologic Basis of Disease 8th ed p. 979

Thrombosis of what vessel will most most likely involve the posterior portion of the ventricular septum? A. Left anterior descending coronary artery B. Left circumflex coronary artery C. Marginal branch of left circumflex artery D. Right coronary artery E. Left main coronary artery

140 D. Right coronary artery (30-40%) infarct involves the inferior/posterior wall of LV, posterior portion of ventricular septum, inferior/posterior RV free wall in some cases; LADA involves the anterior wall of LV near apex, anterior portion of ventricular septum and apex circumferentially; Left circumflex artery involves the lateral wall of left ventricle except the apex. (Robbin's)

A 30 year old male admitted for dengue fever on the 7th day of illness develops pleural effusion, what is the explanation for the pleural effusion? A. Decreased plasma oncotic pressure B. Increased capillary permeability C. Increased capillary hydrostatic pressure D. Decreased platelets E. None of the above

141 B. Increased capillary permeability is the pathophysiologic mechanism of pleural effusion in dengue fever more than decreased platelets

JS, a 70 year old male who was a smoker of 40 pack years was diagnosed with squamous cell carcinoma of the lung, which among the following is the most common site of origin of this lesion? A. Trachea B. Secondary bronchus C. Primary bronchus D. Terminal bronchioles E. None of the above

142 C. The major (primary bronchus) is the most common site of development of squamous cell carcinoma of the lung

RLDL, a 40 year old female was a diagnosed case of SLE, she had a renal biopsy done and results were brought to you, you know that the most common type of renal lesion of lupus has: A. Mesangial lupus glomerulonephritis B. Diffuse proliferative glomerulonephritis C. Membranous glomerulopathy D. Focal proliferative glomerulopathy E. None of the above

143 B. Diffuse proliferative GN is the most common as well as the most severe form of glomerulonephritis

What is the cytokine that stimulates collagen synthesis A. TNF B. IL-1 C. VEGF D. TGF-B E. PDGF

144 D. TGF-B stimulates collagen synthesis, IL-1 stimulates fever as well as TNF, VEGF causes angiogenesis, PDGF causes collagenase secretion.

MIV, a 48 year old female underwent Pap smear, which of the following cytologic findings suggest the presence of HPV infection A. Atypical ductal cells B. Diffuse thickening of the basement membrane C. Koilocytic atypia D. A and B E. None of the above

145 C. Koilocytic atypia is a characteristic finding in the pap smear which suggests HPV infection

RF, a 30 year old male underwent lymph node biopsy, the result showed a starry sky pattern, you know that his malignancy is associated with? A. Hepatitis B infection B. Cytomegalovirus C. Herpes zoster D. Epstein Barr virus E. Infection with viruses belonging to filoviridae family

146 D. Starry sky pattern is seen in Burkitt's lymphoma which is associated with EBV infection

Which among the following is important in the pathology of HIV infection? A. CD8 B. CD4 C. Macrophages D. Dendritic cells E. Neutrophils

147 B. HIV invades the CD4 cells which are also responsible for the cell mediated immunity

A patient was diagnosed with melanoma, one of the following is a prognostic factor for melanoma except A. tumor depth B. presence of tumor infiltrating lymphocytes C. number of dendritic cells D. location of the tumor E. gender

148 C. The number of skin dendritic cells is not a prognostic factor for melanoma

BB, a 70 year old chronic alcoholic presents to you with changes in sensorium, you requested a liver biopsy, which of the following would you expect to see: A. Councilman bodies B. Mallory bodies C. Negri bodies D. Psamomma bodies E. None of the above

149 B. Mallory bodies are eosinophilic cytoplasmic clumps in hepatocytes signifying liver injury, councilman bodies are eosinophilic globules seen in acute hepatitis, Negri bodies are associated with rabies while psamomma bodies are associated with meningioma, prolactinoma, ovarian serous cystadenoma and papillary thyroid CA.

In which of the following nutritional deficiency is hepatic steatosis an expected finding? A. Vitamin A deficiency B. Vitamin E deficiency C. Copper deficiency D. Kwashiorkor E. Marasmus

15 D. Kwashiorkor is a protein deficiency while marasmus a balanced deficiency of all macronutrients. Source: Robbins and Cotran Pathologic Basis of Disease 8th ed p. 429

MBDM, a 29 year old male underwent a tissue section that showed 40% myeloid cells, what is the diagnosis? A. CLL B. AML C. ALL D. Mantle cell lymphoma E. Non-Hodgkins lymphoma

150 B. AML presents with >20% of myeloblasts in the bone marrow.

Most common bone involved in osteoporosis A. Pelvis B. Tibia C. Femur D. Vertebra E. Ribcage

151 D. The vertebra are the most commonly affected as it is a weight bearing bone.

KB, a 32 year old female fond of using beauty products tried a new product on her face, 5 days later, she noticed redness on the areas of application of the new product, what is the most likely explanation: A. IgE mediated immune reaction B. Cytotoxin mediated C. Immunoglobulin-antigen complex D. Cell mediated hypersensitivity E. None of the above

155 D. Type IV or delayed hypersensitivity develops usually after 48 hours on application of offending agent on the skin of the patient.

What is the mechanism of DIC in meningococcemia? A. Massive intravascular coagulation occuring everywhere in the circulation B. Endothelial dysfunction C. Depletion of clotting factors D. All of the above E. None of the above

156 D. All of the above are the mechanism of DIC

A 2 week old infant had been undergoing treatment for sepsis when the patient went into hypotension and died, what is the most likely explanation for the infant's death? A. DIC B. Sepsis C. Thrombocytopenia D. B and C E. None of the above

157 E. The most likely cause is septic shock due to systemic vasodilation.

The classic anaphylactic reaction is due to: A. Th1 B. Th2 C. NK cell D. Macrophage E. None of the above

158 B. SIMILAR TO PREVIOUS BOARD EXAM CONCEPT/PRINCIPLE. Th2 subset of helper T cells are overactive in patients who develop the classic anaphylactic reaction

Among the following adaptations of the body to injury, which of the following is a fertile ground for neoplasia? A. Hyperplasia B. Atrophy C. Hypertrophy D. Metaplasia E. None of the above

159 D. Metaplasia is defined as the change from one adult cell type into another and is a fertile ground for malignancies, examples of malignancies originating from metaplastic change include Barett's esophagus leading to esophageal adenoCA, cervical CA from squamous metaplasia of cervical columnar cells, SQCA of the lung due to squamous metaplasia of respiratory epithelium.

Which of the following is not a characteristic of a benign peptic ulcer? A. Sharply punched out defect B. Heaped-up margins C. Hemorrhage and fibrin deposition in the gastric serosa D. Perforation E. Malignant transformation is very rare

16 B. Benign ulcers may have mucosal margins which overhang the base slightly but is usually level with the surrounding mucosa. Heaped up margins are more characteristic of malignant ulcers. Source: Robbins and Cotran Pathologic Basis of Disease 8th ed p. 780

Valvular vegetations characrerized as small, warty vegetations along the lines of closure of the valve leaflets are usually seen in what disease? A. RHD B. Infective endocarditis C. Non bacterial thrombotic endocarditis D. Libman sacks endocarditis E. marantic endocarditis

163 A. IE are large irregular masses on the valve cusps that can extend onto the chordae. NBTE/marantic endocarditis are small bland vegetations usually attached at the line of closure. LSE are small or medium sized vegetation on either or both sides of the valve leaflets

Which of the following caspase is an example of executioner in apoptosis? A. 8 B. 9 C. 10 D. 6 E. 5

165 D. caspases 8, 9, 10 are initiators while caspases 6 and 3 are executioners

What is the most common and the most severe type of Lupus nephropathy? A. Type I B. Type II C. Type III D. Type IV E. Type V

167 D. type IV or the diffuse proliferative glomerulonephritis is the most common and most severe type.

Minamata disease is an environmental concern because it causes cerebral palsy, deafness, blindness, mental retardation and other major CNS defects in children in utero. Which of the following elements is the cause of the disease? A. arsenic B. lead C. mercury D. cadnium E. chromium

168 C. Mercury poisoning is the one associated with Minamata disease.

All of the following cancers are strongly associated with smoking except: A. Oral cavity B. pancreas C. esophagus D. bladder E. Breast

169 E. cancers of the lung, larynx, esophagus, pancreas, bladder, oral cavity are strongly associated with smoking.

Which of the following is the least likely characteristic of right-sided colonic malignancy? A. Adenocarcinoma B. Napkin ring constriction and luminal narrowing C. Desmoplastic response D. Liver metastasis E. None of the above

17 B. Napkin ring constrictions are characteristic of left sided colonic malignancy. Right sided lesions present as polypoid, exophytic masses. Left sided lesions present clinically as obstruction while right sided ones present as anemia. Source: Robbins and Cotran Pathologic Basis of Disease 8th ed p. 824

Among the types of Hodgkin's lymphoma, this has the highest association with EBV and also has the poorest prognosis. A. Lymphocyte-depleted B. Lymphocyte rich C. Lymphocyte predominant D. Nodular sclerosis E. Mixed cellularity

170 A. lymphocyte depleted has the highest association with EBV and also has the poorest prognosis.Nodular sclerosis is the most common and has excellent prognosis

Chromosome translocations are important in diagnosing and prognosticating certain types of cancers. Which of the following may be seen in Burkitt's lymphoma? A. t9:22 B. T8:14 C. T14:18 D. T11:14 E. T15:17

171 B. 9:22 is associated with CML. 14:18 is associated with Follicular lymphoma. 11:14 is associated with mantle cell lymphoma. 15:17 is AML M3

Which of the following types of vasculitis is highly associated with c-ANCA or the antibodies against proteinase-3? A. PAN B. Takayasu arteritis C. Wegener Granulomatosis D. Microscopic polyangitis E. Churg-Strauss syndrome

172 C. p-ANCA is associated with microscopic polyangitis and Churgstraus while c-ANCA is for Wegener Granulomatosis

A patient came in for second opinion about his newly diagnosed polyarteritis nodosa. All of the following statements are true about this disease EXCEPT A. Does NOT affect lungs B. Affects mainly young adults C. Usually involves medium sized arteries D. NOT associated with hepatitis B E. very responsive to steroid therapy and cyclophospahmide

173 D. 30% of patients with PAN are HBsAg positive. pathology topnotch handout page 106. PAN does not affect the lungs, affects mainly young adults, usually involves medium sized arteries, very responsive to steroid therapy and cyclophosphamide, and 30% of patients are HbsAg positive.

Which among the following arteries is commonly affected in Kawasaki disease? A. Abdominal Aorta B. Thoracic aorta C. Arch of the aorta D. Pulmonary artery E. Coronary arteries

174 E.

Which among the following medications is/are essential in treating a patient with Kawasaki disease? A. ASA B. IVIg C. Steroids D. A and B E. All of the above

175D.Steroids is contraindicated because it promote rupture of coronary vessels and aggration of the disease.

This is a primary malignant small round-cell tumor of the bone and soft tissue which usually involve the mutation of a gene in chromosome 22. Homer-Wright rosettes can be seen histologically and onion-skin appearance can be seen radiographically on patients with this disease. A. Ewing sarcoma B. Osteosarcoma C. Chondrosarcoma D. Giant Cell Tumor E. Fibrous Dysplasia

176 A.

Which of the following conditions will cause edema? A. Increase oncotic pressure in the vessels B. Decrease plasma volume in blood vessels C. Decrease oncotic pressure outside the vessels D. Decrease in plasma proteins E. None of the above

179 D. decrease in plasma proteins will decrease oncotic pressure within the vessel which can cause edema

What is the most common tumor in the stomach? A. adenocarcinoma B. GIST C. carcinoid D. adenoma E. lymphoma

181 D. SIMILAR TO PREVIOUS BOARD EXAM CONCEPT/PRINCIPLE.

True of Luetic aneurysm? A. Inflammation begins in the tunica adventitia B. With characteristic tree-barking appearance C. Involvement of aorta favors development of superimosed atheromatosisof the aortic root D. All of the above E. A and C

182 D. All are true regarding Syphilitic/Luetic aneurysm

Microscopically, the earliest change of systemic Hypertensive Heart Disease is? A. Increase in transverse diameter of myocytes B. Irregular cellular enlargement C. Irregular nuclear enlargement D. Interstitial fibrosis E. All of the above

183 A.

In acute Rheumatic Fever, inflammation and Aschoff bodies are commonly found in which layer of the heart? A. pericardium B. myocardium C. endocardium D. All of the above E. A and B

184 D. SIMILAR TO PREVIOUS BOARD EXAM CONCEPT/PRINCIPLE.. During acute RF, diffuse inflammation and Aschoff bodies may be found in any of the 3 layers of the heart- hence the lesion is called a PANCARDITIS.

What is the most severe form of α-thalassemia which is caused by deletion of all four α-globin genes? A. αThalassemia trait B. Silent carrier state C. Hydrops fetalis D. Hemoglobin H disease E. A and D

186 C. Hydrops fetalis is the most severe form of α- thalassemia. it is caused by deletion of all four α- globin genes. In the fetus, excess γ-globin chains form tetramers (hemoglobin Barts) that have such a high affinity for oxygen that they deliver little to tissues.

A 58 year old male presented with easy fatigability, anorexia and weight loss. On PE, there were noted lymphadenopathies and hepatosplenomegaly. CBC revealed leukocytosis, with absolute lymphocyte count 3000 per mm3. There were numerous small, round lymphocytes with scant cytoplasm which are frequently disrupted in the process of making smears. What is the diagnosis? A. ALL B. SLL C. CLL D. CML E. Multiple Myeloma

187 B. CLL and SLL differ only in the degree of peripheral blood lymphocytosis. Most affected patients have sufficient lymphocytosis to fulfill the diagnostic requirement for CLL (absolute lymphocyte count >4000 per mm3). In this condition, lymph nodes are diffusely effaced by an infiltrate of predominantly small lymphocytes 6 to 12 μm in diameter with round to slightly irregular nuclei, condensed chromatin, and scant cytoplasm. Some of these cells are usually disrupted in the process of making smears, producing so-called smudge cells.

What is the most common lesion of the salivary gland? A. Sialolithiasis B. Pelomorphic adenoma C. Warthin tumor D. Mucocele E. Sialadenitis

188 D. Mucocele is the most common lesion of the salivary glands and it results from either blockage or rupture of a salivary gland duct, with consequent leakage of saliva into the surrounding connective tissue stroma.

Acute hepatitis is characterized by what morphologic feature? A. Swollen hepatocytes with irregulaly clumped cytoplasmic organelles and large clear spaces B. Presence of feathery degeneration C. Deposition of fibrous tissue in the portal tracts and periportal septa D. Bridging inflammation and necrosis E. All of the above

189 A. With acute hepatitis, hepatocyte injury takes the form of diffuse swelling (“ballooning degenerationâ€;), so the cytoplasm looks empty and contains only scattered eosinophilic remnants of cytoplasmic organelles. Feathery degeneration is retention of biliary material causing foamy appearance of hepatocytes seen in cholestatic liver injury. C and D are charcteristics of chronic hepatitis.

A 22 year old male consults for ptosis and diplopia which usually begins to occur in the late afternoon and improves upon waking up after a long good sleep. Recently he also noted dysphagia and muscle weakness which also improves with rest. What is the pathophysiology of his condition? A. Antibodies to presynaptic Ca channel preventing release of Acetylcholine B. Antibodies to Acetylcholine receptors C. Autoimmune peripheral demyelination D. Inflammation of the spinal cord E. Patient appears to be faking it. He should be referred to psych.

19 B. This is a case of myasthenia gravis Antibodies to presynaptic Ca channel preventing release of Acetylcholine â€" Lambert Eaton myasthenic syndrome (paraneoplastic syndrome) Source: Robbins and Cotran Pathologic Basis of Disease 8th ed p. 1275

True statement about Crigler-Najjar Syndrome type I A. Liver morphology is normal B. There is decreased UGT1A1 enzyme activity C. It is generally mild with occasional kernicterus D. There is mutation in MRP2 E. Hyperbilirubinemia is of direct type

190 A. Crigler-Najjar Syndrome type I is an autosomal recessive condition wherein there is ABSENT UGT1A1 activity causing indirect hyperbilirubinemia. Liver pathology is normal and it is fatal in the neonatal period. MRP2 mutation is seen in Dubin-Johnson Syndrome

Hepatitis B carrier state is most commonly acquired via what mode of transmission? A. Heterosexual transmission B. Needle-stick injuries C. Vertical transmission D. Blood transfusion E. Homosexual transmission

191 C. In endemic regions such as Africa and Southeast Asia, spread of Hepatitis B from an infected mother to a neonate during birth (vertical transmission) is common. These neonatal infections often lead to a carrier state for life.

Variant of Renal Cell Carcinoma which is made up of pale eosinophilic cells, often with a perinuclear halo, arranged in solid sheets with a concentration of the largest cells around blood vessels? A. Clear cell CA B. Papillary CA C. Chromophobe CA D. Collecting Duct CA E. Urothelial CA

192 C. SIMILAR TO PREVIOUS BOARD EXAM CONCEPT/PRINCIPLE.. Chromophobe renal carcinoma is made up of pale eosinophilic cells, often with a perinuclear halo, arranged in solid sheets with a concentration of the largest cells around blood vessels.

This agent predisposes susceptible populations to develop prostate cancer? A. Nickel B. Chromium C. Cadmium D. Vinyl chloride E. Benzene

193 C. The uses of cadmium include yellow pigments and phosphors; found in solders; used in batteries and as alloy and in metal platings and coatings. It is associated with prostate cancer. Nickel is associated with nose, lung cancer; Benzene- Leukemia, Hodgkin lymphoma; Chromium- lungs; Vinyl chloride- Angiosarcoma, liver.

Metastasis unequivocally marks a tumor as malignant. What cancer does not metastasize? A. hepatoma B. seminoma C. lipoma D. glioma E. meningioma

194 D. All cancers metastasize except glioma and basal cell carcinoma. Lipoma does not metastasize. But the question asks what "cancer" does not metastasize. Lipoma is benign so it should be ruled out from the choices. Glioma is the correct answer

True of chronic bronchitis except A. Grossly, there may be hyperemia, swelling and edema of the mucus membranes B. The ratio of the number of mucus glands to the thickness of the wall is increased C. There is excessive mucinous to mucopurulent secretions layering the epithelial surfaces D. The major change is in the size of the mucus glands E. All are true

195 B. Although the numbers of goblet cells increase slightly, the major change is in the size of the mucous gland (hyperplasia). This increase can be assessed by the ratio of the thickness of the mucous gland layer to the thickness of the wall between the epithelium and the cartilage (Reid index).

20 year old male presented with history of allergic rhinitis, asthma and recurrent sinusitis. An autoimmune etiology is suspected. What is the most likely autoantibody involved in this condition? A. Anti-myeloperoxidase Ab B. Anti-proteinase 3 Ab C. Anti-endomysial Ab D. Anti-mitochondrial Ab E. Anti-saccharomyces Ab

196 A. Diagnosis for this case is Churg-Strauss Syndrome. It is a small-vessel necrotizing vasculitis classically associated with asthma, allergic rhinitis, lung infiltrates, peripheral hypereosinophilia, and extravascular necrotizing granulomas. The autoantibody implicated in this condition is the anti- myeloperoxidase antibody and p-ANCA. B-Wegener's; C- Celiac Disease; D- primary biliary cirrhosis; E- Crohn's disease.

What is the most common site of ectopic pancreas? A. duodenum B. jejunum C. ileum D. Meckel's diverticulum E. spleen

197 A. Aberrantly situated, or ectopic, pancreatic tissue is found in about 2% of careful routine postmortem examinations. The favored sites for ectopia are the stomach and duodenum, followed by the jejunum, Meckel diverticula, and ileum.

The following are anaphylotoxins A. C3a B. C4a C. C5a D. A and C only E. All of the above

2 E. The anaphylatoxins are components of the complement system which are involved in anaphylaxis. Source: Robbins and Cotran Pathologic Basis of Disease 8th ed p. 57

HSV 1 is the most common cause of viral encephalitis. Which is the least likely expected pathological finding? A. The encephalitis most severely affects the frontal lobe. B. The infection is necrotizing and often hemorrhagic in the most severely affected regions C. Cowdry type A intranuclear viral inclusion bodies may be found D. All of the above E. None of the above

20 A. Accustom yourself to answering question phrased as most likely or least likely. Examiners may want to phrase it that way. HSV1 encephalitis classically most severely affects the temporal lobe. It most commonly presents as alterations in mood, behavior and memory.

What is the immunologically mediated pathologic lesion seen in Acute Rheumatic Fever? A. Vascular dilation, edema, smooth muscle contraction B. Necrotizing vasculitis C. Phagocytosis and cell lysis D. Perivascular cellular infiltrates E. B and C

200 C. Acute Rheumatic Fever is a Type 2 hypersensitivity reaction. It is an antibody mediated reaction causing phagocytosis and lysis of cells; inflammation; in some diseases, functional derangements without cell or tissue injury. A- Type 1; B- type 3; D- type 4

Anti-centromere antibodies are present in: A. Sjogren syndrome B. SLE C. Wegener's granulomatosis D. CREST syndrome E. Churg-Strauss syndrome

202 D. CREST syndrome: calcinosis, Raynaud's phenomenon, esophageal dysmotility, sclerodactyly, and telangiectasia.

The presence of Heinz bodies and bite cells in a patient having hemolytic anemia strongly suggests: A. Sickle cell anemia B. G6PD deficiency C. Alpha thalassemia D. Multiple myeloma E. Hereditary spherocytosis

203 B. Heinz bodies: RBCs with denatured hemoglobin. Bite cells results when splenic macrophages pluck out these inclusions.

A 4-year-old boy presents with recurrent joint pain involving the knees and hips. He had always bruised easily, and recently the parents had seen blood in his urine. A presumptive diagnosis of classic hemophilia (hemophilia A) is made, and coagulation blood tests are performed. Which of the following is the most likely set of findings of coagulation screening tests? A. Normal bleeding time, platelet count, and thrombin time; prolonged PT and APTT B. Normal bleeding time, platelet count, thrombin time, and APTT; prolonged PT C. Normal bleeding time, platelet count, thrombin time, and PT; prolonged APTT D. Normal platelet count and thrombin time; prolonged bleeding time, PT, and APTT E. Prolonged bleeding time, PT, APTT, and thrombin time; decreased platelet count

21 C. SIMILAR TO PREVIOUS BOARD EXAM CONCEPT/PRINCIPLE. Classic hemophilia (factor VIII deficiency) is an abnormality of the intrinsic pathway of coagulation proximal to the final common pathway, which begins at factor X â†' Xa activation. This defect leads to a prolonged APTT. The other laboratory tests listed remain normal, because the bleeding time is a measure of platelet plug formation, the PT is a measure of the extrinsic pathway of coagulation, and the thrombin time is an assay of the conversion of fibrinogen to fibrin. The presumptive diagnosis is confirmed by specific factor VIII assay.

Coffin lid appearance is seen in calculi composed of: A. Calcium oxalate B. Cystine C. Magnesium ammonium phosphate D. Uric acid E. Calcium carbonate

210 C. Struvite stones are composed of magnesium ammonium phosphate.

Which are considered vascular phenomena in infective endocarditis? A. Janeway lesions and Osler nodes B. Splinter hemorrhages and Roth spots C. Janeway lesions and Roth spots D. Splinter hemorrhages and Janeway lesions E. Osler nodes and Roth spots

211 D. Osler nodes and Roth spots are immunologic phenomena. Osler nodes are subcutaneous nodules in the pulp of the digits. Roth spots are retinal hemorrhages in the eyes. Splinter hemorrhages are micro-thromoboemboli. Janeway lesions are erythematous or hemorrhagic nontender lesions on the palms or soles.

Which heavy metal plays a role in the treatment of relapsing acute promyelocytic leukemia? A. Arsenic B. Lead C. Copper D. Mercury E. Zinc

212 A. AML with the t(15,17) (promyelocytic leukemia) is treated with pharmacologic doses of ATRA (all-trans retinoic acid) combined with conventional chemotherapy, or more recently, with arsenic salts, which appear to cause PML-RARa to be degraded.

Which characteristic of a tumor will make it less radiosensitive? A. Peripheral location B. Highly oxygenated C. Poorly vascularized D. A and B E. B and C

213 C. Poorly vascularized, poorly oxygenated, and a central location makes a tumor less radiosensitive.

Subacute combined degeneration of the spinal cord is seen in deficiency of which vitamin? A. B1 B. B2 C. B3 D. B5 E. B12

214 E. The combined degenration of both ascending and descending tracts of the spinal cord is characteristic of vitamin B12 deficiency.

A 38 year old male patient presents with a longstanding history of abdominal pain and intermittent diarrhea. Imaging studies showed cobblestone appearance of the colon with ulcers that were sporadically located. Biopsy revealed non-caseating granulomas. What antibodies might this patient potentially have? A. p-ANCA B. c-ANCA C. Anti-Saccharomyces antibodies D. Antibodies to gliadin E. Anti-mitochondrial antibodies

215 C. Anti-Saccharomyces antibodies are present in Crohn's disease.

What fusion gene is carried in the Philadelphia chromosome of CML? A. C-myc and N-myc B. BCR-ABL C. BRCA1 and BRCA2 D. NOD2 E. JAK/STAT

216 B. CML is distinguished from other myeloproliferative disorders by the presence of a chimeric BCR-ABL gene derived from portions of the BCR gene on chromosome 22 and the ABL gene on chromosome 9.

A 42 year old female presents with a 5 year history of palpable breast mass. Biopsy showed signet ring cells arranged in an Indian file pattern. The mass is most likely: A. Invasive ductal carcinoma B. Invasive lobular carcinoma C. Paget's disease of the breast D. Carcinoma in situ E. Medullary carcinoma of the breast

217 B. The histologic hallmark is the presence of dysohesive infiltrating tumor cells, often arranged in a single file pattern or in loose clusters or sheets. Signet ring cellscontaining an intracytoplasmic mucin droplet are common.

The chest x-ray of a cyanotic infant revealed an egg-shaped heart. Which statement is correct? A. The pulmonary veins empty into the right atrium. B. The underlying pathology is obstruction of the right ventricular outflow tract. C. This condition is common among infants of diabetic mothers. D. The infant probably has Down's syndrome. E. The aorta is constricted at a site just distal to the ligamentum arteriosum.

218 C. This is transposition of the great arteries. Choice A is TAPVC, choice B is TOF, choice D is endocardial cushion defect, and choice E is coarctation of the aorta.

A patient with retroperitoneal fibrosis is also at risk of having: A. Left-sided varicocele B. Hashimoto's thyroiditis C. Crohn's disease D. Primary sclerosing cholangitis E. Whipple's disease

219 D. Retroperitoneal fibrosis is associated with Reidel thyroiditis, PSC, and right-sided varicocele.

A 9-year-old girl is diagnosed with acute rheumatic fever. Instead of recovering as expected, her condition worsens, and she dies. Which of the following is the most likely cause of death? A. Central nervous system involvement B. Endocarditis C. Myocarditis D. Pericarditis E. Streptococcal sepsis

22 C. The most common cause of death that occurs during acute rheumatic fever is cardiac failure secondary to myocarditis.

This subtype of Hodgkin's lymphoma is considered non-classical. A. Nodular sclerosis B. Lymphocyte-predominant C. Lymphocyte-rich D. Lymphocyte-depleted E. Mixed cellularity

220 B. In LP, the Reed-Sternberg cells have a distinctive B- cell immunophenotype that differs from that of the classical types.

Which of the following is true regarding dense deposit disease? A. Characterized by immune complex deposits in the glomerular basement membrance and activiation of classical pathway B. Highly responsive to treatment with immunosuppresive drugs such as pulse cyclophosphamide combined with steroids C. Diminished serum levels of factor B and properdin D. Dense materials are primarily deposited on the podocyte-GBM junction E. Less recurrence among allograft recipients compared to Type I MPGN

221 C. dense deposit disease (MPGN type II) - is a primary type MPGN associated with activation of the alternative pathway. This is based on the diminished serum levels of factor B and properdin along with normal C1 an C4 levels. Ultrastructurally, type II MPGN is characterized by deposition of dense material along the GBM proper. Natural history of patients with this disease showed refractory to combined pulse cyclophosphamide and steroids. there is also high incidence of recurrence among transplant patient compared to type I MPGN . Robbins 8th ed pp 928- 929

54 y/o male patient went for an OPD consult secondary to a chief complaint of urinary frequency and nocturia. PMHx: CABG-2 years ago. FM: (+) DM (+) Htn both parents. he is on insulin therapy since he was 45 y/o; other medications include the ff: losartan, metoprolol, ACEI. pertinent physical examination showed the following data: BP 140/80, PR 98 bpm, T 37.1C; BMI: 34 kg/m2, soft nontender abdomen, no suprapubic tenderness, no CVA tenderness, (+) bipedal edema. which of the following is a correct morphologic characteristics behind his most plausible cause of urinary complaint? A. presence of focal thickening of the glomerular capillary basement membrane B. presence of mesangial proliferation secondary to hyperplasia of the mesangial cells C. PAS negative nodular lesions noted on the periphery of the glomerulus. D. presence of hyaline arteriolosclerosis affecting both afferent and efferent arterioles E. All of the above

222 D. A 54 y/o patient complaining of nocturia and frequency along with constellation of clinical findings such as on diabetic and on insulin therapy, hypertensive, (+) Family history, obese, bipedal edema without any overt clinical evidence of infection is most likely suffering from DM nephropathy. renal morphologic changes include the following. Widespread thickening of the capillary basement membrane. diffuse increase of mesangial matrix secondary to GBM thickening with minimal mesangial cell proliferation. Presence of PAS positive nodules ( Kimmelsteil Wilson nodules) along the periphery of the glomerulus. diffuse hyaline arteriolosclerosis affecting both afferent and efferent arterioles. Robbins 8th ed pp 1140-1141

A 45 y/o male patient brought to ER due to massive hematochezia, he is a known case of liver cirrhosis secondary to Chronic Hep B infection. Clinical findings are of the following: lethargic, BP 60 mmHg palpatory, PR 145 bpm, pale palpebral conjunctiva, icteric sclera, cold clammy extremities. he was successfully stabilized after undergoing endoscopic band ligation along with blood transfusion and intravenous hydration. during his hospital stay, there was a noted increase in the serum creatinine from 1.1 -2.1 mg/dl accompanied by oliguria. which of the following is true regarding the cause of the oliguria? A. the most affected nephron segment is proximal segment of the proximal convoluted tubule B. he is at the stage wherein there is increased risk for generalized infection and electrolyte abnormalities like hypokalemia C. cell swelling, vacuolization and tubulorrhexis primarily along the descending limb of henle D. Eosinophilic hyaline cast noted along the ascending limb of henle and Distal tubules E. all of the above

223 D. the rise of the serum creatinine and presence of oliguria after an hypotensive episode is suggestive of ischemic type AKI. Morphologic changes include patchy necrosis primarily along the the straight portion of the PCT and ascending limb of henle, eosinophilic cast composed of tamm horsfall protein are noted along the ascending limb and the distal tubules. In contrast, toxin mediated AKI is manifested by diffuse damage along the whole length of the PCT. based on lab findings and having oliguria, patient is still on maintenance phase. It is the recovery phase that is associated with increase urinary volume, hypokalemia and susceptibility to infection. Robbins 8th ed pp 937-938

A 66 y.o male went for consult secondary to recent onset of weight loss accompanied by anorexia and easy fatigability. PE findings showed normotensive, tachycardic, pale palpebral conjunctiva, enlarged lymph nodes on bilateral cervical, axillary, inguinal regions. Traube space is obliterated and liver edge is 5cm from right subcostal margin. CBC showed normocytic anemia, thrombocytopenia and lymphocytosis. A diagnosis of chronic lymphocytic leukemia is entertain, which of the following is consistent regarding the diagnosis? A. there is diffuse effacement of lymph nodes by predominantly small lymphocytes along with loose aggregrates of larger activated lymphocytes. B. disruption of normal immune function accompanied with hypergammaglobulinemia C. prolymphocytic transformation to diffuse B cell lymphoma is rare D. There is a high incidence of chromosomal translocation E. all of the above.

224 A. CLL is the most common leukemia of adults in the western countries. It is distinguished to SLL only by absolute lymphocytic count of >4000/mm3. Morphologic characteristics include diffuse effacement of the LN along with loose aggregrates of atypical larger lymphocytes which collectively called proliferation centers. unlike most leukemias, it is rare for CLL to undergo chromosomal translocation. furthermore, CLL also has an unknown mechanism that disrupts the normal immune function resulting to a decrease in antibody production. CLL is also prone to undergo Richter transformation wherein there is transformation of the primary cancer to DLBCL.

Which of the following factors is associated with worse prognosis in Acute Lymphoblastic Leukemia? A. Presence of philadelphia chromosome B. Peripheral blood blast count of 80,000 C. Presentation of symptoms at 5 years of age D. presence of chromosomal translocation t (12,21) E. hyperploidy

225 A. ALL has one of the greatest prognosis since it is highly responsive to chemotherapy, however there are some parameters associated with worse prognosis. These are the following: age under 2 at presentation, presentation at adulthood or adolescence, blast count of more than 100,000; presence of phidelphia chromosome. On the otherhand, favorable prognostic factors include the ff: an age 2-10 years old, a low white cell count, hyperploidy, trisomy of chromosome 4,7,and 10, presence of t(12,21) Robbins 8th ed pp 603

A 72 y/o male went for consult secondary to 3 days of fever accompanied by productive cough. History revealed that he had multiple episodes of pneumonia within the last 3 years .aside from the primary complaint, he also had polyuria, continuous low back pain even at rest and constipation. PE are normal except for the ff: pale conjunctiva, bibasilar crackles, hypotonic bowel sounds, paravertebral tenderness, smooth, non nodular slightly enlarged prostate, weak bilateral LE strength 2/5. Radiographic examination showed pneumonia of bilateral lower lobes, with incidental finding of thoracic compression fractures of t5-t6 t8-t9 level and multiple lucencies along the thoracic vertebral bodies. CBC showed normocytic normochromic anemia, leukopenia, thrombocytopenia. UA revealed massive protenuria. which of the following is consistent with the most possible primary diagnosis A. patient is suffering from hypocalcemia and hypoglobulinemia B. it is the most common plasma cell dyscrasia C. definitive diagnosis can be established only through clinical and radiographic findings D. Infection is the most common cause of death E. All of the above

226 D. expect clinical scenarios in your board exam. An elderly patient presenting with recurrent pneumonia, low back pain and polyuria supported with lab evidence of pancytopenia and radiographic finding of compression fractures of lesions on vertebral column is consistent with multiple myeloma. features of MM stems from the effect of plasmacytic growth on axial skeleton, production of excessive immunoglobulins and alteration of humoral immunity. Because of the factors produced by plasma cells, it causes the activation of osteoclast leading to bone resorption and attendant hypercalcemia. Definitive diagnosis is made only by doing bone marrow examination. Patient usually suffers renal insufficiency secondary to bence jones proteinuria. Infection is the most common cause of death among this patient secondary to abnormal immunoglobulin production and decreased number of white blood cells. MGUS and not MM is the most common plasma cell dyscrasia

Which of the following is not an immune complex mediated disease? A. Systemic Lupus Erythematosus B. Multiple sclerosis C. Polyarteritis nodosa D. Poststreptococcal glomerulonephritis E. None of the above

227 B. Multiple sclerosis is an example of type IV hypersensitivity reaction. Other type III or immune complex mediated reaction aside from the choices are serum sickness, arthus reaction and reactive bacterial arthritis.

What is the most serious complication of Tuberculous meningitis? A. Tuberculoma formation B. Fibrinous basal exudates leading to cranial nerve palsies C. Choroid plexus involvement leading to diffuse meningoencephalitis D. Obliterative endarteritis E. none of the above

228 D. the most serious complication of chronic tuberculous meningitis is arachnoid fibrosis leading to hydrocephalus and obliterative endarteritis leading to brain infarction. This is SIMILAR TO PREVIOUS BOARD EXAM CONCEPT/PRINCIPLE.

A 42 y/o male presents with right abdominal mass associated with gross hematuria. He also had episodes of diaphoresis and refractory hypertension. Imaging showed renal mass and adrenal medullary tumor. What is your primary diagnosis? A. Sipple syndrome B. Tuberous sclerosis C. Von hippel lindau syndrome D. Li Fraumeni syndrome E. None of the above

229 C. Von hippel lindau disease is a type of familial tumor syndrome characterized by renal cell carcinoma, pheochromocytoma and cerebellar hemangioblastoma. Sipple syndrome aka MEN type IIA is a triad of parathyroid hyperplasia, medullary carcinoma, pheochromocytoma. Tuberous sclerosis is associated with renal angiomyolipoma, retinal hamartoma,cortical tubers, cardiac rhabdomyoma, shagreen patches and ash leaf patch. Gorlin syndrome.Li Fraumeni syndrome is caused by p53 mutation and associated with sarcomas, breast cancer, adrenal cortical tumors, leukemia and gliomas.

A 25-year-old man presents with hematuria, periorbital edema, hypertension, and hemoptysis. He has also experienced nausea, vomiting, fever, and chills. Serologic testing is positive for antiglomerular basement membrane antibodies. Which of the following is the classic histologic finding in this renal disease? A. Linear immunofluorescence B. “Lumpy-bumpy†immunofluorescence C. “Spike and dome†appearance of the glomerular basement membrane D. Subendothelial immune complex deposition E. Tram-track appearance of the glomerular basement membrane on electron microscopy

23 A. SIMILAR TO PREVIOUS BOARD EXAM CONCEPT/PRINCIPLE. The clinical description is that of Goodpasture syndrome (antiglomerular basement membrane disease), caused by antibodies directed against antigens in the glomerular and pulmonary alveolar basement membranes. Because antigens are an intrinsic component of the basement membrane, labeled antibodies “paint†the surface of the basement membrane, resulting in the characteristic linear immunofluorescent pattern characteristic of this disorder. “Lumpy-bumpy†immunofluorescence is very coarse, granular immunofluorescence found in poststreptococcal immune complex deposit disease. The tram-track appearance is seen in membranoproliferative glomerulonephritis. The “spike and dome†appearance is seen in membranous glomerulonephritis. Subendothelial immune complex deposition is seen in lupus nephropathy.

What is the most striking histologic finding in Desquamative Interstitial Pneumonia secondary to smoking? A. Thickened alveolar septa due to sparse inflammatory lymphocytic infiltrate B. Mild interstitial fibrosis C. Necrotic type II pneumocytes D. accumulation of a large number of macrophages with brown pigment cytoplasmic bodies. E. none of the above

230 D. SIMILAR TO PREVIOUS BOARD EXAM CONCEPT/PRINCIPLE last aug 2014. taken verbatim Robbins 8th ed pp 704

An 18 yo female patient presents with chronic epigastric pain associated with weight loss, diarrhea, and peripheral edema. Upon endoscopy, menetrier disease was given as a diagnosis. Which of the following is true regarding the diagnosis? A. there is an irregular enlargement of the gastric rugal folds secondary to hyperplasia of gastric connective tissue B. associated with excessive secretion of platelet derived growth factor C. it is a progressive unremmittng condition however no risk for gastric adenocarcinoma D. characterized by hyperplasia of foveolar mucous neck cells E. all of the above

231 D. Menetrier disease is characterized by diffuse hyperplasia of the foveolar epithelium of the body and fundus of the stomach. It is secondary to proliferation of mucous neck cells and not the gastric connective tissue. The hyperplasia is secondary to the excessive secretion of TGF alpha. although it is a self limiting condition, it has an increased risk for the development of gastric adenocarcinoma.

Which of the following pathogenic organism can cause diarrhea by selectively destroying apical mature enterocytes of the small intestine? A. Norovirus B. adenovirus C. Rotavirus D. Norwalk virus E. All of the above

232 C. Rotavirus is the most common cause of severe childhood diarrhea. It affects only the apical and mid villous enterocytes leading to loss of absorptive capacity of the small intestine. All the other virus shows nonspecific affectation of different population of enterocytes. Robbins 8th ed pp 804-805.

A 61 y/o male presents with hesitancy, dysuria, and nocturia. BPH is entertained. Which of the following is true regarding the diagnosis? A. This is secondary to hypertrophy of the stromal and epithelial cells B. the microscopic hallmark is nodularity C. presence of true capsule creates a plane between the normal tissue and prostatic nodules D. the cause of the nodularity is secondary to increase mitosis of the epithelial cells E. all of the above

233 B. Benign Prostatic hyperplasia is characterized by hyperplasia of the stromal and epithelial cells along ther prostatic periurethral zone. The hallmark is the presence of nodularity. The compression of the adjacent normal prostate by the nodules creates a pseudocapsular plane. the pathogenesis behind the hyperplasia is not secondary to increase mitosis but rather impaired cell death.

what is the most serious consequence of alkaptonuria? A. Renal failure B. Hypertrophic cardiomyopathy C. Arthropathy D. Liver failure E. None of the above

234 C. Alkaptonuria is an autosomal recessive disorder secondary to lack of homogentisic oxidase which converts homogentisic acid to methylacetoacetic acid. It causes black discoloration of the urine if it allows to stand on ambient air. It accumulates throughout the body causing blue to black pigmentation of soft tissue such as ears, nose and face. however, it also causes pigmentation of the articular cartilages causing its loss of resiliency and fibrillation. there is no association b/w alkaptonuria and cardiomyopathy,liver failure and renal failure.

Which of the following describes the resolution stage of lobar pneumonia A. Progressive enzymatic digestion of alveolar exudates producing granular semifluid debris ingested by macrophage B. Vascular dilatation with intraalveolar fluid and presence of numerous bacteria C. massive confluent exudation with neutrophils, rbc, and alveolar fibrin D. progressive disintegration of RBC and presence of fibrinosuppurative exudates E. none of the above

235 A. B- congestion, C- red hepatization, D- gray hepatization. Robbins 8th ed pp 712-713

A 28 y/o female veterinarian presents with 3 week history dry cough accompanied by low grade fever and night sweats. CXR revealed cavitary lesion noted on bilateral lung apices. AFB showed negative for 3 specimen. PPD is also negative. Which of the following is true regarding the most plausible diagnosis in this case? A. the organism is internalized primarily by NK cells and atypical lymphocytes after opsonization with antibody B. there is absence of caseation necrosis C. produces concentric calcification of the lesion ( tree bark appearance) when drug control is achieved D. rarely become disseminated even in immunocompromised state. E. all are correct.

236 C. the most likely impression regarding this case is histoplasmosis. TB is set aside because of the absence of positive sputum smear along with negative PPD. Histoplasmosis clinical and morphological presentation greatly mimicks that of TB. H. capsulatum is internalized by macrophage after opsonization. there is also presence of caseation necrosis. fulminant disemminated histoplasmosis can occur in immunocompromised state. This patient is also veterinarian which most likely expose to bird droppings.

A 59 y/o female presents with rapidly enlarging anterior neck mass accompanied with proggressive of dyspnea, dysphagia and hoarseness of 1 month duration. What would be the most consistent pathologic finding if biopsy is done to the mass? A. large pleomorphic multinucleated giant cells with fusiform cells B. spindle cells with amyloid deposits on adjacent stroma C. finely dispersed chromatin giving a optically clear or empty appearance of the nucleus D. uniform cells forming small follicles containing colloid substance E. None of the above.

237 A. based on the rapidity of symptoms, the most likely diagnosis is anaplastic carcinoma. B- Medullary carcinoma. C- papillary carcinoma. D- follicular carcinoma

A 45 y/o male presents with a rapidly growing nodular skin lesion on the forehead with noted telangiectasia on top of the lesion. If biopsy was done, which of the following will be consistent with the most plausible diagnosis? A. Highly anaplastic with necrosis and presence of polygonal cells with numerous areas of keratinization B. basaloid cells with hyperchromatic nuclei with palisading alignment in the periphery of the tumor C. hyperkeratosis with horn cyst D. basaloid cell with with hair like differentiation E. presence of civatte bodies

238 B. based on the characteristics, this is most likely a case of basal cell carcinoma. A- squamous cell carcinoma, c- seborrheic keratoses, d- trichoepithelioma, e- lichen planus

A 49 y/o female presented to the ER secondary to right tibial fracture after a fall from a standing height, Xray revealed middle transverse fracture of the right tibia with noyed enlarged, sclerotic irregular thickening of both cortical and cancellous bone. further physical examination noted weakness of bilateral hip flexors and knee flexors, sensory deficit on L3- S1 dermatome bilateral. patient presents with prominent zygoma and supraorbital ridge with frontal bossing. she also has lateral rectus palsy on (R), (L) peripheral facial palsy. which of the following is the most likely diagnosis of the patient? A. Pagets disease B. early onset idiopathic osteoporosis C. osteomalacia D. multiple enchondromatosis E. Osteopetrosis

239 A. Pagets disease or Osteitis deformans is a rare skeletal disease characterized by haphazard mosiaic pattern of lamellar bone formation. Clinically it affects the axial skeleton leading to compression of multiple spinal and cranial nerve exits. Bony overgrowth of skull base also causes different cranial nerve palsies. they also prominence of facial bones presenting as leontiasis ossea.

A 19-year-old young woman who emigrated from Taiwan 8 years ago presents with fever, malaise, myalgias, and arthritis and “coldness†in her upper extremities. She has a weak radial pulse bilaterally, and a magnetic resonance angiogram demonstrates nearly 75% stenosis of the main arteries originating from the aorta. She likely has which of the following rheumatologic conditions? A. Buerger disease B. Kawasaki disease C. Raynaud disease D. Takayasu arteritis E. Temporal arteritis

24 D. Inflammation and stenosis of branches of the aortic arch is known as Takayasu arteritis, or “pulseless disease.†It most commonly occurs in young Asian females. Buerger disease usually affects young Jewish males and involves the arteries of the extremities. The disease is exacerbated by smoking and can lead to gangrene of the extremities. Kawasaki disease affects the branches of the coronary arteries. Raynaud disease is due to vasospasm of small vessels of the fingers and toes, leading to cyanosis and pallor of the affected tissues. Temporal arteritis is usually encountered in older patients and affects the branches of the carotid artery, most commonly the temporal artery.

What is the most common malignancy of the eyelid? A. Squamous cell carcinoma B. Basal cell carcinoma C. Sebaceous carcinoma D. lymphangioma E. Hemangioma

240 B. basal cell carcinoma is the most common malignancy of the eyelid with predilection on the lower lid and medial canthus. This is followed by sebaceous carcinoma and squamous cell carcinoma. Robbins 8th ed p 1348

In pathologic examination of a brain of a person who died from rabies which of the following areas of the brain where the pathognomonic negri bodies will be found? A. cerebrum B. cerebellum C. hippocampus D. thalamus E. B and C

241 E. SIMILAR TO PREVIOUS BOARD EXAM CONCEPT/PRINCIPLE, in our exam we had to choose between cerebellum or hippocampus, both answers are correct and should be either cerebellum or hippocampus

a 65 year old male patient presents with a chronic history of decreasing urinary stream, dribbling of urine and difficulty in voding, What is the pathophysiologic process involved in a person diagnosed to have BPH? A. hypertrophy B. hyperplasia C. neoplasia D. metaplasia E. none of the above

242 B. SIMILAR TO PREVIOUS BOARD EXAM CONCEPT/PRINCIPLE

What percentage of patients with hepatitis B eventually proceed to have hepatocellular carcinoma? A. 6-15% B. 12-20% C. >30% D. <5% E. 2-8%

243 A. SIMILAR TO PREVIOUS BOARD EXAM CONCEPT/PRINCIPLE, almost exact choices, page 96 of topnotch handout in the diagram, I did not get this as well during our exam :)

A 60 year old patient with longstanding type 2 DM recently underwent renal transplantation of 1 kidney for end stage renal disease secondary to diabetic nephropathy, after a period of 4 weeks after transplantation, he developed sudden weight gain, bilateral pedal edema, pain or tenderness near the right flank area and fever, histologic examination would most likely reveal: A. thrombotic occlusion of capillaries and fibrinoid necrosis B. extensive interstitial mononuclear cell infiltration and edema C. vascular changes, interstitial fibrosis and tubular atrophy with loss of renal parenchyma D. A and B E. all of the above

244 B. SIMILAR TO PREVIOUS BOARD EXAM CONCEPT/PRINCIPLE, A is hyperacute, B is acute, C is chronic

A 56 year old male presenting with recent onset of bipedal edema, BP was 120/80 Heart rate of 90. Does not recall any history of recent infections but has been said to have multiple sexual partners as well. Over the past few weeks, he noticed his urine to be foamy and bubbly, and would have puffy eyelids especially on waking in the morning. Further examination revealed urine protein of +3, RBCs 0-2 WBC 0- 1, pus cells (+1), no casts and crystals, urine specific gravity of 1.015, urine pH of 5.5. A renal biopsy was eventually done revealing large hypercellular glomeruli, thickened glomerular basement membrane and increased mesangial matrix. Immunofluorescence revealed granular pattern of C3, IgG and C1q, electron microscopy revealed subendothelial deposits, which of the following is the most likely finding? A. focal segmental glomerulosclerosis B. membranous glomerulonephritis C. rapidly progressive glomerulonephritis D. membranoproliferative glomerulonephritis E. minimal change disease

247 D. SIMILAR TO PREVIOUS BOARD EXAM CONCEPT/PRINCIPLE, most questions were cases, wordy, 2 step cases. A lot of renal pathology. Subendothelial, granular pattern nephrotic syndrome is membranoproliferative type

What is the most common etiologic factor in acute myeloid leukemia? A. radiation exposure B. benzene C. genetic mutation D. carcinogen exposure E. previous treatment for leukemia

248 C. SIMILAR TO PREVIOUS BOARD EXAM CONCEPT/PRINCIPLE, vague question, a lot of genetic mutations like down syndrome are commonly associated with acute leukemias, all the others are also common causes of leukemias but less common.

What is the pathophysiologic mechanism in which patients with protein energy malnutrition sometimes have fatty liver especially with protein malnutrition? A. increased mobilization of fat stores from adipose tissue B. increased glucose stored as fat C. increased endogenous synthesis of triglycerides D. impaired transport of fats E. all of the above

249 D. SIMILAR TO PREVIOUS BOARD EXAM CONCEPT/PRINCIPLE, malnutrition with low protein levels leads to low synthesis of necessary proteins like globulins and lipoprotein transporters like LDL HDL VLDL that transport cholesterol and trigylcerides thus fats are continuously stored in the liver and not distributed properly to the tissues

A 45-year-old man presents with involuntary facial grimaces and movements of the fingers. His mother had had similar symptoms beginning at about the same age. Her disorder had progressed to dancing movements, writhing of the arms and legs, and eventually coma and death. His maternal grandfather had had a similar disorder but at an age older than the mother. Which of the following is most characteristic of this disease? A. Degeneration of upper and lower motor neurons B. Dopamine depletion and depigmentation of the substantia nigra C. Increased number of trinucleotide repeats in a gene on chromosome 4 D. Neurofibrillary tangles and amyloid plaques in the cerebral cortex E. Pick bodies, characterized by round intracytoplasmic inclusions consisting of neurofilaments

25 C. This is a case of Huntington disease, which is an autosomal dominant, fatal, progressive degeneration and atrophy of the striatum (caudate nucleus and putamen). The disorder is characterized by an increased number of trinucleotide (CAG) repeats in the HD (huntingtin) gene on the short arm of chromosome 4. Degeneration of the upper and lower motor neurons is characteristic of ALS. Dopamine depletion and depigmentation of the substantia nigra is characteristic of Parkinson disease. Neurofibrillary tangles and amyloid plaques are found in Alzheimer disease. Pick bodies can be found in Pick disease, which clinically resembles Alzheimer disease.

A 50 year old male with long standing history of chronic alcoholism presents with jaundice, palmar erythema, distended abdomen, spider nevi near in the upper anterior chest wall. What is the most likely histologic finding present in the liver? A. councilman bodies B. fine foamy appearance with feathery degeneration and accumulation of bile pigment plugs in dilated canaliculi C. apoptosis bridging necrosis, ballooning degeneration, interface hepatitis, ground glass hepatocyes D. hepatocyte apoptosis bridging necrosis and fibrosis, deposition of fibrous tissue, E. hepatocyte swelling necrosis, eosinophilic clumps in hepatocytes, both micro and macronodular cirrhosis

250 E. SIMILAR TO PREVIOUS BOARD EXAM CONCEPT/PRINCIPLE, eosinophilic clumps are mallory bodies with both macro micronodular cirrhosis in the setting of long standing alcoholism and hallmarks of cirrhosis, D is chronic hepatitis,C is acute hepatitis B is cholestatic change

What pathological entity is characterized by widespread small vessel vasculitis, interstitial and perivascular fibrosis of the skin and multiple organs, progressive atrophy and collagenous replacement of alimentary canal especially the esophagus, clawlike deformities and limited range of motion of joints and fingers, raynauds phenomenon, fibrinoid necrosis of renal arterioles A. wegeners granulomatosis B. systemic lupus erythematosus C. systemic sclerosis D. churg strauss E. polyarteritis nodosa

251 C. SIMILAR TO PREVIOUS BOARD EXAM CONCEPT/PRINCIPLE, diffuse skin fibrosis and organ changes including esophageal dysmotility are due to small vessel vasculitis and fibrosis

A 30 year old male patient with 9 months history of chronic cough with sputum production accompanied by low grade afternoon fevers, night sweats, weight loss suddenly presents with fever, altered sensorium and nuchal rigidity. Cranial CT scan shows basal enhancement and multiple infarcts, what is the pathophysiologic mechanism of these infarcts? A. thrombotic occlusion of the vessels causes multiple infarcts B. embolic seeding from extrapulmonary tuberculotic sites C. vasospasm induced by hemorrhage in the CNS D. obliterative endarteritis produced by CNS infection E. accelerated atherosclerotic changes

252 D. SIMILAR TO PREVIOUS BOARD EXAM CONCEPT/PRINCIPLE - obliterative endarteritis cause the multiple microinfarcts from CNS TB

A 16 year old female patient presents with 2 months history of easy fatigability, body malaise, afternoon fevers, night sweats and weight loss. There was noted occasional cough with multiple palpable cervical lymph nodes noted which prompted an initial assessment of pulmonary tuberculosis, however, a chest xray revealed an incidental mediastinal mass with no apical lesions noted. What is the most likely consideration? A. primary bronchogenic carcinoma B. hodgkin's lymphoma C. non hodgkin's lymphoma D. pulmonary tuberculoma E. metastatic pulmonary carcinoma

253 B. B symptoms, mediastinal mass, contiguous growth most likely hodgkin's lymphoma

50 year old postmenopausal female patient presenting with a history of vertebral compression fractures and hip fracture was worked up due to repeated complains of leg and extremity pains. She was noted to have 4 episodes of flu over the past 2 months. Work ups revealed mild anemia, slightly elevated serum calcium and multiple lytic bone lesions were noted over the ribs and some punched out defects were seen in the skull xrays, which of the following conditions does this patient most likely have A. osteoporosis B. multiple myeloma C. waldenstrom's macroglobulinemia D. myelodysplastic syndrome E. osteomalacia

254 B. SIMILAR TO PREVIOUS BOARD EXAM CONCEPT/PRINCIPLE, punched out lesions and multiple pathological fractures

35 year old female presents with an erythematous rash on the cheeks aggravated by exposure to sunlight, she has been having a history of repeated recurrent joint pains and swelling over the ankles and wrists, she has some oral apthous ulcers accompanied by headaches and easy fatigability. She came in for consult due to a recent onset of bipedal edema accompanied by tea colored urine. Initial work ups were requested which revealed urine protein of +2, urine RBC of 5-10, urine WBC of 0-1, pus cells of +1, rbc casts +1, no crystals. ANA titers were also elevated. What is the autoantibody specific to this disease which also associated with renal pathological involvement A. anti-smith B. ANA C. anti-dsDNA D. anti-histone E. Anti-Ro

255 C. SIMILAR TO PREVIOUS BOARD EXAM CONCEPT/PRINCIPLE, most specific and indicates a probable renal involvement of disease

In relation to the question above, if a kidney biopsy is done, what is the most pathognomonic expected finding? A. nonspecific mesangial proliferation B. tram track appearance of the basement membrane C. spike and dome appearance D. chicken wire loop appearance E. focal segmental glomerulosclerosis

256 D. SIMILAR TO PREVIOUS BOARD EXAM CONCEPT/PRINCIPLE, a lot of multiple stem questions

In relation to question above, this disease manifestation belongs to what type of immune hypersensitivity? A. type 1 B. type 2 C. type 3 D. type 4 E. non of the above

257 C. SIMILAR TO PREVIOUS BOARD EXAM CONCEPT/PRINCIPLE, immune complex type 3, ABO incompatibility also came out as type 2 hypersensitivity

Which of the following cytokines secreted by T helper lymphocytes leads to direct strong activation of macrophages? A. IL1 B. TNF alpha C. TNF beta D. IFN gamma E. IL-6

258 D. SIMILAR TO PREVIOUS BOARD EXAM CONCEPT/PRINCIPLE from boards, IL1 and IL6 are pyrogens and initiate inflammation and fever, TNF alpha also promotes inflammation and cachexia, TGF beta inhibits inflammation and promotes cell repair

20 year old patient developed paraparesis. 2 weeks after developing a diarrheal episode, patient started developing gradual ascending paralysis. No other sensory and CN deficits were seen, lumbar puncture was done, which of the following is an expected finding? A. oligoclonal bands IgG, CSF pleocytosis B. increased CSF protein, WBCs 0-1 C. increased CSF protein, neutrophils 5000 D. increased CSF protein, lymphocytes 500 E. normal CSF protein, normal glucose WBCs 0- 1

259 B. SIMILAR TO PREVIOUS BOARD EXAM CONCEPT/PRINCIPLE, GBS albuminocytologic dissociation

A 14-year-old girl presents with prolonged bleeding from wounds and minor trauma and severe menorrhagia. Family history reveals that her father also has prolonged bleeding from wounds and minor trauma, as does her brother. Which of the following is the most likely mechanism of this patient’s disorder? A. Absence of platelet glycoprotein IIb-IIIa B. Antiplatelet antibodies reacting with platelet surface glycoproteins C. Deficiency of factor VIII D. Deficiency of factor IX E. Deficiency of vWF

26 E. von Willebrand disease, a disorder transmitted by autosomal modes of inheritance (both dominant and recessive) is the most common hereditary bleeding disorder. There are many variants, all marked by either qualitative or quantitative deficiencies of vWF.

30 year old female patient presenting with nonspecific pneumonitis, cough, intermittent bouts of bronchial asthma with long standing history of allergic rhinitis. Presents with nodular skin lesions, biopsy of skin lesions reveal capillary vasculitis with granulomas and eosinophilic necrosis. There was no noted renal involvement, the most likely pathological condition in this case is? A. polyarteritis nodosa B. churg strauss C. wegeners granulomatosis D. microscopic polyangitis E. giant cell arteritis

260 B. SIMILAR TO PREVIOUS BOARD EXAM CONCEPT/PRINCIPLE, asthmatic manifestations, pulmonary, skin, other involved organs but spares the renal vessels

All of the following are ultrastructural changes of reversible cell injury except: A. Dilation of the endoplasmic reticulum B. Cell membrane blebbing C. Karyorrhexis D. Detachment of polysomes E. None of the above

261 C. Karyorrhexis is nuclear fragmentation which is irreversible

A patient was diagnosed with Paget's disease and has been treated with bisphophonates. Currently, he presents at your clinic with left hip pain that is not relieved by over the counter pain relievers? He has history of taking low dose steroids for his asthma. Imaging reveals lytic and sclerotic changes with periosteal elevation and reactive new bone formation. What is the most likely diagnosis? A. Osteopetrosis B. Glucocorticoid-induced osteoporosis C. Osteoarthritis D. Osteosarcoma E. Rheumatoid arthritis

262 D. Although the use of glucocorticoids may predispose to osteoporotic fracture. The description indicates Codman's triangle and is indicative of osteosarcoma. Paget's disease is a predisposing factor to osteosarcoma

Which type of small vessel vasculitis is characterized by eosinophil-rich granulomatous inflammation involving the respiratory tract and is associated with asthma and blood eosinophilia with sparing of the kidneys? A. Polyarteritis nodosa B. Wegener granulomatosis C. Takayasu arteritis D. Leukocytoclastic vasculitis E. Churg-Strauss syndrome

263E.Churg-Strauss spares the kidney and is characterized by association with asthma and eosinophilia

Which if the following cytokines has an important role in IFN-γ production in Th1- mediated chronic inflammation? A. IL-12 B. IL-10 C. IL-4 D. IL-2 E. TNF-α

264 A. IL-12 - Th1; IL-4 - Th2; IL10 -suppression of inflammation

A patient with Duchenne muscular dystrophy is a muscular disorder involving the dystrophin gene that usually leads to a male patient being wheelchair-bound during his teenage years. In contrast, Becker muscular dystrophy is a milder but similar form of disease wherein afflicted patients can usually still walk at the same age period. This is an example of: A. Pleiotropy B. Variable expressivity C. Anticipation D. Incomplete penetrance E. Allelic heterogeneity

265 E. Allelic heterogeneity refers to different mutations at the same locus. Variable expressivity is the same mutation but with different phenotypic characteristics. Anticipation occurs in trinucleotide repeats and is characterized by an ealier and more severe onset of disease in suceeding generations. Incomplete penetrance means some patients with the disease causing mutation may not present signs or symptoms; usually in autosominal dominant. Pleiotropy occurs in mitochondrial disorders.

A 34 year old male smoker presents to your clinic with leg pain. History is unremarkable except for a 15 year history of smoking consuming 1 pack per day. Pain worsens with exercise and is also not totally relieved by rest. Biopsy of the affected leg reveals segmental vasculitis of small to medium vessels accompanied with luminal thrombosis and microabscesses extending into contiguous veins and nerve. What is the most likely diagnosis? A. Thromboangiitis obliterans B. Polyarteritis nodosa C. Peripheral arterial atherosclerosis D. Microscopic polyangiitis E. Infectious vasculitis

266 A. Thromboangiitis obliterans or Buerger disease usually in male smokers less than 35 years old.

A 32 year old male patient claims that he has difficulty with prolonged chewing of food and that he experiences diplopia when watching TV for long periods of time. What is neoplasm most associated with this condition? A. Small cell lung Ca B. Thymoma C. Hodgkin's lymphoma D. Renal Cell Ca E. Breast Ca

267 B. Thymoma - associated with myasthenia gravis; Small cell Ca - associated with Lambert-Eaton (improves with prolonged stimulation)

A 10 year old child presents to your clinic with complaints of dark urine, periorbital edema, malaise, and body weakness. You elicit a history of throat infection 3 weeks prior. You expect which of the following pathological lesion for this type of hypersensitivity reaction: A. Vascular dilation edema, smooth muscle contraction B. Phagocytosis and lysis of cells C. Perivascular cellular infiltrates, edema, and granuloma formation D. Inflammation with associated necrotizing vasculitis E. None of these

268 D. A is type 1 hypersensitivity; B is type II, C is type IV, D is type III

An young athlete dies suddenly and unexpectedly. Autopsy reveals massive myocardial hypertrophy with disproportionate thickening of the ventricular septum. What is the most likely pathologic etiology of this condition? A. Previous viral myocardial infection B. Atherosclerotic blockage of coronary arteries C. Radiation-induced myocardial fibrosis D. Mutation of gene encoding beta-myosin heavy chain E. None of the above

269 D. Hypertrophic cardiomyopathy is most commonly caused by a mutation in myosin heavy chain

A 42-year-old man is seen because of a long history of slowly developing congestive heart failure. His blood pressure is normal. Coronary artery angiography reveals no vascular disease. No heart murmurs are heard. The white blood cell count, differential, and erythrocyte sedimentation rate are normal. The most likely diagnosis is A. carcinoid heart disease B. cardiomyopathy C. coarctation of the aorta D. constrictive pericarditis E. myocardial infarction

27 B. Cardiomyopathies are noninflammatory myocardial disorders that are not associated with coronary artery obstruction, hypertension, valvular disease, congenital heart disease, or infectious disease. They are most often characterized by otherwise unexplained ventricular dysfunction, such as cardiac failure, ventricular enlargement, or ventricular arrhythmias.

You are assigned to assist a transplant surgeon who will be performing a kidney transplant. In order to prepare yourself for possible questions that you may be asked during the surgery, you recall your pathology knowledge.You know that chronic rejection will be characterized by: A. Extensive interstitial mononuclear cell infiltration and edema B. Dominated by vascular changes, interstitial fibrosis C. Occurs due to preformed antibodies is the circulation D. Takes the form of necrotizing vasculitis with endothelial cell necrosis E. A and B

270 B. A and D is acute rejection; C is hyperacute rejection

Which of following characteristics indicate neoplastic/malignant lymph node proliferation? A. Marked variation in the shape and size of the follicles B. Presence of phagocytic macrophages and recognizable light and dark zones C. Preservation of the lymph node architecture D. Marked by monoclonal proliferation E. All of these

271 D. A, B, and C is indicative of non-neoplastic proliferation

A patient presents to your clinic with chronic diarrhea and perianal abscess occasionally associated with low-grade fever. She had a prior history of intestinal fistula formation which was managed conservatively. Physical examination reveals mouth ulcers with no other significant findings. What is the most commonly affected site in patients with this condition? A. Ileum B. Sigmoid colon C. Jejunum D. Stomach E. Duodenum

272 A. Crohn's disease is most common in the ileum though it may occur anywhere in the GI tract

A mother brings to you her 1 year old child due to recurrent respiratory bacterial infections and otitis media. You suspect an immunodeficiency syndrome due to failure of preB cells to undergo Ig gene rearrangement and to develop into mature B cells. The mutation responsible is encoded in which chromosome? A. Chromosome 17 B. Chromosome 7 C. Chromosome 16 D. Chromosome 22 E. Chromosome X

273 E. This refers to Bruton's which is an X-linked immunodeficiency syndrome

A female patient presenting with right breast mass but with no other associated symptoms undergoes biopsy which revealed presence of dyscohesive infiltrating tumor cells, often arranged in single file or in loose clusters or sheets with associated signet-ring cells containing an intracytoplasmic mucin droplet. What is the diagnosis? A. Invasive ductal carcinoma B. Ductal carcinoma in situ C. Paget's disease D. Invasive lobular carcinoma E. Medullary carcinoma

274 D. Charateristic description of invasive lobular Ca

A patient complains of polydipsia and polyuria. Laboratory tests reveal a dilute urine and hypernatremia. If this condition is a nephogenic type of diabetes insipidus, what will be expected during water deprivation test? A. Decreased ADH levels B. Elevated ADH levels C. Elevated urine osmolality D. Correction of hypernatremia E. Low serum osmolality

275 B. ADH levels are adequate but kidney does not respond in nephogenic type

An infant born term to G1P1 mother dies. Prior to death, the infant was noted to have intrauterine growth retardation, jaundice, hepatosplenomegaly, anemia, and bleeding due to thrombocytopenia. Periventricular calcification is observed on imaging and histology reveals prominent intranuclear basophilic inclusions surrounded by a clear halo. In utero infection is suspected. Which is the most likely etiologic agent? A. Toxoplasma B. Herpes virus C. Cytomegalovirus D. Rubella E. Treponema pallidum

276 C. Characteristic description of cytomegalovirus in utero infection.

A patient presents with chronic cough associated with night sweats, weight loss, and body malaise. Sputum microscopy reveals acid fast bacilli and X-ray reveals right upper lobe density? Granulomatous reaction which is typical of this condition is brought about by which of the following: A. CD8 lymphocytes, Humoral immune system B. CD4 lymphocytes, IFNγ C. CD4 lymphocytes, IL-4 D. CD8 lymophocytes, NK cells E. B cells and macrophages

277 B. CD4 lymphocytes and IFN- gamma are responsible for granulomatous reaction (type IV)

A 16 year old African American man presents to the emergency room severe chest pain, fever, tachypnea, and cough. Imaging reveals lung infiltrates and ABG shows hypoxemia. Further probing reveals that patient had previous episodes of painful crises and anemia. Peripheral smear reveals target cells and characteristically shaped erythrocytes. An Hb electrophoresis is ordered but patient deteriorates and dies. Autopsy examination of the spleen will most likely demonstrate: A. Red pulp expansion B. Extramedullary hematopoiesis C. Small fibrotic atrophy D. Congestive changes and increase in volume E. A and B

278 C. Autosplenectomy occurs in early childhood in sickle cell anemia

Baretts esophagus is what type of abnormal cell/tissue change? A. Dysplasia B. Metaplasia C. Anaplasia D. Neoplastic change E. Necrosis

279 B. Intestinal metaplasia - replacement with simple columnar with goblet cells that predispose to adenocarcinoma

A 28-year-old woman complains of fatigue, dyspnea, and malaise. She also notes that her urine has been reddishbrown, particularly with the first void of the morning. Subsequent studies confirm that she has paroxysmal nocturnal hemoglobinuria. Which of the following best describes the defect leading to this condition? A. Anti-intrinsic factor antibodies B. Deficiency of the intracellular structural protein spectrin C. Impaired synthesis of the cell-surface GPI anchor D. Ineffective erythropoiesis E. Substitution of a valine for a glutamate residue in the β-globin gene

28 C. SIMILAR TO PREVIOUS BOARD EXAM CONCEPT/PRINCIPLE. Paroxysmal nocturnal hemoglobinuria results in an acquired intracorpuscular defect in the ability to synthesize GPI anchors required for appropriate placement of complement regulatory proteins on the surface of red blood cells. Functional deficiency of such proteins as CD55 and CD59 renders the cells sensitive to complement-mediated lysis. Anti-intrinsic factor antibodies are seen in pernicious anemia. Spectrin is deficient in hereditary spherocytosis. Ineffective erythropoiesis is seen in megaloblastic anemia due to folate or vitamin B12 deficiencies. Substitution of valine for glutamic acid in the β-globin gene underlies the defect in sickle cell anemia.

A 30 year old woman presents to your clinic complaining of a butterfly rash over the face, fever, joint pains, and photosensitivity. Prelimionary laboratory tests reveal positive ANA and thrombocytopenia. The pathologic picture of the most common form of renal disease associated with this condition is: A. >50% of glomeruli exhibiting crescent formation, fibrinoid necrosis, proliferation of endothelial and mesangial cells B. <50% of glomeruli exhibiting crescent formation, fibrinoid necrosis, proliferation of endothelial and mesangial cells C. characterized by diffuse thickening of the capillary walls D. slight mesangial cell proliferation and immune complex deposition E. moderate mesangial cell proliferation and immune complex deposition

280 A. Most common form of kidney disease in SLE is diffuse proliferative (type IV). B is focal proliferative (III). C is membranous (V). D and E are mesangial (I and II)

A 65 year old woman with a heavy smoking history presents with chronic productive cough that has been present for 3 consecutive months over the past 2 years. On PE, she is noted to be overweight with a bluish tinge to her skin. Auscultation revealed rhonchi on both lung fields. Which of the following is the most likely histologic finding in this patient's airways? A. Bronchial smooth muscle hypertrophy with proliferation of eosinophils B. Permanent bronchial dilation, which is filled with mucus and neutrophils C. Dilation of air spaces with destruction of alveolar walls D. Hyperplasia of mucus secreting submucosal glands E. Diffuse alveolar damage with leakage of protein rich fluid into the alveolar spaces

281 D. Dx: Chronic Bronchitis. Hallmark is marked hyperplasia of bronchial submucosal glands and bronchial smooth muscle hypertrophy (with LYMPHOCYTIC) infiltrates.Choice A pertains to Asthma, Choice B is bronchiectasis, Choice C is emphysema, Choice E is ARDS

A newborn infant who presented with cyanosis was diagnosed to have Tetralogy of Fallot (TOF), which of the following features of TOF would primarily determine the severity of the disease process? A. Ventricular Septal Defect B. Right Ventricular Hypertrophy C. Subpulmonary stenosis D. Over riding of the aorta E. None of the above

282 C. The clinical consequence primarily depends on the severity of the subpulmonary stenosis as this determines the direction of blood flow. If the stenosis is mild, the abnormality resembles an isolated VSD and the shunt may be left to right without cyanosis (so called pink tetralogy). As the obstruction increases, the greater is the resitance to the RV outflow, producing right to left shunting and cyanosis

Which of the following is true regarding cancer cachexia? A. There is no correlation between tumor burden and the severity of cachexia B. There is a redued basal metabolic rate accompied by reduced food intake C. The weight loss seen results equally from loss of fat and lean muscle D. Satisfactory treatment can be achieved by giving total parenteral nutrition alone E. All of the above

283 C. There is some correlation between tumor burden and severity of cachexia. The BMR is increased depsite reduced food intake. There is currently no satisfactory treatment to cancer cachexia other than removal of the underlying cause, the tumor.

A 72 year old male presented with progressive weight loss, anorexia and abdominal pain radiating to the back. On PE, the gallbladder was noted to be enlarged. An abdominal CT scan was done which demonstrated a mass in the pancreatic head. Which of the following statements is true regarding the probable disease of this patient? A. Weight loss, anorexia and generalized malaise are early signs of the disease B. Abdominal pain is usually the first symptom C. The most common location is in the body of the pancreas D. Jaundice is more prominent when there is involvement of the tail of the pancreas E. CA 19-9 is a specific and sensitive marker and can be used to screen for this disease

284 B. Dx: (most likely) Pancreatic CA. Weight loss, anorexia and malaise are signs of advanced disease. Majority (60%) arise in the pancreatic head. Only 15% arise in the body and 5% in the tail. Obstructive jaundice is associated with most cases of carcinoma involving the pancreatic head. CA 19-9 are useful in following patient response but are too non specific and lack the sensitivity for screening.

Which of the following tumors of the CNS express progesterone receptors which may result in rapid growth during pregnancy? A. Meningioma B. Oligodendroglioma C. Astrocytoma D. Ependymoma E. Medulloblastoma

285 A. Meningiomas are predominantly benign tumors of adult usually attached to the dura, that arise from the meningthelial cell of the arachnoid. Meningiomas are usually slow growing lesions that present with vague non localizing sign , however due to presence of progesterone receptors there can be rapid growth during pregnancy

A 78 year old female presented with severe bone pain, pallor and frequent respiratory tract infections. Laboratory studies showed hypercalcemia and an elevated creatinine. Radiographs of the skull, long bones and spine demonstrated multiple punch out lesions. Which of the following statement is FALSE regarding this disorder? A. Bone marrow aspiration would demonstrate an increased number of T cells B. Neurologic manifestations such as lethargy and confusion are also present due to the hypercalcemia C. Cellular immunity is relatively unaffected D. Perfomance of protein electrophoresis would yield an abnormal spike (M spike) which is most often an IgG E. Renal failure and infections are major causes of death from this disease

286 A. Dx: Multiple Myeloma. Bone marrow aspiration would yield an increase number of plasma cells (>30%)

A 13 year old boy was brought to the ER due to progressive weakness, easy fatigability and weight loss for the past 4 months. In addition, he has recently presented with nausea, vomiting and abdominal discomfort. On PE his blood pressure was noted to be markedly decreased; in addition there was note of increased pigmentation over the creases of his skin. What is the most likey diagnosis of this case? A. Secondary hyperaldosteronism B. Cushing syndrome C. Osteitis fibrosa cystica D. Pheochromocytoma E. Addison disease

287 E. the clinical findings are consistent with primary adrenocortical insufficiency or addison disease. 70% of cases are autoimmune but recently the most frequent cause was TB. Hyperpigmentation is due to compensatory hypothalamic production of proopiomelanocortin, the precursor peptide of both corticotropin and MSH.

Which of the following interleukins has a predominant anti-inflammatory functions? A. IL-1 B. IL-2 C. IL-5 D. IL-10 E. IL-8

288 D. IL with predominantly anti inflammatory functions: IL-4, IL-3, IL-9, IL-10, IL-11, IL-13 ans IL-19

A 26 year old male presented with hematuria, periorbital edema, hypertension and hemoptysis. Serological testing was positive for anti-glomerular basement membrane bodies. Which of the following is the classic histologic finding of this disease? A. Lumpy bumpy immunofluorescence B. Linear immunofluorescence C. Spike and dome appearance of the glomerular basement membrane D. Subenothelial immune complex deposition E. Tram track appearance of the glomerular basement membrane on electron microscopy

289 B. Dx: Goodpasture syndrome. Anti-GBM antibodies are directed against both glomerular and alveolar basement membranes which would account for the nephritic syndrome an hemoptysis seen. Choice A is seen in post streptococcal glomerulonephritis. Choice C is seen in membranous GN. Choice D is seen in lupus nephropathy and MPGN while Choice E is seen in MPGN

A 45-year-old man presents with abdominal pain and hypertension. On physical examination, he is found to have an abdominal mass. Further workup confirms the diagnosis of adult polycystic kidney disease. Which of the following vascular complications is associated with this condition? A. Arteriovenous fistula B. Atherosclerotic aneurysm C. Berry aneurysm D. Dissecting aneurysm E. Luetic aneurysm

29 C. Berry aneurysms, which occur in 10% to 15% of patients with adult polycystic kidney disease, are small saccular lesions that develop at the site of congenital weakness of cerebral arteries, especially those of the circle of Willis. Rupture of these aneurysms is the most common cause of subarachnoid hemorrhage. Arteriovenous fistulas are often secondary to trauma. Dissecting aneurysm is associated with hypertension or with diseases affecting the vascular media, most notably Marfan syndrome. Syphilitic (luetic) aneurysm is associated with tertiary syphilis.

Which of the following statements is true regarding Severe Combined Immunodeficiency (SCID)? A. It is a syndrome with a characteristic defect involving only the humoral response B. Majority are inherited in an autosomal recessive manner C. It is the first human disease in which gene therapy has been successful D. It more commonly affects girls than boys E. Deficiency of adenosine deaminase enzyme (ADA) is seen in the X-linked variant of SCID

290 C. SCID affects both cell mediated and humoral response. It more commonly affects boys than girls. The most common form is inherited in a X-linked manner and involves a common mutation in the common-gamma chain of cytokine receptors. The remaininf cases are inherited as AR, for which the most common cause is ADA deficiency. BMT is the mainstay of treatment and it is the first human disease in which gene therapy has been successful

A 58 year old male, non-smoker, is referred for evaluation of marked erythrocytosis and splenomegaly. A CBC showed a marked elevation of the RBC, WBC and platelet count. Blood uric acid level was also elevated. Oxygen saturation was normal at 98%. Which of the following is characteristic of this disorder ? A. Frequent association with thrombosis or hemorrhagic phenomenon B. Secondary to increased EPO production C. It is most often secondary to hypoxia D. Usually terminates into chronic myelogenous leukemia E. A manifestation of Cushing syndrome

291 A. Dx: Polycythemia vera. The disorder is characterized by prominent erythrocytosis, moderate granulocytosis and thrombocytosis. Because of the hyperviscoity and sludging of blood, there is frequet association with thrombosis or hemorrhagic phenomenon. Marked splenomegaly and a decreased EPO are other classic characteristics. Cushing syndrome and hypoxia are associated with secondary polycythemia which characterized by an increased EPO. About 3% of patients terminate in Acute leukemia not CML

Which of the following features of liver cirrhosis is not associated with impaired estrogen metabolism and consequent hyperestrogenism ? A. Palmar erythema B. Internal hemorrhoids C. Gynecomastia D. Testicular atrophy E. Spider angiomata

292 B. Hyperestrogenism produces local vasodilation in the skin which could account for palmar erythema and spider angiomata. It also causes gynecomastia and testicular atrophy among males. Hemorrhoids are primarily due to the formation of portosystemic shunts.

A 68 year old woman fell and sustained a pelvic fracture. Due to financial constraints she opted to just take pain medications for the moment. After a couple of days she developed a rapidly progresive respiratory failure which eventually led to her death. On autopsy, there was note of numerous petechiae over the conjunctiva and chest wall. What could have probably caused the death of this woman? A. Saddle embolus B. Acute tubular necrosis C. Epidural hematoma D. Bladder rupture E. Fat embolization

293 E. Fat embolism syndrome occurs 2-3 days after severe fracture injury and includes progressive CNS dysfunction and severe respiratory insufficiency. Thrombocytopenia is common and petechial hemorrhage can result from obstruction of the microvasculature by embolic fat droplets. Respiratory insufficiency may be due to injury to pulmonary microvessels with leakage of fluid into the alveoli resulting in ARDS.

Which of the the following features would point more to Crohn's disease rather than Ulcerative colitis ? A. Diffuse colonic involvement B. Marked pseudopolyps C. Transmural inflammation D. Toxic megacolon E. Absence of non caseating granuloma

294 C. The inflammation in UC is limited to the mucosa/submucosal area. Skip lesions are common in crohn's disease.

An 18 year old male presented with swelling of the left knee, an X-ray was done which showed lytic lesions over the said area. A biopsy was done which showed an abundance of multinucleated giant cells with background of mononuclear stromal cells. What is the most likely diagnosis for this case? A. Osteoclastoma B. Osteosarcoma C. Ewing's tumor D. Chondrosarcoma E. Osteochondroma

295 A. Osteoclastoma or giant cell tumor involve both the epihyses and metaphyses. The majority arise in the knee. The typical location causes arthritis like symptoms. Biopsy of the tumor will reveal an abundance of multinucleated giant cells with background of mononuclear stromal cells

A 33 year old woman sought consult for post coital bleeding. Which of the following risk factors would point to cervical carcinoma as the underlying cause? A. Multiple sexual partners B. Young age at first intercourse C. High parity D. Smoking E. All of the above

296 E. Risk factors for Cervical CA: all of the aforementioned choices + male partner with multiple sexual partners, persistent infection with HPV 16 or 18, Immunosuppression, certain HLA subtypes, use of oral contraceptives

A 56 year old male presented with a 2 day history of fever, chills and dysuria. On PE, the prostate was noted to be exquisitely tender and boggy. Which of the following statements is FALSE regarding this condition? A. The diagnosis should be established by biopsy which would show focal areas of necrosis with diffuse edema B. The causative agens are similar to those that cause UTI C. The organisms are usually implanted by intraprostatic reflux of urine from the posterior urethra. D. Organisms can seed the prostate by lymphohematogenous routes from a distant foci of infection E. None of the above

297 A. Dx: acute prostatitis. Biopsy of a man with prostatitis is contraindicated as it may lead to sepsis

Which of the following features is more suggestive of anemia of chronic disease rather than iron deficiency anemia? A. Low serum ferritin level B. Low MCV C. Low TIBC D. High serum iron E. None of the above

298 C. The presence of increased storage of iron in marrow macrophages, a high serum ferritin level and a reduced total iron binding capacity readily rules out IDA as the cause

A 62 year old male, known hypertensive, presented to the ER due to an acute onset of chest pain. He described the chest pain as having a "tearing" quality and radiating to the back. 12L ECG showed no findings of ischemia or infarction Which of the following statements is true regarding the most likely condition ? A. It is usally initiated by a tear over the adventitia of the blood vessel B. The most serious complications occur when the involvement is distal to the subclavian artery C. More than 90% of patients presenting with this condition have an underlying connective tissue disorder D. A damage to the tunica media is a pre requiste for the the disease condition to occur E. Majoirty of cases are found within 10 cm of the the aortic valve

299 E. Dx: Aortic dissection. The dissection is usually initiated by a tear of the intimal layer of the blood vessel. Most serious complications occur with dissection that involves the aorta from the aortic valve to the aortic arch. More than 90% of cases of aortic dissection are men aged 40-60 with antecedent hypertension. A recognizable medial damage seems to be neither a pre requiste for dissection nor a guarantee that dissection is imminent.

A 56 year old hypertensive male presented with left sided hemiparesis on waking up. A CT scan was done which revealed an infarct in the distribution of the MCA. Which of the following is the expected gross pathological finding in the brain? A. Coagulative necrosis B. Caseous necrosis C. Liquefactive necrosis D. Fibrinoid necrosis E. Gangrenous necrosis

3 C. Infarcts in the brain result to liquefactive necrosis while those in all other organs except the brain exhibit coagulative necrosis. Source: Robbins and Cotran Pathologic Basis of Disease 8th ed p. 15

A 55-year-old man presents with a large, black- colored, asymmetric skin lesion with ill-defined borders on his back. He reports a family history of malignant melanoma. Which of the following clinical variants of malignant melanoma has the poorest prognosis? A. Lentigo maligna melanoma B. Superficial spreading melanoma C. Nodular melanoma D. Acral-lentiginous melanoma E. uveal melanoma

30 C. Malignant melanoma arises from melanocytes or nevus cells, is most often associated with excessive sun exposure, and is most common in fair-skinned persons. Of the clinical variants of malignant melanoma, nodular melanoma has the worst prognosis. Malignant melanomas have a better prognosis when characterized by a long period of radial (superficial) growth, as opposed to early vertical growth (as in nodular melanoma).

A 45 year old woman presented with areas of velvety hyperpigmentation involving the axilla and posterior surface of the neck. The said areas initially started as smaller macules but have now progressed to form palpable plaques. Which of the following is an important association of this skin lesion? A. Viral infection B. Visceral malignancy C. Asthma D. Seizure disorders E. None of the above

300 B. Dx: Acanthosis nigricans. Acanthosis nigricans is a brown to black, poorly defined, velvety hyperpigmentation of the skin. It is usually found in body folds, such as the posterior and lateral folds of the neck, the armpits, groin, navel, forehead, and other areas. It typically occurs in individuals younger than age 40, and is associated with obesity or endocrinopathies, and is also indicative of visceral malignancy such as CA of the lungs, breast, stomach or uterus

Which of the following is true about the intrinsic pathway of apoptosis ? A. The upregulation of Bcl-2 regulates the mitochondrial permeability preventing the leakage of cytochrome aa3 B. Activation of Bax and Bak forms oligomers that create a hole in the mitochondrial membrane C. caspase 3 is the critical initiating caspase for apoptosis D. upregulation of Smac/DIABLO activates the physiologic inhibitors of apoptosis E. all of the above

301 B. intrinsic or mitochondrial pathway is the major mechanism of apoptosis in the mammalian cells. The role of Bcl 2 ( in the presence of a growth signal) is to regulate the permeability of the mitochondrial membranes limiting the leakage of cytochrome c.once stress or damage was done to the cell, there is upregulation of Bax and Bak. this in turn inhibits the Bcl2 and forms oligomers creating holes to the mitochondria leading to release of cytochrome c. cytosolic cytochrome c binds to Apaf 1 forming apoptosome to activate caspase 9. the critical initiator of caspase activation. Other factors such as Smac/DIABLO inhibits the inhibitors of apoptosis including caspase 3. Robbins 8th ed p 29.

various experimental models states that the most important way of promoting cellular longevity is through caloric restriction. Which of the family of proteins is responsible for such effect? A. Insulin Growth factor B. telomerase C. sirtuins D. transforming growth factor beta E. none of the above

302 C. sirtuins have histone deacetylase activity and are thought to promote the expression of several genes whose products increase longevity. These products include proteins that increase metabolic activity, reduce apoptosis, stimulate protein folding, and inhibit the harmful effects of oxygen free radicals. Robbins 8th ed p 41.

Which of the followingl nuclear ultrastructural changes can be seen in the affected cell, when the latter is exposed to transient heat ? A. disaggregation B. Appearance of amorphous densities C. Myelin figures D. Blebbing E. All of the above

303 A. when a cell is exposed to transient heat, it undergoes cellular swelling. These are the ultrastructural changes of reversible cell injury: Plasma membrane - blebbing, blunting and loss of microvilli, mitochondria - swelling, appearance of small amorphous densities, ER- dilation, polysome detachment, myelin figures. nuclear 0 disaggregation of granular and fibirllar elements.Robbins 8th ed p 14.

the following proteins are leukocyte molecules responsible for endothelial adhesion except? A. integrin B. Sialyl Lewis X modified proteins C. L selectin D. P selectin E. None of the above

304 D. P selectin is expressed in the endothelium during leukocyte adhesion. Other endothelial molecutes are E -selectin, Glycam 1, ICAM 1, VCAM 1. Robbins 8th ed p 49.

what type of substance accumulate in Niemann pick disease Type C? A. protein B. TAG C. cholesterol D. calcium E. Glycogen

305 C. cholesterol and cholesterol esters accumulate in the following conditions: atherosclerosis, xanthomas, cholesterolosis of gallbladder, and Niemann pick disease type C. page 34-35.

A 65 y/o male went for consult secondary, fever, chronic malaise, and weight loss. Upon physical examination, multiple lymphadenophathies were noted on the oropharyngeal area. hepatosplenomegaly is also noted. Biopsy of the lymph node showed generalized effacement of sinusoid structure by sheets of large cells 4-5x the size of a normal lymphocyte. which of the following is true about the diagnosis ? A. t(14,18) translocation is the hallmark for this condition B. BCL6 dysregulation is common in around 30% of the cases C. it exhibits a high mitotic index with numerous apoptotic cells interspersed with macrophages D. it is correlated with cyclin D1 overexpression E. none of the above

306 B. this is a case of diffuse Large B cell lymphoma. Its characteristics are anaplastic relatively large cells with diffuse pattern of growth. Its cytogenetic, gene expression, profiling and immunohistochemical is heterogenous. However, 30% presents with BCL6 dysregulation. A- follicular lymphoma. C- ( starry sky pattern) Burkitts lymphoma. D- mantle cell lymphoma. Robbins 8th ed - 606-608

what is the most striking histologic finding in desquamative interstitial pneumonia ( smoking related interstitial disease)? A. Multiple macrophages containing dusty brown cytoplasmic pigments B. Thickening of alveolar septa C. presence of lamellar bodies in the macrophage D. presence of plump cuboidal pneumocytes along the septa E. massive interstitial fibrosis

307 A. (SIMILAR TO PREVIOUS BOARD EXAM CONCEPT/PRINCIPLE) Robbins 8th ed p 704

A 48 y/o male complains of intermittent low grade fever, fatigue and weight loss of approximately 1 month duration. Physical examination showed BP 100/60, PR 105 bpm, PP 25 cpm, pale palpebral conjunctiva, palatal and cutaneous petechia.liver edge was palpated 4 cm below the right subcostal margin.blood smear showed large pleomorphic cells with multiple nucleoli and cytoplasmic needle like azurophilic granules. which of the following is true about his condition ? A. t(8,21) balanced translocation imparts a favorable prognosis B. aberrant tyrosine kinase activation is a universal feature of the disease C. diagnosis requires at least 20% blast in the bone marrow D. all of the above E. none of the above

308 D. (SIMILAR TO PREVIOUS BOARD EXAM CONCEPT/PRINCIPLE). This is a case of AML. Robbins 8th ed page 621-624

a 78 y/o male complains of sever chronic low back pain accompanied by weakness and lethargy. PE reveals pale palpebral conjunctiva, bilateral basal crackles and hepatomegaly. Lumbar Xray showed multiple osteolytic lesion scattered around the vertebral bodies. which of the following is consistent about his condition? A. serum M protein level is 2.5 g/dl B. rearrangements involving the Ig heavy chain is rare C. proliferation and survival of neoplastic cells are dependent on cytokines (IL6) D. RANKL upregulation causes apoptosis of osteoblast E. all of the above.

309 C. this is a case of multiple myeloma. Serum M protein is almost always greater than 3gm/dl. ( asymptomatic patients with < 3gm/dl M protein and without skeletal lesion is called MGUS). IgG rearrangement is universal in MM. myeloma cells produce factors that activated RANKL which in turn stimulates osteoclastic activity. (SIMILAR TO PREVIOUS BOARD EXAM CONCEPT/PRINCIPLE)

When ordering academic attire for a recent graduation, a 65-year-old university professor is surprised to find that his hat size has increased. Shortly thereafter, in a routine checkup, serum alkaline phosphatase activity is found to be markedly elevated. Serum calcium and phosphorus are normal. Examination reveals enlargement of the skull with frontal bossing and enlarged maxilla, and hearing loss is evident. Which of the following abnormalities is associated with the bone disorder suggested by these findings? A. Brown tumor of bone B. Defective calcification of osteoid matrix C. Mosaic pattern of bone D. Polyostotic fibrous dysplasia with severe deformity E. Subperiosteal hemorrhage and osteoporosis

31 C. A mosaic pattern of bone caused by increases in both osteoblastic and osteoclastic activity is characteristic of Paget disease of bone (osteitis deformans). Serum alkaline phosphatase is markedly increased. Hearing loss is common (from narrowing of the auditory foramen and compression of the eighth cranial nerve), and an increase in hat size due to frontal bossing is often noted.

A 45 y/o obese female complains of retrosternal pain aggravated after a high fat meal. She also noted frequent regurgitation of sour tasting gastric contents especially at the middle of her sleep.pastmedical history shows that she is asthmatic, hypertensive and diabetic. EGD was done and biopsy was performed in the lower esophageal region showing mild number of intraepithelial eosinophils and basal zone hyperplasia. what is the diagnosis? A. chemical esophagitis B. reflux esophagitis C. eosinophilic esophagitis D. hiatal hernia E. none of the above

310 B. the difference between reflux and eosinophilic esophagitis is that the latter is characterized by abundance of intraepithelial eosinophils. Robbins 8th ed p770. ( SIMILAR TO PREVIOUS BOARD EXAM CONCEPT/PRINCIPLE)

which of the following is a consistent finding in scleroderma except? A. Anticentromere antibody is correlated to CREST syndrome . B. Intimal vascular proliferation is the most consistent finding C. Renal abnormalities resembles that of a malignant hypertension D. synovial inflammation and joint destruction is similar to that of RA.

311 D. Systemic sclerosis is associated with anti Scl70 while CREST or limited scleroderma is correlated to anticentromere antibody. Intimal proliferation of vascular tissue with progressive fibrosis is characteristic of this disease. joint destruction is not common in systemic sclerosis ( distinguishing feature against RA). Robbins 8th ed p 224

which of the following is true about the neuroblastoma ? A. Children younger than 18months have a worst prognosis regardless of stage B. C myc amplification is the most important genetic abnormality used in risk stratification C. only neuroblastoma cells that are located in the adrenal medulla produces catecholamines D. presence of Homer wright pseuodorosettes E. all of the above

312 E. children diagnosed at <18 months of age generally has a good prognosis regardless of the stage. N myc amplication is related to Neuroblastoma not c myc. 90% of neuroblastoma secretes catecholamines regardless of location. Robbins 8th ed p 475-476

What is the most characteristic lesion of HIV associated nephropathy ? A. Capillary wall hyalinosis B. Duplication of the basement membrane C. Retraction of the entire glomerulus D. Diffuse mesangial proliferation E. Fibrin crescents

313 D. HIV is highly associated with a variant of FSGS known as collapsing glomerulopathy. It is characterized by retraction or collapse of the entire glomerulus and hypertrophy and proliferation of visceral epithelial cells. Robbins 8th ed p 926

What is the most serious complication of Tuberculous meningitis? A. Tuberculoma formation B. Fibrinous basal exudates leading to cranial nerve palsies C. Choroid plexus involvement leading to diffuse meningoencephalitis D. Obliterative endarteritis E. none of the above

314 D. the most serious complication of chronic tuberculous meningitis is arachnoid fibrosis leading to hydrocephalus and obliterative endarteritis leading to brain infarction. This is a SIMILAR TO PREVIOUS BOARD EXAM CONCEPT/PRINCIPLE.

which of the following morphologic changes consistent with hepatitis B associated vasculitis? A. Pulmonary circulation is commonly affected B. Focal transmural necrotizing lesions of all stages of activity C. Associated with peripheral hypereosinophilia D. granulomatous inflammation with elastic lamina fragmentation E. all of the above

315 B. 30% of polyarteritis nodosa is associated with Hepatitis B virus infection. PAN is a segmental transmural necrotizing inflammation of small and medium sized arteries typically involving the renal and visceral vessels sparing the pulmonary circulation. all stages of activity coexist in different vessels. this is unlike microscopic polyangitis wherein all lesions are at the same stage of activity and pulmonary circulation is commonly affected. hypereosinophilia is common in churge strauss syndrome while elastic lamina fragmentation is characteristic of giant cell arteritis. Robbins 8th ed 513-515

A 63 y/o male went for consult secondary to skin lesion noted on his forehead. PE showed multiple pruritic, violaceous polygonal papule with lacelike pattern. Which of the morphologic changes is consistent with the diagnosis? A. civatte bodies at the basal epidermis B. pautrier microabscesses C. thinned out stratum granulosum D. acantholytic blisters E. all of the above

316 A. this is a case of lichen planus. It is a form of interface dermatitis with characteristic necrolytic basal layer at the tip of dermal papilla ( civatte bodies). B-mycoses fungoides, C- psoriasis, D- pemphigus vulgaris Robbins 8th ed p 1192

A 25 y/o female complains of a constant left midthigh pain most severe during at sleep. She claims that taking effectively relieved of her symptoms. Femoral xray done showing 2cm subperiosteal lesion located at the midshaft with dense sclerotic background. which of the following is the most likely diagnosis? A. osteoblastoma B. osteoma C. osteoid osteoma D. osteochondroma E. none of the above

317 C. Robbins 8th ed page 1224. osteoid osteoma are by definition less than 2cm in greatest dimension occuring more on the appendicular skeleton. It usually affects teenagers with 2:1 ratio predilection for women. 50% arise in the femur. Most common symptom of this condition is severe nocturnal pain effectively relieved by aspirin. the pain is secondary to the production of prostaglandin E2 by the proliferating osteoblast. morphology will show a tremendous amount of reactive bone formation around the tumor ( nidus).

what is the hallmark feature in diagnosing parathyroid carcinoma? A. Nuclear atypia B. Follicular formation of oxyphil cells C. Local invasion D. Mitotic index E. Capsular invasion

318 C. diagnosis of parathyroid carcinoma based on cytologic detail is unreliable, and invasion of surrounding tissues and metastasis are the only reliable criteria. (SIMILAR TO PREVIOUS BOARD EXAM CONCEPT/PRINCIPLE) Robbins 8th pp 1127-1128

what major fibril protein responsible for secondary amyloidosis on a chronic kidney disease patient on prolonged hemodialysis ? A. Amyloid light chain B. Beta microglobulin C. transthyretin D. calcitonin E. Amyloid precursor protein

319 B. A - primary amyloidosis, C- systemic senile amyloidosis, D- medullary carcinoma E-alzheimers disease. Robbins 8th ed p 252

A 3-year-old African-American man with a history since early childhood of severe anemia requiring many transfusions has nonhealing leg ulcers and recurrent periods of abdominal and chest pain. These signs and symptoms are most likely to be associated with which of the following laboratory abnormalities? A. Decreased erythropoietin B. Increased erythrocyte osmotic fragility C. Schistocytes D. Sickle cells on peripheral blood smear E. Teardrop-shaped cells

32 D. Sickle cell anemia is the most common hereditary anemia in persons of African lineage. Leg ulcers and recurring painful crises are characteristic. In sickle cell anemia, in contrast to sickle cell trait, sickle cells are often seen on the peripheral blood smear.

A 32 y/o male was brought to ER secondary to personality changes. He appears to be combative, confused and agitated. 30mins at the ER, patient experienced status epilepticus which despite all efforts he eventually perished. Biopsy of the brain revealed single cell necrosis with neuronophagia. what is the primary diagnosis? A. Herpes simplex encephalitis B. CMV encephalitis C. Arboviral encephalitis D. HIV encephalitis E. HSV encephalitis

320 C. SIMILAR TO PREVIOUS BOARD EXAM CONCEPT/PRINCIPLE. Arboviral encephalitis is characterized with neuronophagia and microgial nodules. Robbins 8th ed p 1302.

True of cancer cachexia: A. Weight loss results more from loss of muscle than of fat. B. It is caused by nutritional demands of the tumor. C. It has no satisfactory treatment other than removal of the underlying cause. D. Basal metabolic rate is decreased in patients with cancer. E. It is suspected that Il-4 produced by macrophages mediates cachexia.

321 C. Weight loss in cancer cachexia results equally from a loss of muscle and of fat. It is NOT caused by the nutritional demands of the tumor. BMR is increased in patients with cancer. It is suspected that TNF produced by macrophages mediates cachexia. Robbins & Cotran Pathologic Basis of Disease 8th edition, p.320

A paraneoplastic syndrome characterized by gray-black patches of verrucous hyperkeratosis: A. carcinoid syndrome B. trousseau phenomenon C. dermatomyositis D. acanthosis nigricans E. hypertrophic osteoarthropathy

322 D. Trosseau phenomenon is a migratory thrombophlebitis associated with cancer of pancreas or lungs. Robbins & Cotran Pathologic Basis of Disease 8th edition, p.321-322

Causes of non-immune fetal hydrops, except: A. high output heart failure B. turner syndrome C. parvovirus B19 infection D. maternal Rh isoimmunization E. CMV infection

323 D. Robbins & Cotran Pathologic Basis of Disease 8th edition, p.461

This condition is characterized by a sharply segmental acute and chronic vasculitis of medium sized and small arteries, predominantly of the extremities: A. thromboangiitis obliterans B. Churg-Strauss syndrome C. Takayasu arteritis D. polyarteritis nodosa E. temporal arteritis

324 A. Churg-Strauss syndrome - granulomas with geographic patterns of central necrosis and vasculitis. Takayasu arteritis - histologic appearance is indistinguishable from temporal arteritis. Polyarteritis nodosa - segmental transmural necrotizing inflammation of small and medium arteries. Temporal arteritis - nodular intimal thickening reducing luminal diameter, granulomatous inflammation leading to elastic lamina fragmentation. Robbins & Cotran Pathologic Basis of Disease 8th edition, p.512-514

The most common primary tumor of the heart: A. lipoma B. myxoma C. rhabdomyoma D. sarcoma E. papillary fibroelastoma

325 B. The most frequent primary tumor of the heart in children, on the other hand, is rhabdomyosarcoma. Robbins & Cotran Pathologic Basis of Disease 8th edition, p.584

The most specific morphologic finding in hereditary spherocytosis: A. reticulocytosis B. hemosiderosis C. spherocytosis D. extramedullary hematopoiesis E. cholelithiasis

326 C. Spherocytosis is distinctive but not pathognomonic of Hereditary Spherocytosis. Robbins & Cotran Pathologic Basis of Disease 8th edition, p.643

The most common hereditary disease associated with life threatening bleeding: A. von Willebrand factor deficiency B. hemophilia A C. hemophilia B D. Bernard Soulier syndrome E. Glanzmann Thrombasthenia

327 B. von Willebrand factor deficiency, on the other hand, is the most common inherited bleeding disorder of humans. Robbins & Cotran Pathologic Basis of Disease 8th edition, p.672

The major condition/s associated with bronchiectasis: A. obstruction B. infection C. airway remodeling D. A & B E. all of the above

328 D. Airway remodeling is a histologic finding in bronchial asthma. Robbins & Cotran Pathologic Basis of Disease 8th edition, p.692

True of typhoid fever, except: A. Humans are its sole reservoir. B. Gallbladder colonization is associated with the chornic carrier state. C. Morphologically oval ulcers perpendicular to the axis of the ileum are seen. D. Liver, bone marrow, lymph nodes may show typhoid nodules. E. Patients with sickle cell disease are susceptible to Salmonella osteomyelitis.

329 C. The oval ulcers in typhoid fever are oriented along the axis of the ileum. Robbins & Cotran Pathologic Basis of Disease 8th edition, p.801

For the past week, a 65-year-old woman has been treated for a severe infection with broad- spectrum antibiotics, and she had recovered well. Over the past day, however, she has developed foul-smelling, voluminous, greenish, watery diarrhea, as well as abdominal pain and fever. Which of the following is the mechanism associated with this condition? A. Aggregation of bacterial colonies on the lumen, forming pseudomembranes B. Bacterial release of exotoxin, inducing necrosis of the mucosa C. Physical invasion of bacteria into the superficial mucosa, leading to pseudomembrane formation D. Selective killing of C. difficile bacteria by antibiotics E. Spread of the previous infection to the coon

33 B. Pseudomembranous colitis is caused by overgrowth of C. difficile. This organism produces exotoxin that induces necrosis of the superficial mucosa, leading to pseudomembrane formation. The bacteria itself does not invade the mucosa. This condition most often occurs in patients with a history of broad-spectrum antibiotic use, because elimination of normal intestinal flora promotes overgrowth of C. difficile.

A patient presents with diarrhea, weight loss, malabsorption, along with arthritis and fever. Biopsy of the small intestine reveals accumulation of distended, foamy macrophages containing PAS(+), diastase resistant granules. Rod-shaped bacilli can be identified by electron microscopy. What is the most likely diagnosis? A. Whipple disease B. intestinal TB C. lactase deficiency D. pseudomembranous colitis

330 A. PAS(+) macrophages are also seen in Intestinal TB, but it is also usually AFB(+). Robbins & Cotran Pathologic Basis of Disease 8th edition, p.803

The disease that is most likely to give rise to hepatocellular carcinoma: A. hepatitis C B. hepatitis B C. non-alcoholic steatohepatitis D. hereditary tyrosinemia E. alpha1-antitrypsin deficiency

331 D. Robbins & Cotran Pathologic Basis of Disease 8th edition, p.878

The most common congenital anomaly of the pancreas: A. agenesis B. annular pancreas C. ectopic pancreas D. pancreas divisum

332 D. Pancreas divisum is the failure of fusion of fetal duct systems of dorsal and ventral pancreatic primordia. Robbins & Cotran Pathologic Basis of Disease 8th edition, p.892

Most pancreatic cancers arise from: A. pancreatic head B. pancreatic body C. pancreatic tail D. diffusely involving the entire gland

333 A. Head - origin of 60% of pancreatic cancer; body - 15%; tail - 5%. Robbins & Cotran Pathologic Basis of Disease 8th edition, p.902

Foot process effacement associated with normal glomeruli by light microscopy makes this diagnosis: A. membranous nephropathy B. minimal change disease C. PSGN D. MPGN E. Berger disease

334 B. In Minimal change disease, the prinicipal lesion is in the visceral epithelial cells which show a uniform and diffuse effacement of foot processes. Robbins & Cotran Pathologic Basis of Disease 8th edition, p.925

A 27-year-old female with hypertension was found to have renal artery stenosis. What is the expected morphologic finding on examination? A. a concentrically placed atheromatous plaque with superimposed thrombosis B. eosinophilic granular change in blood vessel wall staining (+) for fibrin C. fibromuscular thickening involving the media of artery D. intimal thickening caused by proliferation of elongated, concentrically arranged smooth muscle cells

335 C. Choice A is seen in males, diabetics, advanced age. Choice B in malignant HPN (fibronoid necrosis described). Choice C (the answer) refers to fibromuscular dysplasia, ocuring in females, in their 3rd-4th decades. Choice D describes onion skinning seen in malignant HPN. Robbins & Cotran Pathologic Basis of Disease 8th edition, p.951

Consistent with benign ulcers, except: A. oval, sharply punched out defect B. mucosal margin with slight overhang from base C. hemorrhage and fibrin deposition on gastric serosa D. heaped up margins E. thin layer of fibrinoid deposit at base underlaid by predominantly neutrophilic inflammatory infiltrate

336 D. SIMILAR TO PREVIOUS BOARD EXAM CONCEPT/PRINCIPLE. Robbins & Cotran Pathologic Basis of Disease 8th edition, p.780-781

True of ulcerative colitis, except: A. normal serosal surface of colon B. toxic megacolon may complicate C. earliest lesion is an aphthous ulcer D. backwash ileitis E. absence of granulomas

337 C. The earliest lesion in Crohn's disease is an aphthous ulcer. Robbins & Cotran Pathologic Basis of Disease 8th edition, p.811-812

A 21-year-old male presenting with recurrent colicky abdominal pain from GI obstruction and transient intussesceptions. On physical examination, dark blue to brown maculesa are noted around his mouth, eyes, nostrils, buccal mucosa. What is the most likely diagnosis? A. Cronkhite Canada syndrome B. Cowden syndrome C. Bannayan-Ruvalbaca-Riley syndrome D. Peutz-Jeghers syndrome

338 D. Robbins & Cotran Pathologic Basis of Disease 8th edition, p.818

The main regulatory factor for iron absorption? A. ferritin B. hepcidin C. transferrin D. serum Fe E. TIBC

339 B. Robbins & Cotran Pathologic Basis of Disease 8th edition, p.862

A 50-year-old woman with a 20-year history of type 2 diabetes mellitus presents with proteinuria, hypoalbuminemia, edema, and hyperlipidemia. She has not monitored her serum glucose levels over the past several years. What is the classic morphologic finding in this condition? A. Crescentic formation in glomeruli on light microscopy B. Intramembranous and epimembranous immune complex deposits in the glomerular basement membrane on electron microscopy C. Nodular accumulations of mesangial matrix on light microscopy D. Sclerosis within capillary tufts that involves only some glomeruli and only parts of affected glomeruli on light microscopy E. Wire-loop abnormalities from immune complex deposits and thickening of the glomerular basement membrane on light microscopy

34 C. SIMILAR TO PREVIOUS BOARD EXAM CONCEPT/PRINCIPLE. Diabetic nephropathy manifests clinically as the nephrotic syndrome; however, this syndrome is compounded by renal failure and hypertension. Ultrastructural changes include a marked increase in the thickness of the glomerular basement membrane and mesangial accumulation of glycosylated basement membranelike material. Light microscopy findings include diffuse glomerulosclerosis (a diffuse increase in mesangial matrix) and nodular glomerulosclerosis (nodular accumulations of mesangial matrix).

What refers to de novo formation of blood vessels during embryogenesis? A. angiogenesis B. neovascularization C. arteriogenesis D. vasculogenesis

340 D. Angiogenesis/neovascularization is new vessel formation in the mature organism. Arteriogenesis involved remodeling of existing arteries in response to chronic changes in pressure and flow. Robbins & Cotran Pathologic Basis of Disease 8th edition, p.489

A 56 year old hypertensive and diabetic male complained of chest pains and was brought to the emergency room. ECG showed ST segment elevation and lab work-up showed elevated troponin and CK-MB. What type of necrosis is expected in the cardiac muscle? A. Coagulation B. Liquefactive C. Gangrenous D. Enzymatic fat

341 A.

An 18 year old girl sustained a gaping wound in the thigh which was not sutured. The healed wound showed an elevated scar which did not go beyond the original margin of the wound. Which of the following abnormalities in repair occurred in her case? A. Fibromatosis B. Deficient wound contraction C. Failure of collagen maturation D. Excessive granulation tissue formation

343 D.

A 15 year old girl had episodes of sneezing with watery eyes and runny nose for the past 2 weeks. On physical examination, she has red, swollen nasal mucosa. She has had similar episodes in the summer, when the amount of pollen in the air is high. Her symptoms are most likely to be mediated by the release of which of the following chemical mediators? A. Complement C3b B. Histamine C. Platelet activating factor D. Immunoglobulin G

345 B. Allergic reactions and atopic diseases, such as allergic rhinitis, are examples of type I hypersensitivity reactions which are mediated by substances released from mast cells, such as histamine.

A 30 year old pedicab driver complains of productive cough, fever, and dyspnea. He was diagnosed with Bacterial Pneumonia. On PE, dullness on percussion over the left lung field was elicited. What microscopic finding can explain these signs and symptoms? A. Interstitial edema and interstitial capillary vessel congestion B. Intraalveolar leukocyte accumulation with red cell exudation C. Minimal fluid accumulation in pleural cavities with fibrin deposition D. Pulmonary cavitary formation with intracavitary necrotic debris accumulation

348 B. PE findings suggest consolidation. Choice B is consistent with pathologic findings of consolidation.

A 60 year old patient with an 80 pack year smoking history was seen at the ER because of productive cough, weight loss, and an episode of hemoptysis. A chest X-ray showed a 6 cm mass on the middle lobe. Bronchoscopy revealed an endophytic mass almost obstructing the segmental bronchus. Which pathology is associated with the most likely diagnosis for this case? A. Keratin pearl formation B. Mucin lakes C. Psammoma body formation D. Giant cell formation

349 A. The most likely diagnosis in this case is Squamous cell carcinoma. Keratin pearls are seen in this type of cancer.

A 23-year-old woman presents with cervical and mediastinal lymphadenopathy. Biopsy of a cervical lymph node reveals a nodular appearance with fibrous bands, effacement of the lymph node architecture, and numerous lacunar cells. Which of the following is characteristic of this disorder? A. Benign neoplasm B. Frequent association with EBV infection C. Most often a complication of human immunodeficiency virus infection D. Peak incidence in early childhood E. Relatively favorable clinical course

35 E. SIMILAR TO PREVIOUS BOARD EXAM CONCEPT/PRINCIPLE. The diagnosis is Hodgkin lymphoma, nodular sclerosing subtype. This form of Hodgkin lymphoma differs from other forms of classical Hodgkin lymphoma in being the most common in young women, having a relatively favorable clinical course, and having little association with EBV infection. Lacunar cells are considered a Reed-Sternberg cell variant, and the diagnosis of NS can be based on the finding of fibrous bands and lacunar cells.

Which of the following is an essential component in the definition of Barrett esophagus? A. Columnar epithelium B. Intestinal type epithelium with goblet cells C. Fundic type gastric epithelium D. Cardiac type gastric epithelium

350 B. Reference: Robbins, Pathologic Basis of Disease, 8th ed. p. 770

A 5 year old boy with stunted growth has worsening headaches for 2 months. A cranial CT scan reveals a 1.5 cm mass expanding the sella turcica. The mass is cystic with scattered calcification. Which of the following is the most likely diagnosis? A. Neuroblastoma B. Craniopharyngioma C. Pituitary adenoma D. Mature teratoma

351B.

In benign hypertension, the arterioles of the kidney would show: A. Arteriolitis B. Onion-skinning C. Fibrinoid necrosis D. Hyaline thickening

352 D. Onion skinning and fibrinoid necrosis are associated with malignant hypertension.

A 5 year old boy is brought to the paediatrician because of periorbital and bipedal edema. Urinalysis showed the following: sugar - negative; protein - 4+; RBC - 0-1/hpf; leucocytes - 0-1/lpf. Blood chemistry shows: albumin - 2g/dL; cholesterol - 1.8 mmol/L. Light microscopy of a kidney biopsy of this patient would most likely show: A. Normocellular glomeruli B. Glomeruli with endocapillary hypercellularity C. Glomeruli with extracapillary hypercellularity D. Hypocellular glomeruli

353 A. The most common cause of nephrotic syndrome in children is Minimal Change Disease. A renal biopsy would show no changes in light microscopy, but may show effacement of podocytes on electron microscopy.

A 20 year old female was diagnosed with SLE 5 years ago. A renal biopsy is performed because she presents now with nephrotic and nephritic syndromes. Light microscopy of the biopsy would most likely show: A. Spike and dome appearance B. Tram-track basement membrane C. Wire-loop lesions D. Normocellular glomeruli

354 C. Wire loop lesions seen in lupus nephritis reflect active disease. Reference: Robbins, Pathologic Basis of Disease, 8th ed. p. 219

A 10 year old boy consulted a physician because of tea coloured urine and puffiness of the eyelids, especially noted in the morning. Blood pressure is 140/90. These symptoms were noted 3 weeks after he had fever and sore throat. Urinalysis showed: protein - 4+; sugar - negative; leucocytes - 0-1/hpf; RBC - 10- 20/hpf; red cell cast - 1-2/lpf; granular cast - 0- 1/lpf; waxy cast - 0-1/lpf. Electron microscopy of this patient’s renal biopsy would most likely show: A. Electron dense deposits with fingerprint appearance B. Subepithelial humps C. No electron dense deposits D. Subendothelial humps and spikes

355 B. Reference: Robbins, Pathologic Basis of Disease, 8th ed. p. 919

A 5 year old boy was admitted for high grade fever with nuchal rigidity on PE. CSF examination revealed moderate pleocytosis with lymphocyte predominance, with increased protein and decreased glucose. What is the most likely diagnosis? A. Acute bacterial meningitis B. Viral meningitis C. Tuberculous meningitis D. Subarachnoid hemorrhage

356 C. CSF findings are characteristic of Tuberculous meningitis. Viral meningitis would present with moderately elevated protein, and usually normal glucose levels. Bacterial meningitis presents with neutrophil predominance.

A 25 year old woman with recent onset of a major depressive disorder ingests an entire bottle of acetaminophen. She becomes progressively obtunded over the next 8 hours. Which of the following microscopic findings is most likely to be present in her liver 3 days following ingestion? A. Normal histology B. Extensive necrosis C. Severe steatosis D. Bridging fibrosis

357 B. Reference: Robbins, Pathologic Basis of Disease, 8th ed. p. 416

A 9 year old girl has a firm well-circumscribed midline nodule on her neck that moves upwards with protrusion of the tongue. FNAB shows mucus and benign epithelial cells. Which is a correct statement about her condition? A. The serum TSH is probably increased B. This is due to cystic dilatation of an embryologic remnant C. This is a very common condition especially among Asians D. Papillary carcinoma is never associated with it

358 B. Diagnosis is Thyroglossal duct cyst.

A 30 year old motorcycle driver develops polyuria and polydipsia following a motor vehicular accident. His condition is most probably the result of which of the following? A. Deficiency of vasopressin B. Excess of growth hormone C. Deficiency of insulin D. Excess of PTH

359 A.

A 15-year-old boy presents with a pathologic fracture following a minor injury on the soccer field. The area of fracture is surrounded by a large tumor which shows marked pleomorphism, high mitotic activity, and extensive cartilaginous differentiation on microscopy. The most likely diagnosis is A. giant cell tumor B. osteochondroma C. chondrosarcoma D. osteosarcoma E. Ewing sarcoma

36 D. In young patients, bone malignancies showing prominent cartilaginous differentiation are almost assuredly chondroblastic osteosarcomas, rather than chondrosarcomas. Conventional chondrosarcomas occur almost exclusively in older patients.

Which of the following is usually produced by an acidophilic pituitary adenoma? A. TSH B. FSH/LH C. Growth Hormone D. ACTH

360 C. Acidophilic cells: GH, prolactin Basophilic cells: FSH, LH, ACTH, TSH

Which of the following is/are the hallmark/s of cancer? A. Evasion of apoptosis B. Metastasis C. Invasion D. Sustained angiogenesis E. All of the above

361 E. There are 8 hallmarks of cancer: Selfsufficiency in growth signals, Insensitivity to growthinhibitory signals, Altered cellular metabolism, Evasion of apoptosis, Limitless replicative potential (immortality), Sustained angiogenesis, Ability to invade and metastasize and Ability to evade the host immune response. Robbins 9th pg 282-283

A 60/F who has recurrent fever, fatigue, weight loss, cough and night sweats came to you for consult. You ordered chest xray which showed enlarged hilar lymph nodes and multiple 1-2 cm non-cavitating lesions. Biopsy of the lesions was done showing non-caseating granulomata with fibrosis. The most probable diagnosis is: A. Lung carcinoma B. Pulmonary tuberculosis C. Mycobacterium avium-intracellulare infection D. Sarcoidosis E. Silicosis

364 D. Sarcoidosis is a multisystem disease of unknown etiology; the diagnostic histopathologic feature is the presence of noncaseating granulomas in various tissues. Clinically, It may be discovered unexpectedly on routine chest films as bilateral hilar adenopathy. It may presents with shortness of breath, cough, chest pain, hemoptysis and constitutional signs and symptoms (fever, fatigue, weight loss, anorexia, night sweats). Robbins 9th pg 693-694

A 60/M presents with seizures and left sided weakness. MRI was done which showed a large heterogenous mass on the right cerebral hemisphere. The patient was then referred to Dr. Karl F who then proceeded to remove the mass as best as he could. As the pathologist, you examined sections of the mass and you noted multiple foci of necrosis, hemorrhage and pseudo-palisading tumor cells. What is your diagnosis? A. Oligodendroglioma B. Glioblastoma multiforme C. Anaplastic astrocytoma D. Ependymoma E. Pilocytic astrocytoma

365 B. The histologic appearance of glioblastoma is similar to anaplastic astrocytoma with the additional features of necrosis and vascular/endothelial cell proliferation.Tumor cells collect along the edges of the necrotic regions, producing a histologic pattern referred to as pseudo-palisading. Robbins 9th pg 1308

A previously well 60/M presents with sudden painless massive LGIB and he was then immediately brought to a local hospital. In the hospital, the bleeding was still massive and persistent. Sigmoid colonoscopy was done which failed to reveal the site of bleeding. The only finding was that there were multiple outpouchings in the sigmoid colon. Due to the difficulty in localizing and in controlling the bleeding, total colectomy was done. As the pathologist evaluating the specimen, you found multiple outpouchings in the sigmoid and tortuous and ectatic submucosal and mucosal vessels in the cecum. What is your diagnosis? A. Diverticulosis B. Diverticulitis C. Angiodysplasia D. Ulcerative colitis E. Mesenteric ischemia

366 C. Angiodysplasia, a lesion with malformed submucosal and mucosal blood vessels, occurs most often in the cecum or right colon and usually presents after the sixth decade of life.The lesions are characterized by ectatic nests of tortuous veins, venules, and capillaries. The vascular channels may be separated from the intestinal lumen by only the vascular wall and limited injury may therefore result in significant bleeding. Robbins 9th pg 780-781; Symptoms of diverticular disease include intermittent cramping, continuous lower abdominal discomfort, constipation, distention, or a sensation of never being able to completely empty the rectum. Occasionally there may be minimal chronic or intermittent blood loss, and, rarely, massive hemorrhage. pg 804

A 45/M was diagnosed to have HNPCC. What is the underlying molecular pathology which led to the development of the patient's condition? A. DNA mismatch repair deficiency B. APC gene mutation at 5q21 C. K-RAS gene mutation at 12p12 D. Loss of heterozygosity at 18q21 E. Loss of function of the gene STK11

367 A. In HNPCC DNA mismatch repair deficiency causes mutations to accumulate in microsatellite repeats, a condition referred to as microsatellite instability . Robbins 9th pg 812. Choice B-D refers to the adenoma-carcinoma sequence with the last step being TP53 17q13 mutation pg 811. Loss-of-function mutations in the gene STK11 are present in approximately half of individuals with familial Peutz- Jeghers syndrome pg 806

Which of the following is/are small round blue cell tumors? A. Neuroblastoma B. Wilm's Tumor C. Ewing sarcoma D. A and C E. All of the above

368 E. Other examples include rhabdomyosarcoma, medulloblastoma, small cell lung carcinoma, small-cell lymphom

A 35-year-old woman had a firm nodule palpable on the dome of the uterus six years ago on routine examination. The nodule has slowly increased in size and is now about twice the size it was when first discovered. She is asymptomatic. She opted to have total hysterectomy done. As the pathologist, you found that the uterus was grossly distorted and asymmetric. The mass measures 14 x 10 x 12 cm located at the posterior midcorpus. Sections of the mass showed a whorled white surface with multiple large areas of necrosis and ill- defined borders. Which of the following is most likely? A. Adenocarcinoma B. Leiomyosarcoma C. Adenomyosis D. Leiomyoma E. Metastasis

369 B. Leiomyosarcomas are uncommon malignant neoplasms that are thought to arise from myometrial or endometrial stromal precursor cells, rather than from degenration of leiomyomas. Leiomyomas are sharply circumscribed with low mitotic index in contrast to leiomyosarcomas Robbins 9th pg 1020

A 45-year-old woman presents to her primary care physician with jaundice, pruritus, and periocular and intradigital xanthomas. Her laboratory results indicate a significantly increased alkaline phosphatase as well as a positive test for antimitochondrial antibodies. The most likely cause of her symptoms is A. leptospirosis B. macronodular cirrhosis C. primary biliary cirrhosis D. primary sclerosing cholangitis E. secondary biliary cirrhosis

37 C. Primary biliary cirrhosis is an autoimmune condition that typically presents in middle-aged women. The itching and hypercholesterolemia are secondary to severe obstructive jaundice. Leptospirosis is a condition caused by a treponemal bacterium that results in jaundice, renal failure, and hemorrhagic phenomena. Macronodular cirrhosis is usually a result of hepatitis B or hepatitis C infection. Primary sclerosing cholangitis is associated with ulcerative colitis and with an increased incidence of cholangiocarcinoma. Secondary biliary cirrhosis is caused by extrahepatic biliary obstruction.

Which of the following is/are responsible for the production IL-1? A. Macrophage B. TH1 cell C. TH2 cell D. B cells E. B and C

370 A. Macrophages produce IL-1, 6, 8,12, and TNF alpha; all T cells produce IL-2, 3; TH1 cells produce IFN gamma; TH2 cells produce IL-4,5,10 Robbins 9th pg 198

A 35/M presents with the classic triad of scanning speech, intention tremor and nystagmus. What type of hypersensitivity reaction is responsible for the patient's disorder? A. Type I B. Type II C. Type III D. Type IV E. The illness described is not caused by a hypersensitivity reaction

371 D. This classic triad or Charcot's neurologic triad is associated with multiple sclerosis. MS is a type IV hypersensitivity reaction Robbins 9th pg 209

A 10/M, diagnosed to have mental retardation, underwent karyotypic analysis. The was no aneuploidy present but a discontinuity of staining was seen in the long arm of the X chromosome. Expansion of what type of trinucleotide repeat sequence can be expected from the patient? A. CTG B. CGG C. GAA D. CAG E. CCC

374 B. CTG - myotonic dystrophy; GAA - Friedreich ataxia; CAG - Huntington disease Robbins 9th pg 168-169

What disorder is due to a single gene mutation causing resistance to cleavage and inactivation by protein C resulting in a hypercoaguable state? A. Factor V Leiden B. Sickle cell anemia C. Protein C deficiency D. Protein S deficiency E. Antithrombin III deficiency

375 A. The mutation results in a glutamine to arginine substitution at amino acid residue 506 that renders factor V resistant to cleavage and inactivation by protein C. Robbins 9th pg 123-124

A 34/F with recurrent headaches was diagnosed to have a suprasellar tumor 2 months ago. While walking she suddenly complained of blindness and was subsequently rushed to the hospital. In the ER she was vomiting and complained of a severe headache. What is your diagnosis? A. Pituitary microadenoma B. Craniopharyngioma C. Pituitary apoplexy D. Pituitary macroadenoma E. Prolactinoma

376 C. Acute hemorrhage into an adenoma is associated with clinical evidence of rapid enlargement of the lesion, a situation appropriately termed pituitary apoplexy. Robbins 9th pg 1075

A 18/M presents with periorbital edema, proteinuria and hematuria. You suspect MPGN type 1 for this case. If you were to do kidney biopsy, you would expect to find? A. Large and hypercellular glomeruli with thickened and duplicated GBM B. Sclerotic segments with collapse of capillary loops, increase in matrix and segmental deposition of plasma proteins along the capillary wall C. Uniform and diffuse effacement of foot processes in the visceral epithelial cells D. GBM shows irregular foci of thickening alternating with thinning and splitting and lamination of the lamina densa E. None of the above

377 A. B - FSGS; C - MCD; D - Alport syndrome Robbins 9th pg 917, 919, 920, 924

Which of the following best describes the pathology of a Rotavirus infection? A. Pseudomembrane formation made up of an adherent layer of inflammatory cells B. The mucosa of the left colon is hemorrhagic and ulcerated, and pseudomembranes may be present C. Selective infection and destruction of mature enterocytes in the small intestine with the villus surface repopulated by immature secretory cells D. Enlargement of Peyer patches in the terminal ileum with oval ulcers that may perforate E. None of the above

378 C. Robbins 9th pg 793 Choice A describes C. difficile pg 791 Choice B describes Shigella pg 788 Choice D describes Salmonella pg 789

The characteristic histologic finding/s of asthma include/s? A. Thickening of the airway wall B. Hypertrophy of the bronchial wall muscle C. Increase in the size of the submucosal glands and number of airway goblet cells D. A and B E. All of the above

379 E. The other characteristic histologic findings of asthma, collectively called airway remodeling, include thickening of airway wall, subbasement membrane fibrosis (due to deposition of type I and III collagen), increased vascularity, increase in the size of the submucosal glands and number of airway goblet cells and hypertrophy and/or hyperplasia of the bronchial wall muscle Robbins 9th pg 682

The chest radiograph of a 23-year-old medical student reveals a calcified cavitary pulmonary lesion. The tuberculin test is positive, but sputum smears and cultures are negative for Mycobacterium tuberculosis. A presumptive diagnosis of secondary tuberculosis is made. If further studies, including a biopsy, were performed, which of the following findings would justify the diagnosis of secondary tuberculosis, as contrasted to primary tuberculosis? A. Calcification B. Caseating granulomas C. Cavitation D. Langhans giant cells E. Positive Tuberculin test results

38 C. Cavitation occurs only in secondary tuberculosis. Both primary and secondary tuberculosis are characterized by caseating granulomas, often with Langhans giant cells, which heal by scarring and calcification. The skin test result for tuberculin sensitivity is positive in both forms.

Choristomas are: A. Excessive focal overgrowths of cells and tissues native to the organ where it occurs B. Malignant germ cell tumors composed of syncitiotrophoblasts and cytotrophoblasts C. Helter-skelter collection of differentiated cells or organoid structures (neural tissue, islands of cartilage, etc) embedded in a myxoid stroma D. Collection of normal cells or tissues in abnormal locations E. None of the above

380 D. Choice A refers to hamartomas Robbins 9th pg 473 Choice B - choriocarcinoma Choice C - teratoma

What is the most common site of metastasis of lung cancers? A. Bone B. Liver C. Adrenal D. Brain E. Kidney

381 C.

The pathognomonic inclusion bodies in nerve cells infected by the rabies virus are most commonly found in which part of the CNS? A. Cerebral cortex B. Hippocampus C. Basal ganglia D. Cerebellum E. Medulla oblongata

382 B. SIMILAR TO PREVIOUS BOARD EXAM CONCEPT/PRINCIPLE. The inclusion bodies refer to Negri bodies, which are most commonly seen in the hippocampus. Cerebellum is the 2nd most common site.

In which of the following conditions is fibrinous pericarditis least likely to occur? A. Dressler syndrome B. Tuberculosis C. Systemic lupus erythematosus D. Rheumatic heart disease E. None of the above

383 B. Caseous and adhesive pericarditis are the forms associated with TB.

Which of the following laboratory findings will you expect in patients with hemophilia A? A. Prolonged prothrombin time B. Prolonhged partial thromboplastin time C. Decreased platelet count D. Prolonged bleeding time E. Both A and D

384 A.

ABO blood type incompatibility is an example of which hypersensitivity reaction? A. Type I hypersensitivity reaction B. Type II hypersensitivity reaction C. Type III hypersensitivity reaction D. Type IV hypersensitivity reaction E. None of the above

385 B.

The following laboratory findings are consistent with iron-deficiency anemia except: A. Hypochromic RBCs B. Decreased MCV C. Increased RDW D. Anisocytosis E. Decreased MCHC

386 D. SIMILAR TO PREVIOUS BOARD EXAM CONCEPT/PRINCIPLE.

Which of the following is not a small-vessel vasculitides? A. Polyarteritis nodosa B. Henoch-Schonlein purpura C. Churg-Strauss syndrome D. Wegener granulomatosis E. None of the above

387 A. Polyarteritis nodosa is classically a medium-vessel disease.

A renal biopsy that shows hypercellular glomeruli on light microscopy, "starry sky" pattern of immunofluorescence and subepithelial immune complex humps on electron microscopy is most consistent with which of the following diagnosis? A. Rapidly progressive glomerulonephritis B. Acute poststreptococcal glomerulonephritis C. IgA nephropathy D. Membranoproliferative glomerulonephritis E. Lupus nephritis

388 B.

Which of the following describes the most common type of choledochal cyst? A. Fusiform dilatation of the common bile duct B. Diverticulum arising from the common bile duct and attached to it by a narrow stalk C. Focal dilatation of the intraduodenal portion of the common bile duct D. Multiple saccular dilatations of the intra- and extra-hepatic bile ducts E. Saccular dilatations of the intrahepatic bile ducts without biliary obstruction

389 A. A: type I. B: type II. C: type III. D: type IV. E: type V (Caroli disease)

A 40-year-old woman presents with a painless mass anterior to her left ear. The mass had been slowly enlarging over the past year. The mass is firm and nontender. Computed tomography and magnetic resonance imaging reveal a well-circumscribed, homogeneous mass within the left parotid gland. Biopsy reveals anastomosing strands of stellate and fusiform epithelial cells embedded in a myxoid stroma. Which of the following is a characteristic of the lesion? A. It is also called papillary cystadenoma lymphomatosum. B. It is most often localized to the submandibular gland. C. It is the most common malignant salivary gland tumor. D. Recurrence often takes place after surgical resection. E. Surgical resection should not be performed, because this condition is usually already metastatic on diagnosis.

39 D. Approximately 80% to 90% of salivary gland tumors originate in the parotid gland and, of these, approximately 70% are pleomorphic adenomas. The term “mixed tumor†properly applies to this benign tumor, which often demonstrates myxoid and cartilage-like elements in addition to stellate or fusiform epithelial cells. Complete surgical resection is difficult because of the tumor’s proximity to the facial nerve, and, thus, recurrence is frequent.

What is the most common clinical presentation of multiple sclerosis? A. Nystagmus B. Motor weakness C. Optic neuritis D. Intention tremor E. Dementia

390 C.

Which of the following features would make you favor a diagnosis of ulcerative colitis over Crohn disease? A. Pseudopolyp formation B. Fistula formation C. Non-caseating granuloma D. Paneth cell metaplasia E. Aphthous ulcers

391 A. The other choices are more characteristic of Crohn disease.

Which of the following features would make you suspect that an ulcer is malignant? A. Edematous ulcer collar with overhanging mucosal edges B. Flat-based ulcer with heaped up edges C. Radiating folds extending into the crater of the ulcer D. Depth of the ulcer is greater than its width E. Smooth ulcer mound with tapering edges

392 B. SIMILAR TO PREVIOUS BOARD EXAM CONCEPT/PRINCIPLE. Option B is a classic feature of a malignant ulcer. In barium studies of the upper GI tract, this will present as the "Carmen meniscus sign". The other options describe a benign ulcer.

Which of the following viral hepatitis infection is least associated with fulminant hepatitis? A. Hepatitis B B. Hepatitis C C. Hepatitis D D. Hepatitis E E. None of the above

393 B. Hepatitis C is associated with 50% chronicity but not with fulminant hepatitis. Hepatitis D can lead to fulminant hepatitis in the setting of co-infection with Hep B. Hepatitis E is associated with fulminant hepatitis among pregnant women.

Among patients with chronic viral hepatitis infection who develop liver cirrhosis, what percentage will continue to progress into hepatocellular carcinoma? A. <2% B. 5% C. 6-15% D. 12-20% E. 50%

394 C. SIMILAR TO PREVIOUS BOARD EXAM CONCEPT/PRINCIPLE.

The presence of a hyperpigmented, ulcerating plaque with irregular borders and asymmetric shape in which of the following body parts should make you worry most? A. Face B. Nape C. Chest D. Palms E. Back

397 D. SIMILAR TO PREVIOUS BOARD EXAM CONCEPT/PRINCIPLE. Acral lentiginous melanoma, which occurs on the palms and soles and beneath the nails, has the worst prognosis among the different types of melanoma.

A 32-year old female presented with symptoms of fatigue, weight gain, cold intolerance, constipation and thinning of her hair. On physical examination, you noted a firm, mildly enlarged but painless thyroid gland. Biopsy revealed diffuse inflammatory infiltrates, atrophy of the thyroid follicles and characteristic Hurthle cells. What is your most likely diagnosis? A. Hashimoto thyroiditis B. Lymphocytic thyroiditis C. Subacute thyroiditis D. Graves disease E. Reidel thyroiditis

398 A. SIMILAR TO PREVIOUS BOARD EXAM CONCEPT/PRINCIPLE. Lymphocytic thyroiditis also present with painless enlargement of the thyroid gland, but it does not exhibit Hurthle cells on biopsy. Subacute thyroiditis presents with painful thryoid enlargement.

A history of bladder exstrophy is a risk factor for the development of which type of bladder cancer? A. Transitional cell carcinoma B. Squamous cell carcinoma C. Adenocarcinoma D. Clear cell carcinoma E. Any of the above

399 C. Schistosoma infection --> Squamous cell CA. Smoking - -> Transitional cell CA

A 45 year old woman presents with recurrent infections and on PE was found to have marked splenomegaly. Her leukocyte count is increased to 300,000. The differential count reveals the presence of myeloblasts and promyelocytes, with predominance of myelocytes, metamyelocytes, bands and segmented neutrophils. Basophils are also increased in number. The patient is not anemic. Leukocyte alkaline phosphatase is decreased. Which of the following describes a major characteristic of this disorder? A. 9:22 translocation B. Expansion of mature B lymphocytes within multiple lymph nodes C. Hypogammaglobulinemia D. Neoplastic cells exhibiting hair-like filamentous projections E. Peak incidence occurs at 65 years

4 A. This is a case of CML. It is associated with 9:22 translocation and may be treated with imatinib mesylate. Basophilia is a rare finding. It is strongly indicative of CML. There are a couple of conditions that cause increased WBC, either a leukemia or an infection. The increase in WBC count due to an infection is called a leukemoid reaction. This can be differentiated from CML via the leukocyte alkaline phosphatase test. In a leukemoid reaction LAP is increased. Source: Topnotch handout on Pathology.

A 70-year-old man presents with severe bone pain and frequent respiratory infections. Serum protein electrophoresis demonstrates an M protein spike in the gamma region. Radiographs of the skull, long bones, and spine demonstrate multiple “punched-out†lesions, and bone marrow aspiration demonstrates large numbers of neoplastic plasma cells. Which of the following statements is true of this disorder? A. Although this patient presents at 70 years of age, the average age of presentation is 50 years of age. B. Renal insufficiency is a common cause of death. C. The M spike is most often an IgM. D. The M spike is most often polyclonal in nature. E. This disorder is the most common T-cell neoplasm.

40 B. The diagnosis is multiple (plasma cell) myeloma, a neoplastic proliferation of malignant plasma cells (mature B cells, not T cells). Death is often caused by renal insufficiency caused by myeloma kidney. The average age of presentation is approximately 70 years of age. IgM myeloma is very uncommon. Both the neoplastic cells and the serum protein spike are monoclonal rather than polyclonal, and the monoclonal spike protein is most frequently an IgG or an IgA.

Which of the following conditions is characterized by findings of giant rugal hypertrophy, thickened gastric mucosa, excessive mucus production, hypoproteinemia and hypochlorydia? A. Non-tropical sprue B. Eosinophilic gastritis C. Crohn gastritis D. Menetrier disease E. Zollinger-Ellison syndrome

400 D.

A patient with chronic back pain came to you due to oliguria and had been abusing NSAIDs chronically for the past years, if renal biopsy will be made, which of the following would be seen? A. Acute Tubular Necrosis B. Papillary Necrosis C. Effaced foot processes D. Coagulative Necrosis of the nephrons E. Nodular Sclerosis

401 B. Chronic NSAID use presents with Papillary necrosis on Renal biopsy. SIMILAR TO PREVIOUS BOARD EXAM CONCEPT/PRINCIPLE

A 3 year-old patient was brought to you by his mother due to an abdominal mass which she palpated while bathing the child with a urine described as "iced-tea-like", upon your thorough PE you noticed that the mass does not cross the midline and involves a synchronous tumor. Your primary consideration would most likely reveal which of the following biopsy findings? A. Varying amounts of immature neuroepithelium, cartilage, bone, muslce and others B. Flexner-Wintersteiner Rosettes C. Homer-Wright Pseudorosettes D. Blastemal, stromal and epithelial cell combinations E. glomerulus-like structures composed of a central blood vessel enveloped by germ cells

402 D. This is a case of Wilm's Tumor, it is the most common renal tumor in childhood, it does not cross the midline and mostly involves bilateral kidneys, biopsy will present with Triphasic combinations of blastema;, stromal and epithelial cell combinations.

An elderly patient with a pulmonary mass and hypercalcemia underwent right pulmonectomy, upon biopsy the pathologist noted the mass to have grown from the alveolar duct and alveolar sacs, what would be the primary consideration? A. Small Cell Lung CA B. Large Cell Lung CA C. Squamous Cell Lung CA D. Lung Adenocarcinoma E. Metastatic Lung CA

403 C. Squamous Cell CA will present with paraneoplastic hypercalcemia and is usually central in location. SIMILAR TO PREVIOUS BOARD EXAM CONCEPT/PRINCIPLE

A patient with Small Cell Lung cancer underwent PET scan, a metastasis was noted, which of the following sites would be the most common site of metastasis? A. Bone (Ribs) B. Liver C. Brain D. Adrenals E. Heart

404 D. Lung cancers may metastasize using lymphatic or hematogenous routes to the Adrenals (50%), Liver (30-50%), Brain (20%), Bone (20%)

A 45-year old male patient with recurrent respiratory infections came to your office with a peripheral smear result with noted Sea-blue histiocytes by the pathologist, what would be your primary consideration? A. AML B. ALL C. CLL D. CML E. Hairy Cell Leukemia

405 D. SIMILAR TO PREVIOUS BOARD EXAM CONCEPT/PRINCIPLE

A walk-in patient with multiple purpuric rashes came to your clinic asking for your interpretation of his self-requested lab results: Platelet Count 245,000/L and INR of 1.00 Bleeding Time was 15 mins, PTT was prolonged. What would be your impression? A. TTP B. Bernard-Soulier Syndrome C. Glanzmann's Thrombasthenia D. von Willebrand Disease E. Hemophilia

406 D. vWD will present with a prolonged bleeding time due to inadequate adhesion of platelets to the severed epithelium and a prolonged PTT due to the lack of Factor 8 from vWF.

A 78 year old male patient with a parotid mass and a chronic smoking history underwent parotidectomy with findings of follicular germinal centers and cystic spaces on biopsy, but because of your thirst for knowledge you read about it and you knew that this tumor is virtually restricted to the parotid gland only and is the second most common salivary neoplasm. What is this tumor? A. Pleomprphic Adenoma B. Adenoid Cystic Adenoma C. Papillary Cystadenoma Lymphomatosum D. Mucoepidermoid adenoma E. Parotid Metastasis from a Lung CA

407 C. Papillary Cystadenoma Lymphomatosum or Warthin's tumor is the second most common tumor of the parotid gland, occurs virtually in the parotid gland only and is classically seen in elderly patients with a chronic smoking history.

Seborrheic Keratoses in a patient with Gastric Adenocarcinoma is termed as what sign? A. Breslow's Sign B. Lesser-Trelat Sign C. Charcot-Marie-Tooth Sign D. Ormond Sign E. Zellballen Sign

408 B.

A 34 year old man diagnosed as having type II diabetes mellitus. Laboratory evaluation of his serum also finds hypertriglyceridemia, which is due to his diabetes. The most common type of secondary hyperlipidemia associated with diabetes mellitus is characterized by elevated serum levels of which one of the following substances? A. Chylomicrons B. High-density lipoproteins C. Intermediate-density lipoproteins D. Low-density lipoproteins E. Very-low-density lipoproteins

409 E. The reason for this is that with decreased levels of insulin with diabetes mellitus there is increased mobilization of free fatty acids from adipose tissue (increased lipolysis). This increases delivery of free fatty acids to the liver, which increases production and secretion of VLDL by the liver.

Spongiotic dermatitis is the histologic pattern for which of the following dermatoses? A. Discoid lupus erythematosus B. Contact dermatitis C. Psoriasis D. Erythema nodosum E. Erythema multiforme

41 B. Contact dermatitis is a type of spongiotic dermatitis with edema and perivascular lymphocytic infiltrate.

Which of the following cytokines is secreted by macrophages and functions as a major mediator of acute inflammation by stimulating acute phase reactions with increasing vascular permeability and stimulating fibroblasts? A. Interleukin-1 B. Interleukin-2 C. Interleukin-3 D. Interleukin-4 E. Interleukin-5

410 A. IL1 and TNF-alpha are secreted by Macrophages, IL2 Tcells, IL3 BM stem cells, IL4 stimulates IgE, IL5 stimulates IgA

You encountered a 35-year old patient with necrotizing granulomas along the respiratory tract and the lungs. He eventually died of crescentic glomerulonephritis. Which of the following markers would turn out positive in this patient? A. c-ANCA B. p-ANCA C. Anti-centromere D. ANA E. Anti-Smith

411 A. This is a classic case of Wegener's Granulomatosis, a small vessel vasculitis, positive for c-ANCA.

Trinucleotide repeat Mutations are amplifications of a sequence of three nucleotides, they also are associated with an Anticipation-type of inheritance, An example is the second most common cause of Mental retardation with a prominent PE finding of Macroorchidism. What Nucleotide repeat will be seen in this patient? A. CAG B. GAA C. CTG D. CGG E. GCA

412 D. Fragile X Syndrome is the 2nd most common cause of mental retardation next to Down's Syndrom, classic finding is macroorchidism

A 1 year old patient was brought to you by his mother due to noticeable developmental delay, you noticed that the patient has specific facies like flat facial profile, epicanthal folds, macroglossia and a prominent Simian Crease. What is the most common chromosomal structural abnormality seen in this type of congenital disorder. A. Nondisjunction B. Mosaicism C. Deletion D. Translocation E. Transversion

413 A. This is a case of Down Syndrome/ Trisomy 21, 95% of cases are due to nondisjunction of chromosomes on cell division causing 3 chromosome 21.

A patient came to your office with a chief complaint of erythema and intense pruritus on both hands after wearing Latex gloves. What type of Hypersensitivity reaction is this patient having? A. Anaphylactic B. Cytotoxic C. Immune Complex D. Delayed E. None of the Above

414 D. Contact Dermatitis is a Type IV/Delayed Hypersensitivity Reaction

An AIDS patient was diagnosed to have Pneumocystis jiroveci Pneumonia, What is probably the patient's CD4 count? A. <500 B. <300 C. <200 D. <100 E. <50

415 C.

What is the most common type of congenital heart defect seen in a patient with Down Syndrome? A. Total Anomalous Pulmonary Venous Return B. Endocardial Cushion Defect C. Transposition of Great Arteries D. Atrial Septal Defect, Secundum Type E. None of the above

416 B. Endocardial Cushion defect and ASD primum type are the most common CHD in Down Syndrome patients.

A 50 year old patient died of Myocardial Infarction, upon Autopsy/Histologic studies of his cardiac muscles there was noted preservation of architecture of dead tissues, noted as Ghost cells. If the MI occurred at 1:00 PM what is the earliest possible time of the autopsy? A. 1:30 PM B. 2:00 PM C. 3:30 PM D. 5:00 PM E. 6:30 PM

417 D. the histologic description is that of a Coagulative type of necrosis, In the cardiac muscles, this is usually evident as early as 4 hours (4-12 hours) after the onset of MI.

Dilated Cardiomyopathy maybe due to the following, EXCEPT: A. Pregnancy B. Friedrich's Ataxia C. Chaga's Disease D. Doxorubicin E. Coxsackie B Myocarditis

418 B. Friedrich's Ataxia may cause a restrictive type of cardiomyopathy

An autopsy of a patient who died of Libman- Sacks Endocarditis will have lesions on the heart grossly described as which of the following vegetation descriptions: A. Small or medium sized vegetations on both sides of the valve leaflets B. Small bland vegetations, usually attached at the line of closure C. Large irregular masses on the valve cusps that extend into the chordae D. small warty vegetations along the lines of closure of the valve leaflets E. Large, multiple, regular masses on the valve cusps that extend into chordae

419 A. LSE is seen in SLE. B:NBTE, C:IE, D:RHD

Oral lesions may be seen secondary to fungal pulmonary infections which spread by hematogenously or by direct inoculation of infected sputum. Which of the following maybe used to stain these fungal elements A. Gomori methenamine stain B. PAS C. Gram's stain D. Both A and B E. AOTA

42 D. Special histochemical stains (Grocott-Gomori methenamine silver (GMS), or periodic acid-Schiff (PAS) highlight fungi in sections.

A patient working for an Aircraft Industry was diagnosed to have Lung Carcinoma, what substance may be responsible for this patient's morbidity? A. Silicon B. Cadmium C. Arsenic D. Vinyl Chloride E. Beryllium

420 E. SIMILAR TO PREVIOUS BOARD EXAM CONCEPT/PRINCIPLE

A 52-year old female came for consult due to on and off fever, easy bruising and easy fatigability. Significant in P.E. were pallor, wasting and presence of hepatosplenomegaly. 3 CBC showed WBC count of >100,000/mm and thrombocytopenia; sea-blue histiocytes were also noted in her bone marrow. The most likely diagnosis for this patient is: A. Acute promyelocytic leukemia B. Chronic myelogenous leukemia C. Histiocytosis X D. Non-Hodgkin lymphoma

421 B. Here we have the typical findings in leukemia - recurrent infection/fever (defective immune cells), pallor and other signs of anemia, and bruising/bleeding (thrombocytopenia). Sea-blue histiocytes (clincher) are found in CML and in other myelodysplastic syndromes (MDS). These cells have wrinkled, green blue cytoplasm hence the name.

Which segment of the respiratory tree is primarily affected in centriacinar emphysema (smoking-related)? A. Terminal bronchioles B. Respiratory bronchioles C. Alveolar ducts D. Alveolar sacs

422 B. In centriacinar/centrilobular emphysema, the central or prximal parts of the acini are affected, whereas the distal parts are spared. These are more common and usually more severe in the upper lobes, particularly in the apical segments. In contrast, panacinar emphysema involves segments from the respiratory bronchiole to the terminal blind alveoli. It tends to occur more commonly in the lower zones and most severe at the bases.

A 49-year old hypertensive male is noted to be refractory to most oral medications save for ACE inhibitors. Upon examination, a bruit is heard on auscultation of his kidneys. Elevated plasma renin was also determined. If we are to biopsy the kidneys of this patient we are most likely to identify: A. Occlusion of renal artery by an atheromatous plaque B. Fibromuscular dysplasia of renal artery C. Adventitial hyperplasia of renal artery D. Endotheliolysis and vasculogenesis

425 A. This is a case of renal artery stenosis and occlusion by an atheromatous plaque is the most common cause (70%, Robbins). This pathology is most common in elderly males. Although fibromuscular dysplasia can also cause renal artery stenosis, we find it less appropriate in this case because it is frequent in younger females. Again, the rule is to choose the best answer. =)

Nodular hyperplasia of the prostate, a common disorder of men above 50, arises most frequently and almost exclusively in which part of the gland? A. Peripheral zone B. Posterior aspect C. Transition zone D. Any portion of the prostate

426 C. And hence the obstructive symptoms of BPH. Prostatic tumors, on the other hand, tend to originate from the periphery.

This is a rare clinical syndrome with major features of adenomatous colonic polyposis and CNS tumors, including medulloblastomas and gliobastomas: A. Gardner syndrome B. HNPCC syndrome C. Crohn syndrome D. Turcot syndrome

427 D.

A 55-year old chronic alcoholic male exhibiting ascites, splenomegaly, periumbilical caput medusae and skin spider angiomata was admitted due to anorexia and severe malnutrition. Hepatic pathology was immediately suspected and a liver biopsy was eventually done over the course. Which of the following may not be true about the patient's condition? A. The biopsy will most likely show bridging fibrous septae and parenchymal nodules in the liver. B. There is reversibility upon correction of the malnutritive state. C. It is the predominant cause of intrahepatic portal hypertension. D. None of these. E. All of these.

428 B. This is a case of liver cirrhosis. It has 3 defining characteristics: 1. bridging fibrous septae; 2. parenchymal nodules; and 3. disruption of the architecture of the entire liver. The vascular reorganization and disruption of the normal structure leads to portal hypertension (intrahepatic). Finally, it is said that once cirrhosis has developed, reversal is already rare; correction of malnutrition may improve the patient's clinical presentation but will not cure definitively the liver problem.

What is the most common clinically significant congenital anomaly of the pancreas? A. Annular pancreas B. Pancreatic agenesis C. Pancreas divisum D. Ectopic pancreas

429 C.

A 72 y/o patient with diabetes complains of severe pain and bloody discharge of the right ear. Which of the following would a Gram stain of material from surgical debridement be expected to show? A. Gram-positive cocci B. Gram-negative cocci C. Gram-positive bacilli D. Gram-negative bacilli E. Septate, branching hyphae

43 D. Malignant otitis externa is most commonly associated with Pseudomonas aeruginosa, which is a gram- negative bacillus.

What consitute the membrane attack complex of the complement system? A. C1q,r,s B. C1q, C3b C. C3b, C4a, C5b D. C5b, C6-9

430 D.

All of the following are immune-mediated hypersensitivity (type III) except for: A. Type I DM B. SLE C. PSGN D. Serum sickness

431 A. Master the 4 hypersensitivity class and their respective examples. (high yield)

A 42-year old female came in for consult for presence of a slowly enlarging non-tender nodule in her anterior neck. Surgical removal was done and studies revealed a thyroid tumor with light-tan appearance containing small foci of hemorrhage; (+) fairly uniform cells forming small follicles containing colloid with some Hurthle cells but no psammoma bodies. Should it progresses, one can assume that this would most likely metastasize thru: A. Direct seeding B. Lymphatic spread C. Hematogenous dissemination D. Metastasis not possible

432 C. This is a case of follicular thyroid carcinoma. It has little tendency to invade the lymphatics, but vascular invasion is common, with spread to bone, lungs, liver and elsewhere. Buzz words: papillary thyroid CA - orphan Annie eye, psammoma; follicular thyroid CA - vascular invasion, Hurthle cell; medullary thyroid CA - amyloid deposits, C-cell hyperplasia.

What is the most accepted theory about the pathogenesis of vitiligo? A. Post-infectious B. Poor nutrition C. Autoimmunity D. Chemical toxicity

433 C.

Which of the following statements is most consistent with brain abscess? A. It is a discrete, non-encapsulated lesion with central liquefactive necrosis. B. It usually originates from adjacent structures to the brain via the lymphatics. C. CSF is under increased pressure, with elevated WBC and protein levels but normal sugar. D. The most favored site is the cerebellum. E. All of the above.

434 C.

What clotting factor is lacking in Hemophilia A? A. Factor VII B. Factor VIII C. Factor IX D. Factor XIII E

435 B. Hemophilia A - factor VIII; Hemophilia B - factor IX/Christmas factor; Hemophilia C - factor XI. All will have increased/prolonged PTT.

A potent vasodilator released during inflammation which reduces platelet aggregation and adhesion, inhibits several features of mast cell-induced inflammation and serves as an endogenous regulator of leukocyte recruitment. It is released from endothelial cells, hence also known as endothelial-derived relaxing factor: A. Leukotrienes B. Bradykinin C. Nitric oxide D. Nitrous oxide E. Prostaglandin

436 C. NO = nitric oxide/EDRF; N2O = nitrous oxide/laughing gas, your inhalational anesthetic.

Tay-Sachs disease is a lysosomal storage disease predominated with affectation of neurons in the brain and the retina. The enzyme deficient is hexosaminidase alpha subunit and the major accumulating metabolite is: A. Glycogen B. Glucocerebroside C. Dermatan sulfate D. GM2 ganglioside

437 D. Disease: enzyme deficient: major accumulating metabolite --- Tay-Sachs: hexosaminidase alpha: GM2 ganglioside --- Gaucher: glucocerebrosidase: glucocerebroside --- Neimann-Pick: sphingomyelinase: sphingomyelin --- Hurler: alpha-L- iduronidase: dermatan sulfate, heparan sulfate --- Hunter: L-iduronosulfate sulfatase: dermatan sulfate, heparan sulfate

The carcinogenic agent that is mainly found in missile fuel and space vehicles: A. Beryllium B. Radon C. Chromium D. Benzene

438 A.

This virus causes a benign, self-limited lymphoproliferative disorder with absolute lymphocytosis (atypical lymphocytes); associated with hairy leukoplakia and a number of neoplasms including lymphomas and nasopharyngeal carcinoma: A. CMV B. EBV C. Adenovirus D. HTLV-1

439B.

This refers to the ratio of thickness of the mucous gland layer in the airways to that of the wall between the epithelial lining and the cartilage. It is utilized to assess chronic bronchitis: A. Reid index B. De Ritis ratio C. Gleason score D. Air passage ratio

440 A.

Hemolysis and vasoocclusive crisis are common in this form of anemia caused by mutation of glutamic acid to valine at amino acid 6 of beta chain A. Hemoglobin C B. Thalassemia C. Hereditary spherocytosis D. Sickle cell anemia E. G6PD Deficiency

441 D. Sickle-cell anaemia is caused by a point mutation in the β-globin chain of haemoglobin, causing the hydrophilic amino acid glutamic acid to be replaced with the hydrophobic amino acid valine at the sixth position.

A 58 year-old male, hypertensive was rushed to the Emergency Room due to loss of consciousness. His son recalled that his father suddenly complained of very severe headache after their breakfast followed by loss of consciousness. What is the most probable diagnosis? A. Intraparenchymal Hemorrhage B. Epidural Hemorrhage C. Subarachnoid Hemorrhage D. Subdural Hemorrhage E. Interventricular Hemorrhage

442 C. A subarachnoid hemorrhage is a bleeding into the subarachnoid space, the area between the arachnoid membrane and the pia mater surrounding the brain. This may occur spontaneously, usually from a ruptured cerebral aneurysm, or may result from head injury. Symtpoms of SAH include a severe headache with a rapid onset (thunderclap headache), vomiting, confusion or a lowered level of consciousness, and sometimes seizures.

Which of the following malignant neoplasms is correctly paired with the appropriate paraneoplastic manifestation? A. Squamous cell carcinoma - hypercalcemia B. Glioblastoma multiforme - hyperglycemia C. Adrenal cortical carcinoma - hyperuricemia D. Hepatocellular carcinoma - polycythemia E. Gastric carcinoma - achlorydia

443 A. Paraneoplastic syndromes are common in lung cancer, and may be the first manifestation of the disease or its recurrence. Hypercalcemia is frequently found in patients with squamous cell carcinoma. It may arise from bone metastasis but can also be induced in a paraneoplastic manner by secretion of parathyroid hormone-related protein (PTHrP).

The sequence of cellular events in inflammation is: A. Diapedesis-margination-chemotaxis- phagocytosis B. Margination-diapedesis-chemotaxis- phagocytosis C. Diapedesis-chemotaxis-margination- phagocytosis D. Margination-chemotaxis-diapedesis- phagocytosis E. Chemotaxis-margination-diapedesis- phagocytosis

444 B. The sequence of cellular events in inflammation is: Margination-diapedesis-chemotaxis-phagocytosis

A 22 year-old man has lifelong hemorrhagic diathesis. The PT and bleeding time are normal, but the aPTT is prolonged. The most likely cause of the bleeding disorder is: A. Von Willebrand's Disease B. Hemophilia C. Vitamin K Deficiency D. Factor VII Deficiency E. Bernard-Soulier Syndrome

448 B. Hemophilia is a group of hereditary genetic disorders that impair the body's ability to control blood clotting or coagulation. In Hemophilia, platelet count, bleeding time and prothrombin time are all normal. Only the aPTT is prolonged such as in Hemophilia A and B.

The histologic hallmark of chronic bronchitis: A. Inflammation and fibrosis B. Hypertophy of goblet cells C. Hyperplasia of mucosal glands D. Marked narrowing of bronchial lumen E. Reversible bronchoconstriction

449 C. Chronic bronchitis is defined clinically as persistent cough and sputum production for at least 3 months in at least 2 consecutive years. Although the number of goblet Cells inCrease slightly, the major histologiC Change is in the size of the muCous gland (hyperplasia). This increase is assessed by the ratio of the thickness of mucous gland layer to the thickness of the wall between the epithelium and the cartilage, the Reid Index.

The most important prognostic feature in invasive breast cancer is: A. Grade of tumor B. Histologic type of tumor C. Size of tumor D. Status of sentinel lymph nodes E. Metastasis

451 D. Prognostic information is important in counseling patients about the likely outcome of their disease and choosing appropriate treatment. Axillary lymph node status is the most important prognostiC faCtor for invasive CarCinoma in the absenCe of distant metastasis. The size of an invasive CarCinoma is the seCond most important prognostiC faCtor.

An 8 year-old female can bend her thumb back to touch her forearm. She can pull her skin out from her abdomen about 10cm and a cut to her skin gapes open with difficulty in repairing. Inherited defect is suspected causing the above findings which is defect in: A. LDL receptor B. factor VIII C. dystrophin D. alpha-1 antitrypsin E. collagen

452 E. Inherited defects in the synthesis or structure of fibrillar collagen results to Ehler's Danlos syndrome. Skin of patients with this defect is extraordinarily stretchable, extremely fragile and vulnerable to trauma. Affected individual also has joint hypermobility, joint laxity, dislocations or easy bruising.

Honesto is a 6 year-old boy who had fever associated with vesicular lesions in different ages all over his body. He was given aspirin and subsequently developed fatty change of the liver. The most likely diagnosis is: A. Viral Hepatitis B. Varicella-Zoster Infection C. Reye's Syndrome D. Subactue sclerosing panecephalitis E. Impetigo

453 C. Reye's syndrome is a rare but serious condition that causes swelling in the liver and brain. Reye's syndrome most often affects children and teenagers recovering from a viral infection (Varicella and Flu) who has been given Aspirin.

A 45 year-old male office worker has keratoconjunctivitis. He has oral mucosal atrophy with buccal mucosal ulceration. A biopsy of minor salivary glands revealed plasma cell infiltrates and biopsy of his lips revealed marked lymphocytic infiltrates. The antibody that is most likely found to this patient is: A. anti-dsDNA B. anti-centromere C. anti-SS-A D. anti-Scl-70 E. anti-RNP

454 C. Sjogren's syndrome is is a chronic autoimmune disease which destroys the exocrine glands, specifically the salivary and lacrimal glands which leads to the development of xerostomia and keratoconjunctivitis sicca, which takes place in association with lymphocytic infiltration of the glands. The antibody that is most likely found in this patient is anti-SS-A.

It refers to a special form of necrosis usually seen in immune reactions involving blood vessels due to deposits of fibrin complexes, together with fibrin that has leaked out of vessels resulting in a bright pink amorphous appearance in H&E stains: A. Fat necrosis B. Fibrinous necrosis C. Coagulative necrosis D. Liquefactive necrosis E. Gangrenous necrosis

456 B. Fibrinous pericarditis is a special form of necrosis usually seen in immune reactions involving blood vessels due to deposits of fibrin complexes, together with fibrin that has leaked out of vessels resulting in a bright pink amorphous appearance in H&E stains, called "fibrinoid" (fibrin-like) lesions.

A 42 year-old female was diagnosed with follicular adenoma. The characteristics of the diagnosis are enumerated below, except: A. Usually solitary and spherical B. Papillary structures within the lesion C. Fibrous encapsulation D. Contain multiple nodules on cut surface E. Morphology within and outside the capsule are different

457 B. Typical thyroid adenoma is a solitary, spherical, well- encapsulated lesion that is well-demarcated from the surrounding thyroid parenchyma. Follicular adenomas contain multiple nodules on their cut surface. Papillary formation is not a typical feature of follicular adenoma.

In viral hepatitis infection, the folllowing is considered the histologic marker of irreversible liver injury: A. Piecemeal necrosis B. Bridging necrosis C. Bridging fibrosis D. Interface hepatitis E. Ductular reaction

459 C. Bridging fibrosis is the deposition of fibrous tissue with linking of fibrous septa which is the hallmark of chronic hepatitis.

Paradoxical embolus is usually seen among patients with: A. Tetralogy of Fallot B. Transposition of great vessels C. Cardiac septal defect D. Tricuspid atresia E. NOTA

46 C. In patients with cardiac septal defects, an embolus may detour into the left heart as a paradoxical embolus to organs and the brain.

A bone marrow aspirate was obtained in a 48 year-old male, nosmoker who complained of easy fatigability, weakness, weight loss and anorexia associated with LUQ abdominal pain. Laboratory revealed anemia. The bone marrow aspirate revealed numerous scattered macrophage with abundant wrinkled green- blue cytoplasm called "sea-blue histiocytes". The most probable diagnosis is: A. Chronic myelogenous leukemia B. Multiple myeloma C. Hodgkin's lymphoma D. Burkitt's lymphoma E. Adult T-cell lymphoma

460 A. CML is a disease primary of adults between ages 25 and 60 years. Patient presents with mild to moderate anemia, easy fatigability, weakness, weight loss, anorexia, abdominal pain due to splenomegaly which may lead to splenic infarction. The bone marrow aspirate of CMP shows numerous scattered macrophage with abundant wrinkled green-blue cytoplasm called "sea-blue histiocytes".

A 7 year old boy presents with pallor and recurrent fever. CBC showed increased WBC and decreased RBC. Bone marrow analysis showed hypercellularity with predominance of lymphoblasts, MPO negative, PAS positive, TDT and CALLA positive. What is not true of this disease? A. it may spread to CNS and testes B. least responsive to chemotherapy C. the primary drud used for this is asparaginase D. it is composed of immature B or T cells E. all of the choices are correct

461 B. this is a case of acute lymphoblastic leukemia, the most common cancer of children. It has hypercellular marrow packed with lymphoblasts - immature T or B cells. It is MPO-negative, PAS positive, TDT and CALLA positive. It may spread to the CNS and testes. It is the most responsive to chemotherapy, and the primary drug used is asparaginase..

A 35 year old male complained of bone pain and recurrent fever. His Hemoglobin was low and his creatinine is moderately high. On imaging, you see lytic bone lesions with punch out defects. What is your primary consideration? A. chronic kidney disease B. multiple myeloma C. waldenstrom's macroglobulinemia D. monoclonal gammopathy of undetermined significance E. none of the choices

462 B. The primary consideration is multiple myeloma presenting with bone pain, recurrent infection, anemia, and renal insufficiency. Lytic bone lesions and punched-out defects are characteristics of multiple myeloma, which are not found in waldenstroms macroglobulinemia. MGUS is asymptomatic. CKD will not present with bone pain and lytic bone lesions.

In chronic myeloproliferative disorders, one of the following has no JAK2 mutation. A. Polycythemia vera B. Essential thrombocytosis C. Primary myelofibrosis D. Chronic myelogenous leukemia E. No exception

463 D. CML has no JAK2 mutations.

What is the major red cell hemoglobin present in Cooley's anemia? A. Hemoglobin F B. Hemoglobin A C. Hemoglobin H D. Hemoglobin Barts E. A and B

464 A. In Cooley's anemia or Beta-thalassemia major, the major red cell hemoglobin is Hemoglobin F. Hemoglobin A is absent. Hemoglobin H is present in hemoglobin H disease, while hemoglobin Barts is present in hydrops fetalis.

What is the most common inherited bleeding disorder? A. Hemophilia A B. Hemophilia B C. Von Willebrand disease D. Bernard-Soulier syndrome E. Glanzmann's thrombasthenia

465 C.

A 65 year old male smoker had a mass in the mandibular area. Biopsy of the mass revealed an epithelial and lymphoid elements, with cystic spaces and follicular germinal centers. What is the diagnosis? A. Pleomorphic adenoma B. paraganglioma C. mucocoele D. warthin tumor E. mucoepidermoid carcinoma

466 D. Warthin tumor or papillary cystadenoma lymphomatosum arises almost exclusively in the parotid gland, and is associated with smoking. It is composed of epithelial and lymphoid elements with cystic spaces and follicular germinal centers.

A 5 year old girl with atopic dermatitis constantly scratching her elbow and arm producing a thickened rough skin with prominent markings. What is the skin lesion? A. plaque B. lichenification C. excoriation D. scale E. Wheal

467 B. lichenification - thick rough skin with prominent markings; excoriation- linear, trauma, epidermal breakage; plaque - elevated, flat lesion, larger than 5mm; scale- dry, platelike, from conification; wheal - pruritic, erythematous with dermal edema.

What is the most common form of cancer in men? A. Testicular cancer B. Lung cancer C. Colon cancer D. Prostate adenocarcinoma E. Gastric cancer

468 D. prostate adenocarcinoma is the most common form of cancer in men. Lung cancer is the leading cause of cancer mortality overall.

A 50 year old female complained of vaginal dryness, stiffening, and constriction. On examination, there is a white parchment like patches of vulvar skin and atrophy of labia. Which one of the following is/are true? A. It can lead to cancer B. there is thinning of the epidermis C. there is hydropic degeneration of basal cells D. there is lymphocytic infiltrate E. all of the choices are correct

469 E. this is a case of lichen sclerosus or chronic atrophic vulvitis. All choices describe lichen sclerosus

A 13 y/o F girl with asthma seeks medical attention at her pediatrician’s office complaining of shortness of breath. What would sputum cytology tests reveal? A. Fungal hyphae B. Acid-fast bacilli C. Keratin pearls D. Charcot-Leyden crystals E. Atypical squamous epithelial cells

47 D. Charcot-Leyden crystals are a sign of eosinophilic degranulation, and Curschmann spirals are a sign of excess mucus in sputum of asthma patients.

Osteopetrosis is the first genetic disease treated with bone marrow transplantation. It is also known as: A. Marble bone disease B. Brittle bone disease C. Osteitis deformans D. Osteitis fibrosa cystica E. None of the choices

470 A. osteopetrosis aka marble bone disease; osteogenesis imperfecta aka brittle bone disease; paget disease aka osteitis deformans; osteitis fibrosa cystica is seen in hyperparathyroidism.

A 30 year old male was riding on his motorcycle, suddenly he had a head on collision with another vehicle. He was thrown 1meter away from his motorcycle hitting his head on the ground. He was brought to the ER unconscious, but later regain his consciousness. On CT scan, there was a lenticular shaped lesion most probably a bleed. What is the source of bleed? A. middle cerebral artery B. brain parenchyma C. middle meningeal artery D. bridging veins E. ventricles

471 C. this is epidural hematoma with lucid interval and lenticular shaped lesion on CT scan. The source of bleed is middle meningeal artery; Subdural hematoma has delayed presentation with fluctuating levels of consciousness, and on CT scan it has crescent shaped lesions. The source of bleed is the tearing of bridging veins.

What is the most common malignant primary brain tumor in adults? A. meningioma B. Cystic cerebellar astrocytoma C. ependymoma D. Glioblastoma multiforme E. Medulloblastoma

473 D.

A 34 year old female complained of polyuria and thirst. You are considering a posterior pituitary syndrome. If this is cause by a transection of the pituitary stalk. What is the treatment? A. thiazides B. indomethacin C. desmopressin D. water restriction E. demeclocycline

474 C. diabetes insipidus will present with polyuria and thirst. There are 2 types: nephrogenic and central DI. In central DI, there is lack of ADH due to transection of pituitary stalk, hypothalamic disease or posterior pituitary disease. The treatment for central DI is desmopressin. Indomethacin and thiazides are used for nephrogenic DI. Water restriction is indicated in SIADH. demeclocycline can cause nephrogenic DI.

A 45 year old female sought consultation to a surgeon due to a mass on the upper outer quadrant of her Right breast. The surgeon decided to have biopsy of the mass which revealed a dyscohesive infiltrating tumor cells arranged in single file or in sheets. A. invasive lobular carcinoma B. invasive ductal carcinoma C. inflammatory breast CA D. phyllodes tumor E. fibroadenoma

475 A. The following are the morphologic feature of each cancer type: invasive lobular CA - signet ring cells arranged in Indian file pattern, or dyscohesive infiltrating tumor cells arranged in single file or in loose clusters or sheets; invasive ductal CA- has characteristic grating sound due to small, central pinpoint foci or streaks of chalky-white elastotic stroma; phyllodes tumor - is a lobulated tumor with cystic spaces; inflammatory breast CA - tumor emboli in dermal lymphatics.

One of the following embryologic remnant is not correctly paired. A. Umbilical arteries : medial umbilical ligaments B. Foramen ovale : fossa ovalis C. Ductus venosus : ligamentum venosum D. Right umbilical vein : ligamentum teres hepatis E. ductus arteriosus : ligamentum arteriosum

476 D. Right umbilical vein degenerates and has no remnant. Left umbilical vein becomes the ligamentum teres hepatis

A 40 year old female with anterior neck mass came in for consultation. You palpated a diffusely enlarged thyroid gland without nodularity. The rest of the physical examination is unremarkable. What is your primary consideration? A. Multinodular goiter B. Diffuse nontoxic goiter C. thyroid adenoma D. Graves disease E. none of the choices

477 B. diffuse nontoxic goiter is characterized by diffusely enlarged thyroid gland without nodularity. Multinodular goiter is characterized by multilobulated, assymetrically enlarged thyroid gland. In graves disease, there is also diffuse enlargement of thyroid gland but ophthalmopathy is prominent. in thyroid adenoma, there is a discrete solitary mass

Hemorrhagic or red infarct occurs in tissues with dual circulation such as: A. liver B. lungs C. intestines D. All of the above E. A and B only

478 D.

A 50 year old male came in for an eye examination due to blurring of vision. After a thorough eye examination, he was told by the ophthalmologist that he has cataract. What are the possible systemic diseases causing cataract? A. diabetes mellitus B. wilson disease C. atopic dermatitis D. all of the above E. A and C only

479 D.

This type of gastritis is characterized by auto- antibodies to gastric parietal cells and intrinsic factor. A. Type A B. Type B C. Type AB D. Type C E. Type O

48 A. Type A - Autoimmune gastritis (10%), autoantibodies are made against parietal cells and intrinsic factor, gland atrophy leads to pernicious anemia; Type B - Helicobacter pylori infection (90%), the most common cause of gastritis, Increased risk of peptic ulcers and carcinoma

A 37 year old female has a salmon-colored plaques on her elbows, knees, and scalp with adherent silvery-white scales. Upon scratching the lesion, pinpoint bleeding was noted. Which of the following is true: A. there is dilated, tortuous blood vessels within the papillae B. this is called koebner phenomenon C. this is called spongiform pustules D. this is called munro microabscesses E. bleeding is caused by acanthosis

480 A. The lesion contains suprapapillary plates with dilated, tortuous blood vessels within papillae which leads to pinpoint bleeding when scrathed called the auspitz sign. When formation of the lesion is induced by local trauma, it is called koebner phenomenon.

A 50 year-old man presents with recurrent fever, infections, bleeding tendencies and pallor. On bone marrow biopsy, sea-blue histiocytes were seen. The most likely diagnosis is: A. AML B. CML C. CLL D. ALL

481 B. SIMILAR TO PREVIOUS BOARD EXAM CONCEPT/PRINCIPLE. Sea-blue histiocytes and massive splenomegaly are the buzzwords for CML.

Miliary tuberculosis spreads via which route? A. hematogenous B. lymphogenous C. lymphohematogenous D. Contiguous

482 C. SIMILAR TO PREVIOUS BOARD EXAM CONCEPT/PRINCIPLE.

A patient who suffered a massive stroke years ago would be expected to have: A. Liquefactive necrosis B. Coagulative necrosis C. Fibrinoid necrosis D. Caseous necrosis

483 A. Coagulative necrosis happens in the heart, spleen, kidney, caseous necrosis for TB in the lungs, and fibrinoid necrosis for blood vessels.

The movement of a leukocyte towards the site of injury is called: A. chemotaxis B. diapedesis C. migration D. rolling

484 A. SIMILAR TO PREVIOUS BOARD EXAM CONCEPT/PRINCIPLE. There was a big confusion here but to be strict on the definition: Diapedesis is the travel of the leukocyte between endothelial cells as it exits the blood vessel, migration is the condition where in the leukocyte travels through the interstitium to the site of injury or infection while chemotaxis is the orientation of a cell towards a chemical stimulus.

The diagnosis of endometriosis can be confirmed by the presence of: A. Endometrial stroma B. Endometrial glands C. Both A and B D. Either A or B

485 C. SIMILAR TO PREVIOUS BOARD EXAM CONCEPT/PRINCIPLE.

A patient palpated a mass on her left breast. Which characteristic would point to a possible malignancy? A. The mass is firm and doughy. B. It is movable. C. It is tender on palpation. D. There is a palpable node on the axilla.

486 D. Malignant breast masses are usually hard, fixed and nontender on palpation.

A 40 pack-year smoker presents with dyspnea on exertion. On PE he has a barrel chest and hyperresonant lungs. Based on spirometry and chest xray, you diagnosed him to have emphysema. Which part of the respiratory system is damaged the most? A. respiratory bronchiole B. alveoli C. alveolar duct D. major bronchi

487 A. Centroacinar emphysema (mostly affects respiratory bronchioles) is found in smokers, while panacinar emphysema (respiratory bronchioes, alveolar ducts, alveoli) is more common for those with anti- trypsin/anti-elastase deficiency. SIMILAR TO PREVIOUS BOARD EXAM CONCEPT/PRINCIPLE.

A term neonate is delivered via cesarean section because of cephalopelvic disproportion. The amniotic fluid is clear and the infant cried almost immediately after birth. Minutes after birth however, there was noted increase in respiratory rate and grunting. A chest xray showed fluid in the fissures and prominent pulmonary vascular markings. Which is the most likely diagnosis? A. meconium aspiration B. idiopathic hyaline membrane disease C. pneumonia D. transient tachypnea of the newborn

488 D. This is a classic case describing TTN. The condition is due to retained lung fluid, commonly in term infants delivered by cesarean section.

A 6-day-old premature infant born at 29 weeks AOG presents with gross bloody stools, abdominal distention and autonomic instability. What is the initial diagnostic step? A. Stool culture B. Plain abdominal xray C. Meckel scan D. Barium enema

489 B. Necrotizing enterocolitis is a life-threatening condition seen mostly in premature infants. The characteristic finding on plain radiograph is pneumatosis intestinalis.

A 32 y/o F has progressive dyspnea, cough, and fever. A chest radiograph shows bilateral infiltrates. She also notes dark brown urine. Which of the following statements is FALSE? A. Low levels of serum complement may be seen B. Blood cultures should be performed C. Serum ANCA should be performed D. The differential diagnosis includes systemic lupus erythematosus E. Polyarteritis nodosa is the most likely diagnosis

49 E. Polyarteritis nodosa typically does not involve the pulmonary vessels.

A patient presents with bilateral acoustic schwannoma. Which is the most likely diagnosis? A. Neurofibromatosis 1 B. Neurofibromatosis 2 C. Von Hippel Lindau D. Von Recklinghausen

490 B. NF1 is the same with Von Recklinghausen which presents with cutaneous neurofibromas among others. Von Hippel Lindau also predisposes individuals to tumors and cysts (eg. Pheochromocytoma, and other CNS tumors). SIMILAR TO PREVIOUS BOARD EXAM CONCEPT/PRINCIPLE.

This type of cancer predisposes to a paraneoplastic syndrome wherein antibodies against presynaptic calcium channels are produced: A. Squamous cell lung carcinoma B. Small cell lung carcinoma C. Lung Adenocarcinoma D. Lymphoma of the lung

491 B. The syndrome described is known as Lambert-Eaton syndrome which presents with muscle weakness. Small cell lung carcinoma is notorious for causing this syndrome.

In aortic dissection, the blood accumulates between: A. Tunica intima and tunica media B. The layers of tunica media C. Tunica media and tunica adventitia D. Any of the above

492 B. SIMILAR TO PREVIOUS BOARD EXAM CONCEPT/PRINCIPLE.

The basic pathology of atherosclerosis is: A. Fat deposition on the tunica intima B. Thickening and loss of elasticity of arterial walls C. Endothelial injury D. Deposition of atheromatous plaque causing clogged arteries

493 C. Endothelial dysfunction or injury is the key event in the development of atherosclerosis. SIMILAR TO PREVIOUS BOARD EXAM CONCEPT/PRINCIPLE.

A patient complains of chest heaviness unrelieved by nitrates and rest. Which among the following enzymes is expected to rise first? A. troponins B. CKMB C. myoglobin D. LDH

494 C. Myoglobin rises for the first 2-3 hours (has high sensitivity but poor specificity), CKMB rises 4-6 hours after, Troponins rise 6-12 hours after an MI.

A patient presents with compressive symptoms due to a fixed, hard and painless goiter. Thyroidectomy showed massive amounts of fibrous tissue. Which is the likely diagnosis? A. Papillary carcinoma B. Follicular carcinoma C. Riedel's thyroiditis D. de Quervain's thyroiditis

495 C. The keyphrase fibrous tissue pertains to Reidel's thyroiditis. A differential would be thyroid carcinoma, probably anaplastic type.

A very active 17-year-old boy is noted by the family to have recurrent deep pains in his thigh that awaken him from sleep. The family brings him to your office with a complaint of swelling over his distal thigh which he said is probably due to playing basketball a week ago. A radiograph of the leg showed sunburst appearance. Which would likely explain the condition of the patient? A. osteosarcoma B. ewing's sarcoma C. osteomyelitis D. bone fracture

496 A. A classic description for osteosarcoma is the key phrase "sunburst appearance" (in contrast to Ewing sarcoma which is "onion skin appearance"). Periosteal lifting is also a clue to the diagnosis. Osteosarcoma usually occurs at the metaphysis of the distal femur or proximal tibia.

A young boy presents with gait instability and diplopia. On CT scan, there was a cerebellar tumor which is hyperdense, and noncalcified. Which of the following is the most likely diagnosis? A. cerebellar glioma B. craniopharyngoma C. ependymoma D. medulloblastoma

497 D. SIMILAR TO PREVIOUS BOARD EXAM CONCEPT/PRINCIPLE Craniopharyngoma is a tumor from the Rathke's pouch and ependymoma arises from the lining of the ventricles.

Which is FALSE regarding autosomal dominant polycystic kidney disease (ADPKD)? A. It is due to mutations in PKD1 or PKD2. B. It is associated with congenital hepatic fibrosis. C. It has been noted in patients with mitral valve prolapse and benign hepatic cysts. D. It presents with multiple, large bilateral cysts.

499 B. Congenital hepatic fibrosis is associated with autosomal recessive polycystic kidney disease, previously infantile polycystic kidney disease.

A 50 year old man seeks consult due to a pruritic rash which he has had over the past 8 months. On PE, there were erythematous, eczematoid patches and raised plaques distributed asymmetrically over the chest and abdomen. On biopsy of the lesions, atypical CD4+ T cells with cerebriform nuclei were found. What is a possible outcome in the course of this condition? A. Acute leukemia B. Myelofibrosis C. Sezary Syndrome D. A and B only E. All of the above

5 C. This is a case of Mycosis fungoides, which is a T cell lymphoma of the skin. Atypical CD4+ T cells with cerebriform nuclei are found on biopsy. The disorder may remain confined to the skin for several years. When the neoplastic cells invade the skin and become systemic, this is called Sezary syndrome. Sezary syndrome is the leukemic form of this cutaneous T cell lymphoma and is characterized by the combination of skin lesions and circulating neoplastic cells. Acute leukemia and myelofibrosis are courses in the natural history of the myeloproliferative syndromes. Source: Topnotch handout on Pathology

A patient presents with ptosis and weakness more notable during the end of the day. Tensilon test is positive. Which type of hypersensitivity does this patient exhibit? A. Type 1 B. Type 2 C. Type 3 D. Type 4

500 B. Myasthenia gravis is under type 2 (cytotoxic/antibody-mediated) hypersensitivity. The body produces antibodies against acetylcholine receptors on NMJ.

Which of the following is made up of Type 2 collagen?? A. Cornea and lens B. Vitreous humor and nucleus pulposus C. Basal lamina and bone D. Late wound repair E. None of the above

501 B.

Burton's lines is found in which poisoning? A. Mercury B. Copper C. Lead D. Arsenic E. None of the above

502 B. Burton's lines are lines on the gingiva and on the epiphysis of long bones in lead poisoning.

The following are part of the Duke's Major Criteria for Infective endocarditis, except: A. Positive blood culture indicating charcteristic organism B. New valvular regurgitation C. Echocardiographic finding of valve-related mass or abscess D. All of the above E. None of the above

503 E. Robbins 7th ed., 598

These are small erythematous or hemorrhagic, macular, nontender lesions on the palms and soles and are a consequence of septic embolic events: A. Janeway lesions B. Osler nodes C. Roth spots D. All of the above E. None of the above

504 A. SIMILAR TO PREVIOUS BOARD EXAM CONCEPT/PRINCIPLE:Osler nodes- are small, tender subcutaneous nodules that develop in the pulp of the digitd or occassionally more proximally in the fingers and persist for hours to several days; Roth spots-are oval retinal hemorrhages with pale centers Robbins 7th ed., 598

The following features can be seen in rheumatoid arthritis except: A. Rice bodies B. Pannus formation C. Heberden nodes D. All of the above E. None of the above

505 C. rice bodies are aggregation of organizing fibrin covering portions of the synovium and floating in the joint space; pannus is a mass of synovium and synovial stromaconsisting of inflammatory cells, granulation tissue, and fibroblasts, which grows over the articular cartilage and causes its erosion. Both rice bodies and pannus is found in RA. Heberden nodes in the fingers of patients with osteoarthritis represent prominent osteophytes at the distal interphalangeal joints. 1305

Which of the following is true regarding Sickle cell disease? A. It is caused by a point mutation at the 6th position of the Beta-globin chain leading to the substitution of a valine residue for a glutamic acid residue. B. It is caused by a point mutation at the 9th position of the Beta-globin chain leading to the substitution of a valine residue for a glutamic acid residue. C. It is caused by a point mutation at the 6th position of the Beta-globin chain leading to the substitution of a glutamic acid residue for a valine residue. D. It is caused by a point mutation at the 9th position of the Beta-globin chain leading to the substitution of a glutamic acid residue for a valine residue. E. None of the above

506 A. SIMILAR TO PREVIOUS BOARD EXAM CONCEPT/PRINCIPLE:Robbins 7th ed., 628

This is the most common leukemia of adults and the elderly. Peripheral blood smear shows smudge cells and nucleated red blood cells. What is the condition? A. ALL B. CLL C. AML D. CML E. None of the above

507 B. SIMILAR TO PREVIOUS BOARD EXAM CONCEPT/PRINCIPLE

Which type of Hodkin's lymphoma has the best prognosis? A. Nodular sclerosis B. Mixed cellularity C. Lyphocyte-predominant D. Lymphocyte-rich E. Lymphocyte-depleted

508 D.

The following characteristics refer to Crohn Disease but not to Ulcerative colitis: A. Skip lesions or sharp demarcation of diseased bowel segment from adjacent uninvolved bowel B. Fistula or sinus tract formation C. Absence of granulomas D. Transmural inflammation E. All of the above statements refer to Crohn Disease.

509 C. Noncaseating granulomas-Crohn; No granuloma in Ulcerative colitis

An elderly woman complains of fatigue, anemia, and bright red blood in stool. Which of the following is the MOST LIKELY diagnosis? A. Rectal adenocarcinoma B. Anal squamous cell carcinoma C. Endometriosis D. AOTA E. NOTA

51 A. 10% of colon cancers occur in the rectum. Anal carcinoma is usually human papillomavirusâ€" related and occurs in younger patients; endometriosis causing rectal bleeding would not be expected in a postmenopausal patient.

The following statements are true regarding colorectal cancer, except: A. The most common site of colorectal cancer is in the rectosigmoid area. B. Carcinomas in the distal colon tend to be annular, encircling lesions that produce so- called napkin-ring constrictions of the bowel. C. Left-sided colonic cancers usually presnt with occult bleeding, changes in bowel habit, or crampy left lower quadrant discomfort. D. Cecal or right-sided colonic cancers usually present with fatigue, weakness and iron- deficiency from bleeding. E. None of the above

510 E. Robbins 7th ed., 865-66

Which liver disease is characterized histologically by hepatocyte swelling and necrosis, mallory bodies, neutrophillic reaction and fibrosis? A. Fatty liver B. Alcoholic hepatitis C. Alcoholic cirrhosis D. All of the above E. None of the above

511 B.

Hemosiderin would most likely deposit in which organ in patients with hereditary hemochromatosis? A. Pituitary gland B. Myocardium C. Liver D. Myocardium E. Thyroid gland

512 C. in decreasing order of severity:liver, pancreas, myocardium, pituitary gland, adrenal gland, thyroid and parathyroid glands, joints and skin (detected by Prussian blue histologic reaction) p909

Kayser-Fleischer ring of Wilson's disease is the green to brown deposits of copper in which layer of the corneal limbus? A. Corneal epithelium B. Bowman's layer C. Corneal stroma D. Descemet's membrane E. Corneal endothelium

513 D.

Which of the following conditions refer to Budd-chiari syndrome? A. Obstruction of a single main hepatic vein by thrombosis. B. Obstruction of 2 or more major hepatic veins produces liver enargement, pain and ascites. C. Obliteration of hepatic vein radicles by varying amounts of subendothelial swelling and fine reticulated collagen. D. Sinusoidal dilation with impediment of hepatic blood efflux. E. None of the above

514 B. A. Obstruction of a single main hepatic vein by thrombosis is clinically silent. C. Obliteration of hepatic vein radicles by varying amounts of subendothelial swelling and fine reticulated collagen - veno-occlusive disease.D. Sinusoidal dilation with impediment of hepatic blood efflux - Peliosis hepatica.

These are membrane proteins that recognize a variety of microbe-derived molecules and stimulate innate immune responses againsts the microbes: A. Toll-like receptors B. Cancer antigen C. Hemagglutinin D. Lipopolysaccharide E. Endotoxin

515 A. SIMILAR TO PREVIOUS BOARD EXAM CONCEPT/PRINCIPLE:Robbins 7th ed., 195

These mediators effect increase in vascular permeability and neutrophil recruitment to site of injury: A. Opsonins B. Coagulation factors C. Complement molecules D. Leukotrienes E. None of the above

516 D. SIMILAR TO PREVIOUS BOARD EXAM CONCEPT/PRINCIPLE: p.208

Which of the following karyotypes is associated with the classic pattern of Klinefelter syndrome? A. 45, X B. 46, XY C. 47, XXY D. 47, XYY E. None of the above

517 C. SIMILAR TO PREVIOUS BOARD EXAM CONCEPT/PRINCIPLE: Klinefelter syndrome is best defined as male hypogonadism that occurs when there are 2 or more X chromosomes and 1 or more Y chromosomes. P.179

This disease is characterized by a distinctive heliotrope discoloration of the upper eyelids with periorbital edema that may accompany or precede the onset of muscle disease? A. Inclusion body myositis B. Dermatomyositis C. Myotonic Dystrophy D. Myasthenia gravis E. None of the above

518 B. SIMILAR TO PREVIOUS BOARD EXAM CONCEPT/PRINCIPLE

These are benign tumors of adults, usually attached to the dura, that arise from the meningothelial cell of the arachnoid. A. Primary CNS lymphoma B. Meningioma C. Medulloblastoma D. Ganglion cell tumor E. None of the above

519 B. SIMILAR TO PREVIOUS BOARD EXAM CONCEPT/PRINCIPLE

True of HBsAg: A. First evidence of infection B. Appears in the serum after symptoms C. Co-incide with the elevation of ALT D. AOTA E. Both A and C

52 A. HBsAg (surface antigen) provides the first evidence of infection and appears in the serum before symptoms.

In Tetralogy of Fallot, the heart is often enlarged and may be boot-shaped owing to what? A. Marked right ventricular hypertrophy, particularly of the apical region B. Marked right ventricular hypertrophy, particularly of the base of the heart C. Marked left ventricular hypertrophy, particularly of the apical region D. Marked left ventricular hypertrophy, particularly of the base of the heart E. None of the above

520 A. SIMILAR TO PREVIOUS BOARD EXAM CONCEPT/PRINCIPLE: p. 569

An otherwise healthy 16 year old girl comes to the physician because of a 4 year history of heavy bleeding with menses. She has a history of excessive bleeding after a dental extraction but has never had spontaneous bleeding. Her father has a history of frequent nosebleeds and post-operative bleeding. Her mother, sisters, and brother have no history of bleeding disorders. Examination of the patient shows no abnormalities except for pallor. Laboratory studies show Hb 8, Hct 25%, Reticulocyte count 2%, platelet count 200000, bleeding time 12 min, INR 1, APTT 60 seconds. Pelvic ultrasonography shows no abnormalities. Which of the following is the most likely mechanism of this patient’s excessive bleeding? A) Abnormal structure of von Willebrand factor B) Capillary fragility C) Inadequate production of factor VIII D) Autoimmune platelet destruction

521 A.

A 67 year old man comes to the physician for a follow-up examination. Three years ago, he underwent radical dissection of a T3 N0 M0 epidermoid carcinoma of the floow of his mouth and supraomohyoid dissection of his neck. He currently takes no medications. He is a 120 pack year smoker but stopped 3 years ago. Vital signs are within normal limits. Examination shows well-healed surgical scars. There are no signs of local recurrence. An X-ray of the chest shows a 3 cm mass in the medial upper lobe of the right lung. Which of the following is the most likely cause of these findings? A) Bronchioalveolar carcinoma B) Metastatic carcinoma C) Primary squamos cell carcinoma D) Mesothelioma

522 C.

A previously healthy 57 year old man comes to the physician because of impotence for 1 year. Examination shows bronze-colored skin. His serum ferritin concentration is 4050 ng/mL. This patient is at increased risk for which of the following complications? A) Hepatocellular carcinoma B) Interstitial lung disease C) Progressive pancytopenia D) Renal failure

523 A.

A 42-year-old computer science professor is brought to the physician by her husband, who reports insidious changes in his wife's personality and behavior. He reports that she believes that aliens have been speaking to her and tampering with their heating and air- conditioning systems. He says that she was upset when she turned 40 years old, and her symptoms have developed since that time. She was adopted, and her family history is unknown. Physical examination shows vermicular movements of the tongue and bilateral writhing motions of the upper extremities. Mental status examination shows indifference to her condition and mild to moderate difficulty with memory and calculations. What is the mechanism of the most likely diagnosis? A) Substantia nigra degeneration B) Build up of tau proteins in neurons C) CAG repeats D) Copper accumulation in tissue

524 C.

A 14-year-old boy is brought to the physician by his parents because of a 2-year history of increasing academic problems. His parents say that he has always been hyperactive and distractible, but now his academic performance has deteriorated to the point that he is failing ninth grade. His teachers say that his hyperactivity is disrupting the classroom. He weighs 54 kg (120 lb) and is 152 cm (60 in) tall. Sexual development is Tanner stage 5; examination shows macro-orchidism, which was not shown on previous examinations. He has a high forehead and long, protruding ears. He exhibits poor eye contact during the examination. Psychoeducational testing shows an IQ of 70. Which of the following is the most likely diagnosis? A) Lesch-Nyhan syndromE B) Prader-Willi syndrome C) Fragile X syndrome D) Klinefelter's syndrome

525 C.

A 19-year-old man comes to the physician because of frequent nosebleeds over the past 3 weeks. He has bipolar disorder currently well controlled with lithium carbonate, bupropion, and valproic acid. Physical examination shows no abnormalities except for dried blood in the nares. Mental status examination shows an anxious mood and slight motor restlessness. Serum studies show a lithium carbonate level of 1.3 mEq/L (therapeutic range=0.6â€"1.2), and valproic acid level of 77 μg/mL (therapeutic range=40â€"100). Which of the following is the most appropriate next step in management? A) Measurement of serum aspartate aminotransferase (AST, GOT) activity B) Measurement of serum bupropion level C) Platelet count D) Discontinuation of lithium carbonate therapy

526 C.

An asymptomatic 32-year-old man comes for a routine health maintenance examination. He has a 10-year history of frequent sinus and pulmonary infections. He had an anaphylactic reaction to a blood transfusion following a motor vehicle collision 3 years ago. His temperature is 37 C (98.6 F). Examination shows mild erythema in the posterior pharynx. The lungs are clear to auscultation. A complete blood count and serum protein electrophoresis are within normal limits. Which of the following is the most likely cause of the frequent infections? A) Colonization with Streptococcus pneumoniae B) Common variable immunodeficiency C) HIV infection D) Selective IgA deficiency

527 D. buzz phrase >> anaphylactic reaction to blood transfusion

A 67-year-old man with long-standing signs and symptoms of congestive heart failure is admitted to the hospital because of progressive shortness of breath. Examination shows no other abnormalities. An x-ray film of the chest shows cardiomegaly, cephalization of blood vessels, and a right-sided pleural effusion. Which of the following sets of pleural fluid findings is most likely in this patient? Leukocyte Segmented Protein Glucose count neutrophils Monocytes (g/dL) (mg/dL) (/mm3) (%) (%) A) 2.5 10 10,000 50 50 B) 2.5 90 2000 60 40 C) 3.8 40 30,000 80 20 D) 4.5 60 10,000 20 80

528B.

A 6-year-old boy is brought to the physician by his mother because of progressive visual loss over the past year. Over the past 2 years, he has had deterioration of his hearing, speech, writing, and intellectual performance. His maternal uncle had similar symptoms. Visual acuity is 20/200 bilaterally. Funduscopic examination shows optic atrophy. His hearing is markedly impaired. There is weakness and spasticity of all extremities. Deep tendon reflexes are extremely hyperactive. Babinski's sign is present bilaterally. On mental status examination, he is not oriented to place, year, month, or the names of his siblings. An MRI of the brain shows marked symmetric white matter disease involving all lobes. Diagnostic studies are most likely to show which of the following? A) Abnormally decreased serum cholesterol level B) Acanthocytes on blood smear C) An excess of very long chain fatty acids D) Normal nerve conduction studies

529 C. adrenoleukodystrophy

Hallmark of malignant transformation A. Keratin pearls B. Dedifferentiation C. Lack of differentiation D. Metastasis E. Skip lesions

53 C. Anaplasia or lack of differentiation is the hallmark of malignant transformation.

A 5-year-old boy is brought to the emergency department 30 minutes after he fainted at home after standing up from a sitting position. His symptoms began 3 days ago with diarrhea and vomiting. He has had no urine output for 18 hours. He is alert but quiet. His temperature is 37.5 C (99.5 F), blood pressure is 75/45 mm Hg, pulse is 120/min, and respirations are 28/min. Examination shows dry lips and tenting of the skin. There is no abdominal tenderness. Bowel sounds are hyperactive. The remainder of the examination shows no abnormalities. His capillary refill time is 5 seconds. Intravenous bolus doses of 0.9% saline are administered. Bladder catheterization yields 5 mL of urine. Urinalysis is most likely to show which of the following? A) Oxalate crystals B) Erythrocyte casts C) Hyaline casts D) Leukocyte casts

530 C. hyaline casts can be seen in setting of dehydration

A county health officer investigates an outbreak of illness among persons attending a church picnic. The illness is characterized by the onset of nausea and vomiting 3 to 4 hours after attending the picnic. All affected persons recover without specific therapy. The investigation implicates egg salad as the vehicle of transmission. This episode is consistent with a foodborne outbreak caused by which of the following? A) Clostridium perfringens B) Giardia lamblia C) Salmonella species D) Staphylococcus aureus

531 D.

A 4-year-old girl is brought to the physician because of pallor and jaundice for 2 days. She had previously been well, although she was treated for jaundice with phototherapy for 2 weeks while a newborn. Her mother and two additional maternal relatives underwent splenectomy during childhood for unknown reasons. Examination of the patient shows jaundice. The spleen tip is palpated 4 cm below the left costal margin. Which of the following blood smear findings is most likely to explain this family's condition ? A) Elliptocytes B) Howell-Jolly bodies C) Schistocytes D) Spherocytes

532 D. hereditary spherocytosis

A 57-year-old man comes for a routine follow- up examination. He has a 10-year history of an intermittent facial rash. He has been taking propranolol for 2 months for hypertension. Examination shows several erythematous pustules and papules involving the nose and central face. There are telangiectasias at the base of the papules. Which of the following is the most likely explanation for these findings? A) Acne rosacea B) Acne vulgaris C) Basal cell carcinoma D) Discoid lupus erythematosus

533 A.

A 27-year-old woman comes to the physician because of a 3-week history of fever, night sweats, rash on both legs, nonproductive cough, and pain and swelling in her wrists and knees. She has not had weight loss. Her temperature is 37.7 C (99.8 F), blood pressure is 110/70 mm Hg, pulse is 96/min, and respirations are 14/min. The lungs are clear to auscultation. Cardiac examination shows no abnormalities. There is swelling and warmth over the wrists and knees bilaterally and tender red nodules on the anterior surface of both lower extremities. An x-ray film of the chest shows bilateral hilar fullness. Which of the following is the most likely diagnosis? A) Carcinoma of the lung B) Histoplasmosis C) Tuberculosis D) Sarcoidosis

534 D.

A 49-year-old woman is admitted to the hospital because of renal failure. She has had episodes of flank pain over the past 20 years. She has also had nocturia 2 to 3 times nightly for 10 years. Her blood pressure is 160/100 mm Hg. Examination shows pale mucous membranes. A mass is palpated in the right flank. Which of the following is the most likely diagnosis? A) Horseshoe kidney B) Nephrolithiasis C) Papillary necrosis D) Polycystic kidney disease

535 D.

A 5-year-old female presents with a new onset hematuria and oliguria after 1 week of experiencing sore throat. Other pertinent findings revealed hypertension, periorbital edema and impaired renal function. A renal biopsy most likely would reveal electron-dense deposits in which of the following sites? A) Between basement membrane and endothelial cells of the glomeruli B) Between the basement membrane and epithelial cells of the glomeruli C) Between the basement membrane and epithelial cells of the proximal tubules D) Within the mesangium of the glomeruli

536 B. PSGN >> deposits between basement membrane and epithelial cells of the glomeruli

A 32-year-old female was noted to have a breast mass with axillary lymph node enlargement. Further work ups, revealed that the mass was cancerous. A diagnosis of invasive ductal carcinoma was made. Modified radical mastectomy was done with axillary lymph node dissection. Pathological examination of the axillary lymph nodes will show? A) Follicular Hyperplasia B) Marginal zone B-cell hyperplasia C) Sinus histiocytosis D) Paracortical lymphoid hyperplasia

537 C. Sinus histiocytosis (also called reticular hyperplasia refers to the distention and prominence of the lymphatic sinusoids. This particular form of hyperplasia is prominent in lymph nodes draining cancers such as carcinoma of the breast. (SIMILAR TO PREVIOUS BOARD EXAM CONCEPT/PRINCIPLE)

A 5-year-old male developed a demyelinating neuropathy associated with Campylobacter jejuni. He presents clinically with ascending neuromuscular paralysis and areflexia. He was then treated with plasmapharesis, IV immunoglobulins and corticosteroids. The diagnosis to this case belongs to what type of hypersensitivity? A) Type I Hypersensitivity B) Type II Cytotoxic Hypersensitivity C) Type III Immune Complex Hypersensitivity D) Type IV Cell mediated Hypersensitivity

538 D. Guillan Barre Syndrome >> type IV reaction

The leading causative agent of injection drug use hepatitis: A. Hepatitis A B. Hepatitis B C. Hepatitis C D. Hepatitis D E. Both B and C

54 C. In hepatitis C, chronic disease occurs in 85% of patients. The leading cause is injection drug use. Cirrhosis will develop in 20% of patients within 20 years.

A 32 year old male was admitted due to fatigue, unexplained fever and spontaneous mucosal and cutaneous bleeding lasting for about 2 weeks. Laboratory work ups revelaed anemia, neutropenia and thrombocytopenia. Peripheral blood smear examination revealed a red staining peroxidase-positive structures with abnormal azurophilic granules. What is the possible diagnosis to this case? A) Acute Lymphoblastic Leukemia B) Acute Myelogenous Leukemia C) Chronic Lymphoblastic Leukemia D) Chronic Myelogenous Leukemia

540 B. red-staining peroxidase-positive structure >> auer rods

Which of the following describes a malignant condition? A. A small nodule of well-developed and organized pancreatic substance is found in the submucosa of the small intestine. B. Biopsy of a pulmonary "mass" reveals disorganized but histologically normal cartilage, bronchi and vessels C. A gelatinous mass is seen to fill the peritoneal cavity in a person known to have an "enlarged appendix." D. In chronic gastroesophageal reflux, squamous epithelium in the lower esophagus is replaced by glandular epithelium. E. None of the above

541 C. A (choristoma), B (hamartoma), and D (metaplasia) describe nonmalignant conditions. C refers to pseudomyxoma peritonei; which occurs due to seeding of metastatic cells into the peritoneum from appendiceal carcinomas.

A 67/M, a known case of lung cancer, came in due to easy fatigability and generalized weakness. Patient was noticeably pale. Your primary impression is anemia secondary to chronic disease. In this condition, one expects low levels of the following parameters except: A. Fe saturation B. Total iron binding capacity C. Transferrin D. Ferritin E. Serum iron

542 D. Ferritin is a soluble iron binding storage protein. It is decreased in IDA; but increased in sideroblastic anemia (iron overload disease) and in anemia of chronic disease / ACD (IL-1 and TNF-a induces its release.) Serum iron levels may differentiate IDA/ACD (decreased iron) from iron overload diseases (increased iron.) Total iron binding capacity correlates with transferrin levels. Whether transferrin increases or decreases may be predicted by what happens to ferritin since decreased ferritin stores increase synthesis of transferrin in the liver. Thus high ferritin -> low transferrin / TIBC; and vice versa. Fe saturation represents the percentage of binding sites on transferrin occupied by Fe. It is increased in Fe overload states; and decreased in IDA and ACD.

On physical examination, a cystic 3 x 3 cm breast mass was palpated on a 27 year old woman who just gave birth 6 months ago, and who had exclusively breastfed her child. What is your primary impression? A. Macrocyst B. Galactocele C. Mastitis D. Fibrocystic changes E. Breast abscess

543 B. History of breastfeeding and recent pregnancy suggest that the cystic mass is a galactocele. Without such a history, primary impression would be a macrocyst.

A 66-year old diabetic, nulliparous woman complains of post-menopausal vaginal bleeding. Prior to menopause, which occurred at age 55, she had irregular menses. She denies the use of estrogen replacement therapy. Her examination is significant for obestity and hypertension. The following are risk factors for her condition except: A. Early age at menarche B. Late menopause C. Diabetes D. Smoking E. Obesity

544 D. History of postmenopausal bleeding in patient with known risk factors (obesity, nulliparity) raises suspicion for endometrial cancer. Increased estrogen exposure increases risk. Coffee, smoking, OCP use, physical activity, and use of raloxifene are protective.

A pre-term infant born to an 18-year old primigravid developed abdominal distension and hypotension on the fourth day of life. You noted that the patient passed bloody stools. Abdominal radiographs demonstrate gas within the intestinal wall. Microscopic examination of the diseased bowel segment in this condition would reveal: A. Fat necrosis B. Coagulative necrosis C. Liquefactive necrosis D. Caseous necrosis E. Apoptisis

545 B. Onset of bloody stools, circulatory collapse and abdominal distension is typical of necrotizing enterocolitis. Prematurity is an important risk factor. Microscopically, mucosal or transmural coagulative necrosis is seen.

A 28/M presents with a month-long history of low-grade fever. On PE, an apical systolic murmur is appreciated on auscultation; the Traube's space was obliterated; and hemorrhagic nontender lesions on the palms and soles were noted. Patient has an unremarkable past medical history. Blood culture woud most likely grow which infectious agent: A. Staphylococcus aureus B. Cardiobacterium sp. C. Streptococcus pyogenes D. Haemophilus influenzae E. Streptococcus viridans

546 E. A classic case of infective endocarditis. The progression of symptoms over one month suggests subacute IE; most commonly due to less virulent organisms, most commonly, Streptococcus viridans.

A 50/M came in with a chief complaint of dysphagia, heartburn and regurgitation of sour tasting fluid. He has weekly episodes of retching and vomiting after drinking sprees. On endoscopy the distal 3rd of his esophagus is seen to have tongues of red, velvety mucosa extending upward from the gastroesophageal junction alternating with residual smooth squamous mucosa and interfaces with light brown columnar mucosa distally. 6cm of the esophagus is affected. Upon biopsy of the area, histologic findings include abundant metaplastic goblet cells, atypical mitoses, nuclear hyperchromasia and failure of epithelial cells to mature as they reach the esophageal surface. What is his most likely diagnosis? A. Mallory-Weiss tear B. Reflux esophagitis C. Barrett esophagus D. Adenocarcinoma of the esophagus E. Squamous cell carcinoma of the esophagus

547 C.

A 7-year old boy presents to the ER with a 3- day history of vomiting. A week prior to consult, the patient complained of fever and malaise; for which he was given Aspirin by his mother. On PE, the patient is tachycardic and tachypneic. Serum chemistries reveal elevated transaminases. The key pathologic finding in the liver of patients with this condition is: A. Microvesicular steatosis B. Concentric bile duct fibrosis C. Massive hepatocellular necrosis D. Piecemeal hepatocellular necrosis E. Portal bridging fibrosis

548 A. Patient has Reye syndrome.

Hepatocellular carcinoma with the best prognosis A. Hepatitis B variant B. Fibrolamellar variant C. Scirrhous type D. Mucinous type E. NOTA

55 B. Fibrolamellar variant of HCC may have a better prognosis than conventional HCC. It usually occurs in young adults without a history of prior liver disease. It is characterized by oncocytic-like hepatocytes (abundant intracellular mitochondria) infiltrating fibrous stroma.

A 52-year old woman presents with a 4-month history of profuse watery diarrhea, weakness and dehydration. Testing reveals a serum calcium level of 11.4 mg/dL and a potassium level of 2.1 mEq/L. Abdominal CT done revealed a pancreatic mass. What is the most likely diagnosis? A. Glucagonoma B. Somatostatinoma C. Insulinoma D. Carcinoid tumor E. VIPoma

550 E. VIPoma (aka Verner Morrison syndrome) is a rare endocrine tumor, usually originating from the non islet cells of the pancreas that produces VIP. It is also known as the WDHA syndrome because massive amounts of VIP cause Watery Diarrhea with resultant Dehydrataion, Hypokalemia and Achlorydia. Patients also present with hypercalcemia hyperglycemia and metabolic acidosis.

A 70/M, retired teacher, complains of multiple raised pigmented lesions over his back. Some of the lesions are pruritic. On PE, you note multiple roughened brown waxy lesions that appear "stuck on" over the patient's back. Biopsy of the lesion reveals sheets of small cells that resemble basal cells of normal epidermis; associated with the presence of small keratin- filled cysts. What is your primary impression? A. Actinic keratosis B. Seborrheic keratosis C. Malignant melanoma D. Basal cell carcinoma E. Squamous cell carcinoma

551 B. The histologic features of the lesion does NOT suggest malignancy, ruling out C, D and E. Seborrheic keratosis is a common, multiple, benign skin tumor; and appear as well-circumscribed brown plaques with a stuck on appearance. Actinic keratosis lesions on the other hand are skin colored, yellowish or erythematous ill- defined irregular shaped scaly macules or plaques localized in sun-exposed areas of the body. Usually, a case of actinic keratoses would feature a 'farmer' or any other character with a history of chronic sun exposure.

A 35/M with a 10-pack year history of smoking comes in due to a cold painful fingertip with a beginning ulcer. You elicit a history of Raynaud phenomenon in the patient. On PE, you note decreased brachial, ulnar and radial pulses. What is your primary impression? A. Microscopic polyangitis B. Polyarteritis nodosa C. Churgg-Strauss syndrome D. Takayasu arteritis E. Thromboangiitis obliterans

552 E. Thromboangiitis obliterans or Buerger's disease is a non-atherosclerotic vascultiis resulting in segmental occlusion of medium-sized vessels. Patients are usuall young, male, with a strong history of smoking. It can affect the radial artery, presenting as recurrent Raynaud's phenomenon; or the tibial artery, presenting as instep claudication that persists even after exercise.

Urinalysis reveals increased urobilinogen, but absent urine bilirubin. This is consistent with which of the following conditions: A. Hereditary spherocytosis B. Hepatitis C. Pancreatic head malignancy D. Common bile duct obstruction E. Rotor syndrome

553 A. The point of the question is to differentiate between urobilinogen and urine bilirubin. Urobilinogen is formed by intestinal bacteria from conjugated bilirubin after it is secreted into the lumen; it is then reabsorbed by the enterohepatic circulation. Urine bilirubin, on the other hand, comes from conjugated bilirubin in the serum that is filtered into the urine. Recall that unconjugated bilirubin is NOT water- soluble and bound to albumin; and is not filtered. In obstructive jaundice (C and D), bile is not secreted into the intestinal lumen where bacteria converts conjugated bilirubin into urobilinogen. Thus, no urobinogen is found in the urine. In hepatitis, mixed bilirubinemia occurs (both unconjugated and conjugated bilirubin increase). Thus, we expect an increase in urine bilirubin due to increased amounts of conjugated bilirubin filtered by the kidney. In hemolytic anemia, unconjugated bilirubinemia predominates. Some of this excess bilirubin is secreted into the lumen where it is converted into urobilinogen. Urine bilirubin on the other hand is decreased, since most of the serum bilirubin in hemolytic anemia is UNCONJUGATED and not filtered by the kidneys.

A 60/M presents due to gross hematuria. You elicit a 3-month history of right-sided flank discomfort and abdominal fullness. Vital signs are as follows: BP: 170/100, HR: 122, RR: 22, T: afeb. On PE, you palpate a right-sided lower abdominal mass. You ordered an abdominal CT; the findings of which are consistent with renal cell carcinoma. Renal cell carcinoma is MOST COMMONLY derived from cells of the: A. Glomerulus B. Proximal tubule C. Loop of Henle D. Distal tubule E. Collecting tubule

554 B.

A 7-year old boy presents with a 5-day history of gradually worsening headaches and intermittent vomiting without fever. Patient was also noted to be walking with poor balance and coordination. MRI done revealed a cerebellar mass. Patient subsequently underwent excision of the tumor. Biopsy reveals that the tumor is extremely cellular, with sheets of anaplastic ("small blue") cells. Individual tumor cell are small, with little cytoplasm and hyperchromatic nuclei. Mitotic figures are abundant. What is the diagnosis? A. Pilocytic astrocytoma B. Pleomorphic xanthoastrocytoma C. Medulloblastoma D. Craniopharyngioma E. Glioblastoma multiforme

555 C. Medulloblastoma occurs predominantly in children and exclusively in the cerebellum. Histology reveals extremely cellular tumor with sheets of anaplastic small blue cells. Another commonly occuring cerebellar tumor in childen in plemorphic astrocytoma; but this benign condition would not usually present with necrosis and mitoses on histology.

Your 80-year old lola has become increasingly forgetful. She is no longer able to help in household tasks. She later become more irritable. Systemic physical examination and routine work-up is unremarkable. Her mini- mental status examination (MMSE) score is 20/30; there are no other significant neurologic examination findings. Her condition is characterized by which of the following histologic features: A. Neurofibrillary tangles B. Pick bodies C. Lewy neurites D. Fibrillary gliosis E. Rosenthal fibers

556 A. At the microscopic level, Alzheimer's Disease is diagnoed by the prsence of plaques and neurofibrillary tangles. Lewy bodies are seen in Parkinson's disease; Pick bodies are seen in Pick disease, a subtype of Frontotemporal lobar degeneration. (In frontotemporal lobar degeneration, behavior changes precede memory disturbances which assist in their separation from AD on clinical grounds.) Rosenthal fibers are features of pilocytic astrocytoma.

A 70/M is evaluated for a heart murmur. On PE, a 3/6 systolic ejection murmur radiating to the neck is heard on auscultation. Echocardiogram shows aortic stenosis and normal systolic function. Which of the following cellular changes is BEST demonstrated in this condition? A. Atrophy B. Coagulative necrosis C. Hypertrophy D. Dystrophic calcification E. Metastatic calcification

557 D. "Dystrophic calcification of the aortic valces is an important cause of aortic stenosis in elderly persons." Basic Patho 9e p.25

On the 28th day of the menstrual cycle, a woman undergoes menstrual bleeding lasting 2-4 days consuming 3 pads per day. Which of the following cellular changes is BEST demonstrated in this condition? A. Atrophy B. Apoptosis C. Coagulative necrosis D. Liquefactive necrosis E. Hypertrophy

558 B. Menses is an example of apoptosis or programmed cell death.

A 45/M has been drinking alcoholic beverages heavily for the past 10 years. The following are laboratory findings expected in alcohol abuse except: A. Hyperuricemia B. AST>ALT C. Increased GGT D. Hypertriglyceridemia E. Fasting hyperglycemia

559 E. Hyperuricemia occurs in chronic alcohol use because lactic acid and beta-hydroxybutyrate compete with uric acid for excretion in the proximal tubules. Alcohol is a mitochondrial toxin that causes release of ALT, located in the mitochondria. It also induces hyperplasia of SER causing increased synthesis of GGT. Increased levels of NADH also lead to the conversion of DHAP into G3P which is used as a substrate for triglyceride synthesis in the liver. Fasting HYPOglycemia occurs because excess NADH causes pyruvate (the substrate for gluconeogenesis) to be converted to lactate.

This pathology is characterized by distinctive combination of mesangial and endothelial proliferation, along with thickening and duplication of the capillary basement membrane A. Amyloidosis B. Diabetic nephropathy C. Membranoproliferative glomerulonephritis D. IgA nephropathy E. Post-streptococcal glomerulonephritis

56 C. The decription stated is of a "tram track," characteristic of MPGN.

A 30/M, call-center agent, known case of HIV- AIDS, presents with progressive generalized swelling. On PE, patient has non-pitting edema over the lower extremities extending up to the mid-abdomen. Edema was refractory to steroid therapy. Renal biopsy done revealed increased mesangial matrix, obliterated capillary lumina, and deposition of hyaline masses and lipid droplets in affected glomeruli. Immunofluorescence microscopy revealed trapping of immunoglobulins and complement in areas of hyalinosis. On electron microscopy, the podocytes exhibit effacement of foot processes. What is the most likely diagnosis? A. Membranous nephropathy B. Minimal change disease C. Focal segmental glomerulosclerosis D. Membranoproliferative glomerulonephritis E. IgA nephropathy

560 C. Remember that FSGS is the predominant glomerular lesion in patients with HIV-associated nephropathy.

Drugs can cause alteration in both acute and chronic alterations in respiratory structure and function. Among these drugs that cause pulmonary disease is nitrofurantoin which is associated with: a. interstitial fibrosis b. hypersensitivity pneumonitis c. bronchiolitis obliterans d. eosinophilic pneumonia

561 B. Hypersensitivity pneumonitis. Table 15-7 of Robbins

Certain pulmonary entities exhibit a histologic picture of eosinophilic infiltration. Which of the following disease is characterized by a benign clinical course? a. Acute eosinophilic pneumonia b. Loeffler syndrome c. Tropical eosinophilia d. Chronic eosinophilic pneumonia

562 B. Loeffler syndrome or Simple pulmonary eosinophilia is characterized by transient pulmonary lesions, blood eosinophilia and a benign clinical course.

Patient presented with fever, headache, muscle aches, leg pains and cough. PE did not show findings of consolidation. Histologic pattern of the disease show a predominant interstitial nature of inflammatory reaction virtually localized within the walls of the alveoli. What is the most probable causative agent? a.) Klebsiella pneumonia b.) Streptococcus pyogenes c.) Mycoplasma pneumoniae d.) Staphylococcus aureus

563 C. Atypical pneumonia include viral pneumonias and mycoplasma pneumonia. P714 Robbins.

Fleur de lis pattern is characteristic of which infectious agent? a. Pseudomonas aeruginosa b. Mycoplasma pneumonia c. Streptococcus pyogenes d. Staphylococcus aureus

564 A.

Which of the following antibodies can cross the placental circulation? a. IgA b. IgG2 c. IgE d. IgG4

565 D. IgG4. IgG2 is the only subclass of IgG that may be unable cross the placenta. Source: Henrys Diagnostics

10 month old male presented with paralytic poliomyelitis after given an oral polio vaccine. What is the most probable underlying etiology? a. Severe combined immune deficiency syndrome b. Bruton’s Aggamaglobulinemia c. DiGeorge Syndrome d. Isolated IgA deficiency

566 B. Bruton’s /x-linked aggamaglobulinemia can predispose to paralytic poliomyelitis after live poliovirus immunization. Robbins p232

10 month old male patient had recurrent bouts of infection with Haemophilus and Staphylococcus aureus. He also had 3 episodes of Giardiasis since 6 months of age. What is the most probable defect? a. Complement b. B cells c. T cell d. NK cells

567 B.

15 year old patient presented with recurrent sinopulmonary infections of Haemophilus or Streptococcal origin. He has had recurrent bouts diarrhea despite treatment. What is the expected histologic picture of the patient? a. Decrease number of B cells in lymphoid tissues and blood. b. Decrease number of B cells in the lymphoid follicles of spleen, liver and gut. c. Normal or near-normal numbers of B cells in the blood and lymphoid tissues. d. Increase numbers of B cells in the peripheral circulation.

568 C. Answer is C as per Robbins p233

Helicobacter pylori is associated with blood type O individuals. The reason for this lies in the organisms capacity to bind to certain antigens expressed on the surface of cells like H antigen of the ABO blood group systems and the antigens of which other blood group system? a. Lutheran b. Lewis c. Kell d. Rh blood group

569 B. Lewis as per Henry’s diagnostics

This pathological process is characterized by acute vascular lesions include fibrinoid necrosis of renal arteries and arterioles, with onion skin lesions A. Renal artery stenosis B. Malignant hypertension C. Interstitial nephritis D. Hypertensive nephrosclerosis E. Vasculitis

57 B. Malignant hypertension is a rapid disease results in acute vascular lesions and parenchymal injury referred to as “malignant nephrosclerosis.â€

40 year old male has had recurrent episodes of epigastric pain for the past 3 weeks, relieved by eating. He had an endoscopy with biopsy done. Which of the following is true regarding the expected histologic picture of the gastric biopsy specimen? a. Intraepithelial neutrophils and subepithelial plasma cells are characteristic of H. pylori gastritis b. Although there is a good concordance between colonization of the antrum and cardia, infection of the cardia occurs at somewhat higher rates. c. H. pylori shows tropism to gastric epithelia and is generally generally found in association with gastric intestinal metaplasia or duodenal epithelium. d. Atrophic gastric mucosa with lymphoid aggregates can be seen.

572 D. A as per robbins p778 in the setting of an acute H pylori gastritis. D occurs in the setting of chronic H. pylori gastritis. Infection of the cardia occur at lower rates and H. pylori generally does not colonize duodenal epithelium.

Which of the following is not found the mesangium of a Post-Infectious Glomerulonephritis? a. IgG b.IgM c. IgA d. Complement

573 C. IgA is not found in PSGN but is more prominent in Buerger’s disease.

Which of the Gatrointestinal Polyposis syndromes does not have a hereditary component? a. Peutz-Jegher’s syndrome b. Juvenile Polyposis c. Cowden Syndrome d. Cronkhite-Canada syndrome

574 D. A is associated with an LKB1/STK11 mutation. Juvenile polyposis is associated with a mutation in the SMAD4 gene. Cowden with PTEN. Cronkhite-Canada syndrome makes its distinction by being nonhereditary.

70 year old female presented with a slow growing breast mass with a rubbery consistency. Histologic picture show islands of cells in pale gray-blue gelatin. What is the most probable tumor? a. Medullary breast cancer b. Ductal carcinoma c. Colloid Carcinoma d. Papillary breast CA

577 C. Colloid CA as per robbins

Which of the following is not associated with advancing age in the human heart? a. sigmoid septum b. mitral valve prolapse c. Lipofuscin deposistion d. Lambl excresences

578 B. all are associated with aging except MVP robbins p532

Crescentic glomerulonephritis is most associated with: A. p-ANCA B. HIV C. Cytomegalovirus D. Cryoglobulin E. Nephritic factor

58 A. ANCAs (anti-neutrophil cytoplasmic antibodies) are associated with pauci-immune crescentic glomerulonephritis. HIV is associated with focal segmental glomerulosclerosis; cryoglobulin and C3 nephritic factor may be associated with different types of membranoproliferative glomerulonephritis. Cytomegalovirus infection causes tubulointerstitial nephritis and is seen in immunocompromised patients.

Intestinal metaplasia of the stomach raises a predisposition towards adenocarcinoma development. Which cell most likely found confirms intestinal metaplasia? a. Mucous cells b. Goblet cells c. Isolated islands of Brunner glands d. Simple columnar epithelia

580 B. Goblet cells and intestinal columnar absorptive cells when present points to intestinal metaplasia

A 7 year old male was admitted due to easy fatigability, fever, and easy bruisability for 2 months. CBC done showed the following: Hgb=94 mg/L, Hct=0.29, Plt=54, WBC=45.50, Segmenters=0.10, Lymphocytes=0.66, Metamyelocyte=0.04, Blast=0.16, Myelocyte=0.04. Bone marrow biopsy showed hypercellular marrow with cells which have scant cytoplasm with nuclei containing coarse and clumped chromatin. Immunostaining showed (-) myeloperoxidase and (+) periodic acid-Schiff cytoplasmic material. What is the most likely diagnosis? A. Acute lymphoblastic leukemia B. Acute myeloblastic leukemia C. Chronic lymphocytic leukemia D. Chronic myelogenous leukemia E. Multiple myeloma

581 A. By definition, in ALL, blasts compose more than 25% of the marrow cellularity. The nuclei of lymphoblasts in Wright-Giemsa-stained preparations have somewhat coarse and clumped chromatin and one or two nucleoli; myeloblasts tend to have finer chromatin and more cytoplasm, which may contain granules. The cytoplasm of lymphoblasts often contains large aggregates of periodic acid-Schiff-positive material, whereas myeloblasts are often peroxidase positive.

A 30-year-old woman presents with malaise and increasing fatigue. On physical examination, she has anicteric sclerae and jaundice, and laboratory evaluation finds elevated AST and ALT. Liver biopsy showed shrunken hepatocytes with highly eosinophilc cytoplasm and fragmented nuclei. There were no noted inflammatory infiltrates. Which of the following terms best describes this process occurring in the hepatocytes? A. Apoptosis B. Autophagy C. Heterophagy D. Necrosis E. Pinocytosis

582 A. Apoptosis involves single cells, not large groups of cells, and with apoptosis the cells shrink and there is increased eosinophilia of cytoplasm. The shrunken apoptotic cells form apoptotic bodies, which may be engulfed by adjacent cells or macrophages. With apoptosis there is no inflammatory response, the cell membranes do not rupture, and there is no release of macromolecules

A 60-year-old man died secondary to coronary artery disease. During autopsy, the lumen of the coronary arteries was covered with fibrous plaques and an area with ruptured plaque and superimposed thrombosis. Sections from these abnormal areas revealed a fibrous cap and a central core (largely lipid) core with an area showing disruption of the fibrous cap with thrombosis. Which one of the following substances promotes atherosclerosis by stimulating smooth-muscle cells to migration and proliferation in sites of endothelial injury? A. α-interferon (α-INF) B. β-transforming growth factor (βTGF) C. Interleukin-1 (IL1) D. Platelet derived growth factor (PDGF) E. Vascular endothelial growth factor (VEGF)

583 D. The pathogenesis of atherosclerosis depends in part on the inflammatory function of macrophages, which involves the release of numerous cytokines. Platelet- derived growth factor (PDGF) is mitogenic and chemotactic for smooth-muscle cells. This may explain the recruitment and proliferation of smooth-muscle cells in atherosclerosis.

A 56-year old post-gastrectomy patient consults due to pallor and easy fatigability. CBC done showed anemia. Peripheral smear showed large, egg-shaped macro-ovalocytes and hypersegmented neutrophils. Patient presents with? A. Sideroblastic anemia B. Iron deficiency anemia C. Anemia of chronic disease D. Aplastic anemia E. Megaloblastic anemia

584 E. In the peripheral blood the earliest change is usually the appearance of hypersegmented neutrophils, which appear even before the onset of anemia. Normally, neutrophils have three or four nuclear lobes, but in megaloblastic anemias neutrophils often have five or more. The red cells typically include large, egg-shaped macro-ovalocytes

A 40-year old consulted due to a slowly enlarging, soft, painless, pre-auricular mass. Surgical excision of the mass was done. Microscopically, the mass was composed of small, dark epithelial cells intermingled with a loose, myxoid connective tissue stroma with islands and strands of myoepithelial cells. These findings are suggestive of? A. Warthin tumor B. Pleomorphic adenoma C. Mucoepidermoid carcinoma D. Myoepithelioma E. Adenoid cystic carcinoma

585 B. The most common neoplasm of the parotid gland is the pleomorphic adenoma (mixed tumor), which histologically reveals epithelial structures embedded within a mesenchyme-like stroma consisting of mucoid, myxoid, or chondroid tissue.

A 46-year old female came in due to prolonged heavy menses. Dilatation and curettage was done. Endometrial scrapings revealed an increase in the number and size of endometrial glands, marked gland crowding and branching (back-to-back appearance) with little intervening stroma. The epithelial cells remain cytologically normal. This is suggestive of? A. Endometrial polyp B. Leiomyoma C. Endometriosis D. Endometrial hyperplasia E. Endometrial carcinoma

586 D. Most commonly, the prolonged unremitting estrogen stimulation results in endometrial hyperplasia. World Health Organization (WHO) classification takes into account both the architectural and cytologic features, in the sense of dividing the hyperplasias into simple and complex on the basis of the architecture, and subdividing each into typical and atypical on the basis of their cytology. Endometrial hyperplasia is most commonly seen during the perimenopausal period.

A 10-year old male complained of a painful, enlarging mass on the right femur. X-ray of the affected area showed layers of bone deposited in an onion-skin fashion. Surgical excision was done. Microscopic examination of the mass showed sheets of uniform, round cells with small amounts of clear cytoplasm with few mitoses and little intervening cytoplasm. What is the diagnosis? A. Osteosarcoma B. Chondroblastoma C. Ewing sarcoma D. Chondrosarcoma E. Rhabdomyosarcoma

587 C. Ewing sarcoma and PNETs arise in the medullary cavity and invade the cortex and periosteum to produce a soft tissue mass. The tumor is tan-white, frequently with hemorrhage and necrosis. It is composed of sheets of uniform small, round cells that are slightly larger than lymphocytes with few mitoses and little intervening stroma. Ewing sarcoma and PNETs typically present as painful enlarging masses in the diaphyses of long tubular bones (especially the femur) and the pelvic flat bones. Some patients have systemic signs and symptoms, including fever, elevated erythrocyte sedimentation rate, anemia, and leukocytosis that can mimic infection. X-rays show a destructive lytic tumor with infiltrative margins and extension into surrounding soft tissues. There is a characteristic periosteal reaction depositing bone in an onionskin fashion.

A 50-year old male consulted due to recurrent left ankle pain and swelling relieved by intake of NSAIDS. Arthrocentesis was done and microscopic examination of the synovial fluid showed long, slender, needle-shaped crystals which are negatively bifringent. This is indicative of? A. Gouty arthritis B. Calcium pyrophosphate deposition disease C. Viral arthritis D. Psoriatic arthritis E. Pseudogouty arthritis

588 A. The major morphologic manifestations of gout are acute arthritis, chronic tophaceous arthritis, tophi in various sites, and gouty nephropathy. Acute arthritis is characterized by a dense neutrophilic infiltrate permeating the synovium and synovial fluid. Long, slender, needle-shaped monosodium urate crystals are frequently found in the cytoplasm of the neutrophils as well as in small clusters in the synovium. The synovium is edematous and congested, and contains scattered mononuclear inflammatory cells. When the episode of crystallization abates and the crystals resolubilize, the attack remits

An 80-year old female was noted to have insidious onset of memory loss accompanied by alterations in mood and behavior, and impairment in higher level of intellectual function. There were no noted symptoms of ataxia, language disturbances or alterations in personality. This disease entity is differentiated from other causes of dementia due to the presence of? A. Cytoplasmic round to oval filamentous inclusion bodies that stain strongly with silver (Pick Bodies) B. Tau-containing neurofibrillary tangles C. Ballooned neurons positive for phosphorylated neurofilaments D. Patchy perivacuolar or plaque type patterns of misfolded protein deposition E. Spherical collections of dilated, tortuous, silver-staining neuritic processes (dystrophic neurites), often around a central amyloid core (neuritic/senile plaques)

589 E. At the microscopic level, Alzheimer disease is diagnosed by the presence of plaques (a type of extracellular lesion); and neurofibrillary tangles (a type of intracellular lesion). Neuritic plaques are focal, spherical collections of dilated, tortuous, silver- staining neuritic processes (dystrophic neurites), often around a central amyloid core.

Pretibial myxedema is characterized by: A. Hyaluronic acid deposition in the subcutaneous tissue B. Lymphocytic inflitrates in the dermis C. Chondroitin sulfate deposition within the dermis D. Both A and B E. AOTA

59 B. Pretibial myxedema is secondary to hyaluronic acid deposition and lymphocytic infiltrates within the dermis.

A 20-year old female complained of multiple bullae and vesicles on the face, axilla, and trunk. The lesions were noted to rupture easily that leaves shallow erosions. This disorder is caused by autoantibodies that result in dissolution of intercellular attachments within the epidermis and mucosal epithelium. The autoantibodies are directed against? A. Desmoglein B. Hemidesmosomes C. Reticulin D. Laminin E. Keratin

590 A. In pemphigus vulgaris, acantholysis selectively involves the layer of cells immediately above the basal cell layer, giving rise to a suprabasal acantholytic blister. It is caused by a type II hypersensitivity reaction. Patient sera contain pathogenic IgG antibodies to intercellular desmosomal proteins (desmoglein types 1 and 3) of skin and mucous membranes. The distribution of these proteins within the epidermis determines the location of the lesions.

A 6-year old female was noted to have bipedal edema. Urinalysis done showed no hematuria and massive proteinuria. Blood pressure was normal. There was a rapid response in the disease after administration of steroids. Electron microscopy will show? A. Thickened GBM, and double contour or tram-track appearance of glomerular capillary wall B. Irregular thickening of the GBM, lamination of lamina densa, and foci rarefraction C. Diffuse effacement of foot processes of podocytes, normal GBM, and absence of deposits D. Diffuse effacement of foot processes of podocytes and denudation of underlying GBM E. Thickening of the GBM with effacement of foot processes and subendothelial deposits

591 C. This relatively benign disorder is the most frequent cause of the nephrotic syndrome in children. It is characterized by glomeruli that have a normal appearance by light microscopy but show diffuse effacement of podocyte foot processes when viewed with the electron microscope. Even with the electron microscope, the GBM appears normal. The only obvious glomerular abnormality is the uniform and diffuse effacement of the foot processes of the podocytes. The cytoplasm of the podocytes thus appears flattened over the external aspect of the GBM, obliterating the network of arcades between the podocytes and the GBM. There are also epithelial cell vacuolization, microvillus formation, and occasional focal detachments. When the changes in the podocytes reverse (e.g., in response to corticosteroids), the proteinuria remits.

A 16-year old female complains of a midline anterior neck mass. The mass was 2 cm in diameter, soft, painless and moves when tongue is protruded. Excision of the mass was done and histology showed ducts and cysts lined by stratified squamous epithelium with adjacent lymphocytic infiltrate. This suggest? A. Brachial pouch cyst B. Thyroglossal duct cyst C. Epidermal inclusion cyst D. Thyroid cyst E. Sebaceous cyst

592 B. This is a congenital anomaly of the thyroid. A persistent sinus tract may remain as a vestigial remnant of the tubular development of the thyroid gland. Parts of this tube may be obliterated, leaving segments to form cysts. Segments of the duct and cysts that occur in the neck are covered by stratified squamous epithelium identical to the tongue in the region of foramen cecum. Anomalies in the lower neck area more proximal to the thyroid gland are lined by epithelium resembling the thyroid acinar epithelium. Adjacent to the lining epithelium are lymphocytic infiltrates.

A 50-year old female, menopause for 4 years will have a cytohormonal maturity index of? A. Parabasal=0, Intermediate=60, Superficial Cells=40 B. Parabasal=20, Intermediate=80, Superficial Cells=0 C. Parabasal=20, Intermediate=40, Superficial Cells=40 D. Parabasal=80, Intermediate=10, Superficial Cells=10 E. Parabasal=20, Intermediate=0, Superficial Cells=80

593 D. Intermediate cells are responsive to progesterone and superficial cells are responsive to estrogen. At the onset of menopause, the low levels of estrogen and progesterone will favor proliferation of parabasal cells making it the most predominant cell type on Pap smear.

A 23-year old medical student who had overnight binge drinking suddenly developed boring abdominal pain relieved by doubling up. Serum amylase and lipase were elevated. What type of necrosis occurs when pancreatic lipases were released in the substance of the pancreas and peritoneal cavity? A. Liquefactive necrosis B. Coagulative necrosis C. Caseous necrosis D. Fat necrosis E. Gangrenous necrosis

594 D. Fat necrosis refers to focal areas of fat destruction, typically resultingfrom release of activated pancreatic lipases into the substance of the pancreas and peritoneal cavity. On histologic examination, the necrosis takes the form of foci of shadowy outlines of necrotic fat cells, with basophilic calcium deposits, surrounded by an inflammatory reaction.

A 30-year old male had recurrent diarrhea, crampy abdominal pain, and fever for 3 months. Colonoscopy done showed areas of long serpentine linear ulcers oriented along the axis of the bowel with surrounding mucosal sparing. Biopsy of the lesion will show? A. Mucosal ulceration and mononuclear inflammatory infiltrates in lamina propia B. Transmural involvement of the bowel by inflammatory process with mucosal damage, non-casseating granuloma and fissuring with formation of fistula C. Submucosal fibrin and mucosal architectural disarray D. Inflammatory infiltrates predominantly lymphocytic without granuloma formation E. Submucosal polyps with underlying plasmacytic infiltrates

595 B. Crohn disease is characterized by (1) sharply delimited and typically transmural involvement of the bowel by an inflammatory process with mucosal damage, (2) the presence of noncaseating granulomas in 40% to 60% of cases, and (3) fissuring with formation of fistulae.

A 76-year old male consulted due to urinary frequency, dribbling, and sensation of incomplete voiding. Ultrasound of kidney, ureter, bladder and prostate showed a nodularly enlarged prostate with microcalcifications. PSA was 20 ng/mL. Prostatectomy was done and histolopath showed prostatic adenocarcinoma. Prostatic cancers occurs mostly in what area/region of the prostate? A. Inner/central glands B. Median lobe C. Outer/peropheral glands D. Transitional zone E. Peri-urethral zone

596 C. Outer/peripheral glands Prostate cancers usually occur in the outer/peripheral glands while benign lesions usually occur in the inner and transitional zones.

A 37-year-old woman presents with the acute onset of a productive cough, fever, chills, and pleuritic chest pain. CBC showed leukocytosis with predominance of neutrophils. A chest x- ray reveals consolidation of the entire lower lobe of her right lung. Which of the following is the most likely diagnosis? A. Bronchiectasis B. Bronchopneumonia C. Interstitial pneumonitis D. Lobar pneumonia E. Adult respiratory distress syndrome

597 D. Classically the onset of bacterial pneumonia is sudden, with malaise, shaking chills, fever, peripheral leukocytosis, and a cough with sputum production. Bacterial infections generally result in a polymorphonuclear (neutrophil) response. Bacterial infection of the lung (pneumonia) results in consolidation of the lung, which may be patchy or diffuse. Patchy consolidation of the lung is seen in bronchopneumonia (lobular pneumonia), while diffuse involvement of an entire lobe is seen in lobar pneumonia.

A 54-year-old female consults due to epigastric pain. History revealed she was taking mefenamic acid for joint pains. Gastroscopy revealed multiple, hemorrhagic ulcers <1 cm in diameter scattered throughout the stomach. Mucosal folds were normal. Biopsy of the lesions reveal sharply demarcated ulcers with normal adjacent mucosa. Mucosal erosions showed edema and hemorrhage. Which of the following is the most likely diagnosis? A. Active chronic gastritis B. Active gastritis C. Chronic gastritis D. Peptic ulcer disease E. H. pylori gastropathy

598 B. Gastritis is a nonspecific term that describes any inflammation of the gastric mucosa. Acute gastritis refers to the clinical situation of gastric mucosal erosions (not mucosal ulcers). Acute gastritis is also known as hemorrhagic gastritis or acute erosive gastritis. Acute gastritis is associated with the use of nonsteroidal anti-inflammatory drugs, such as aspirin, ibuprofen, and corticosteroids, and also with alcohol, chemotherapy, ischemia, shock, and even severe stress. Grossly acute gastritis appears as multiple, scattered, punctate (less than 1 cm) hemorrhagic areas in the gastric mucosa. This is helpful in differentiating acute gastritis from peptic ulcers, which tend to be solitary and larger. Microscopically the gastric mucosa from a patient with acute gastritis is likely to reveal mucosal erosions, scattered neutrophils, edema, and possibly hemorrhage.

20-year-old male consults due to red-brown urine noted 2 weeks after an upper respiratory tract infection. Urinalysis showed RBC of 50/hpf and red cell casts. CBC, BUN, creatinine, glucose, antinuclear antibodies (ANAs), and serum complement levels (C3 and C4) are within normal limits. Renal biopsy showed mesangial proliferation and matrix increase in light microscopy. Immunofluorescence reveals the presence of large, irregular deposits of IgA/C3 in the mesangium. A linear staining pattern is not found. Which of the following is the most likely diagnosis? A. Berger's disease B. Focal segmental glomerulosclerosis C. Goodpasture's syndrome D. Membranoproliferative glomeulonephritis E. Post-streptococcal glomerulonephritis

599 A. When hematuria follows within 2 days of the onset of an upper respiratory infection without skin lesions in a young patient, IgA nephropathy (Berger’s disease) should be considered. The characteristic immunoflorescent picture is mesangila deposition of IgA often with C3 and properdin with lesser amounts of IgG or IgM.

A 70 year male presented with a 12 hour history of anginal chest pain. The stat troponin I and ECG studies confirm the diagnosis of acute myocardial infarction. Unfortunately, he succumbed to a fatal arrythmia during his 30 minute minute stay in the ER before a definitive intervention could be applied. What are the expected microscopic findings on LM for his heart? A. Dense collagenous scar B. Variable waviness of the fiber C. Coagulation necrosis with neutrophilic infiltrate D. Beginning disintegration of muscle fibers with macrophage infiltrate E. None, it’s too early for histopathologic changes to occur.

6 C. 0-4hrs: None to Variable waviness of the fiber 1-3 days: neutrophilic infiltrate 3-7 days: macrophage infiltrate 1-2 wks: granulation tissue >2 mos: dense collagenous scar Source: Robbins and Cotran Pathologic Basis of Disease 8th ed p. 550

This is a disorder characterized by peripheral resistance to circulating thyroid hormone A. Refetoff syndrome B. Zellweger syndrome C. Reiter syndrome D. Thyroid dysplasmocytic syndrome E. NOTA

60 A. Refetoff syndrome is a disorder of peripheral resistance to circulating thyroid hormone.

A 21-year-old male presents with a testicular mass. Surgical resection of the mass was done and microscopic examination of the mass showed lace-like network of medium-sized cuboidal cells and Schiller-Duval bodies. Which of the following substances is most likely to be increased in this patient’s serum as a result of being secreted from the cells of this tumor? A. Acid phosphatase B. Alpha-fetoprotein C. Alkaline phosphatase D. Human chorionic gonadotropin E. Lactate dehydrogenase

600 B. Germ cell tumors of the testis often secrete enzymes or polypeptide hormones, examples of which include α-fetoprotein (AFP) and human chorionic gonadotropin (hCG). AFP is synthesized by the fetal gut, liver, and yolk sac. It may be secreted by either yolk sac tumors (endodermal sinus tumors) or embryonal carcinomas. AFP may also be secreted by liver cell carcinomas.

The following functions can be attributed to fibroblast growth factors, EXCEPT: A. Angiogenesis B. Platelet aggregation C. Skeletal muscle development D. Hematopoiesis E. Lung maturation

601 B. FGFs have the following functions: 1.New blood vessel formation (angiogenesis) 2.Wound repair: FGFs participate in macrophage, fibroblast, and endothelial cell migration in damaged tissues and migration of epithelium to form new epidermis. 3.Development: FGFs play a role in skeletal muscle development and in lung maturation. 4.Hematopoiesis

A 55-year old male, hypertensive, had his first attack of myocardial infarction. The type of tissue repair that will subequently occur in the affected areas of this patient's myocardium should be: A. Healing with scar formation B. Restitution of normal tissue C. Fibrosis D. A and C E. All of the above

602 A. Myocardial infarction would result to coagulative necrosis. Acute parenchymal inflammation will ensue along with tissue framework damage. Since myocardial cells are permanent cells, no regeneration would occur. Healing will involve scar formation. Fibrosis occurs in chronic inflammatory diseases, such as cirrhosis, pulmonary fibrosis and chronic pancreatitis.

Which of the following is NOT true regarding heparin-induced thrombocytopenia? A. This syndrome occurs after administration of unfractionated heparin (for purposes of therapeutic anticoagulation) B. There will be induced formation of antibodies that bind to molecular complexes of heparin and platelet factor 4 membrane protein. C. The result is platelet activation, endothelial injury, and a prothrombotic state D. A and C E. None of the above

603 E. Seen in upward of 5% of the population, this syndrome occurs when administration of unfractionated heparin (for purposes of therapeutic anticoagulation) induces formation of antibodies that bind to molecular complexes of heparin and platelet factor 4 membrane protein. This antibody can also bind to similar complexes present on platelet and endothelial surfaces; the result is platelet activation, endothelial injury, and a prothrombotic state.

The following are true regarding x-linked genetic diseases, EXCEPT: A. An affected male does not transmit the disorder to his sons. B. All daughters of an affected male are carriers. C. Sons of heterozygous women have 50% chance of receiving the mutant gene. D. Heterozygous female usually does not express the full phenotypic change because of the paired normal allele. E. None of the above

604 E. All statements are correct regarding x-linked diseases

Membrane proteins that recognize a variety of microbe-derived molecules and stimulate innate immune responses against the microbes: A. Major histocompatibility complex B. T-cell receptor (TCR) C. CD 44 adhesion molecule D. Toll-like receptors E. LPS-binding proteins

605 D. The Toll-like receptors (TLRs) are membrane proteins that recognize a variety of microbe-derived molecules and stimulate innate immune responses against the microbes. The first protein to be identified in this family was the Drosophila Toll protein, which is involved in establishing the dorsal- ventral axis during embryogenesis of the fly, as well as mediating antimicrobial responses

A bone marrow aspirate was obtained from a 70-year old who complained of weakness, weight loss, recurrent infections and bone pain. Laboratory findings showed anemia, proteinuria and an abnormal component in serum proteins. The bone marrow aspirate showed numerous plasma cells. The MOST probable diagnosis is: A. Multiple myeloma B. Monocytic leukemia C. NonHodgkin lymphoma D. Myelomonocytic leukemia

606 A. MM is characterized by a proliferation of malignant plasma cells and a subsequent overabundance of monoclonal paraprotein (M protein). The proliferation of plasma cells in MM may interfere with the normal production of blood cells, resulting in leukopenia, anemia, and thrombocytopenia. The aberrant antibodies that are produced lead to impaired humoral immunity, and patients have a high prevalence of infection, especially with encapsulated organisms. The overproduction of these antibodies may lead to hyperviscosity, amyloidosis, and renal failure

A patient with new skin lesions is found to have a gastric carcinoma. The following cutaneous lesions are consistent with this situation, EXCEPT: A. Seborrheic keratosis B. Leser-Trelat sign C. Acanthosis nigricans D. Erythema nodosum E. Pyogenic granuloma

607 E. Gastric carcinoma is associated with the following cutaneous lesions: leser-trelat sign, acanthosis nigricans, seborrheic keratosis and erythema nodosum.

Which of the following is/are NOT true regarding Familial Hypercholesterolemia? A. This is the consequence of a mutation in the gene encoding for the production of low density lipoprotein (LDL). B. There is a loss of feedback control and elevated levels of cholesterol that induce premature atherosclerosis, leading to a greatly increased risk of myocardial infarction C. Homozygotes, having a double dose of the mutant gene, are much more severely affected and may have fivefold to sixfold elevations in plasma cholesterol levels. D. Affected individuals develop skin xanthomas and coronary, cerebral, and peripheral vascular atherosclerosis at an early age. E. None of the above

608 A. Familial hypercholesterolemia is a "receptor disease" that is the consequence of a mutation in the gene encoding the receptor for low density lipoprotein (LDL), which is involved in the transport and metabolism of cholesterol. As a consequence of receptor abnormalities, there is a loss of feedback control and elevated levels of cholesterol that induce premature atherosclerosis, leading to a greatly increased risk of myocardial infarction.

A 53-year old male, who is a chronic IV drug user and alcoholic, presented with scleral icterus, abdominal tenderness on RUQ and positive fluid wave. Biopsy of the liver was done showing bridging fibrosis and hepatocytes which appear to have finely granular cytoplasm. The most likely diagnosis for this patient is: A. Acute alcoholic hepatitis B. Chronic hepatitis C infection C. Chronic hepatitis B infection D. Acute viral hepatitis E. None of the above

609 C. Bridging fibrosis is the hallmark of chronic hepatitis. Chronic hepatitis B infection will present with ground- glass hepatocytes (hepatocytes with finely granular cytoplasm), while chronic hepatitis C infection will show lymphoid aggregates, bile duct reactive changes and steatosis.

What is the interluekin that is vital for neutrophil chemotaxis? A. IL-1 B. IL-2 C. IL-6 D. IL-8 E. IL-10

61 D. IL-8 is the major chemoatactic factor for neutrophils. IL-1 is mostly resposnsible for fever. IL-2 stimutales grwoth of T cells. IL-6 stimulates acute phase protein production and IL-10 modulates the immune response

A 55 year old male was diagnosed with COPD. Pulmonary function test would reveal: A. Increased FRC B. Decreased FRC C. Increased FEV1 D. Increased FVC E. Decreased Surfactant

610 A.

Histologic pattern of carcinoma of the breast characterized by higher incidence of multicentricity and bilaterality, but not necessarily biologic aggression: A. Mucinous carcinoma B. Invasive lobular carcinoma C. Invasive ductal carcinoma D. Medullary carcinoma E. None of the above

611 B. Invasive lobular carcinoma, also known as infiltrating lobular carcinoma, is a type of breast cancer that starts in a lobule and spreads to surrounding breast tissue.

An 18-year old male noted recurrent painless hematuria after having bloody diarrhea and fever 2 weeks before. No other sign or symptom was noted. The most likely diagnosis is: A. IgA Nephropathy B. Minimal change disease C. Membranoproliferative glomerulonephritis D. Post-streptococcal glomerulonephritis E. None of the above

612 A. IgA nephropathy is the most common glomerulonephritis, presents with recurrent gross or microscopic hematuria.

A 66-year old female, nonsmoker, presented with chronic cough, weight loss, occasional chest pain and dyspnea. Chest x-ray revealed a single nodular density at the periphery of R middle lobe along with pleural effusion, blunting the R costodiaphragmatic recess. The most likely diagnosis is: A. Large cell lung CA B. Small cell lung CA C. Bronchioalveolar CA D. Adenocarcinoma E. Tuberculoma

613 C. Bronchioalveolar CA is a variant of adenocarcinoma that is not associated with smoking. It usually presents as a single nodule or consolidation at the periphery. Histologically, alveolar architecture remains intact along the tumor growth.

Cardiac condition characterized by small sterile vegetations on valve leaflets, usually associated with Systemic Lupus Erythematosus (SLE): A. Marantic endocarditis B. Libman-Sacks disease C. Infective endocarditis D. Rheumatic heart disease E. Non-bacterial thrombotic endocarditis

615 B. Marantic or nonbacterial thrombotic endocarditis is more commonly found in patients with mucinous adenoCA of the pancreas. It also presents with setrile, nondestructive vegetations. Infective endocarditis is usually due to bacteria, and vegetations are usually infected. Rheumatic heart disease is a chronic complication of inadequately treated GABHS pharyngeal infection.

A 25-year old female came in for pre- employment check up. Auscultation revealed a fixed widely split S2 and midsystolic murmur along the upper parasternal border. She is otherwise normal. The most likely diagnosis is: A. Atrial septal defect, sinus venosus type B. Atrial septal defect, primum type C. Atrial septal defect, secundum type D. Patent ductus arteriosus E. Ventricular septal defect

616 C. ASD is the most common congenital heart defect in adults. It usually presents with fixed widely split S2, with midsystolic murmur along the upper border of the sternum. 90% of the time, ASD is of secundum type. Primum type is usually associated with Down syndrome.

A 22-year old female was diagnosed with hyperthyroidism. Which of the following would be the earliest clinical manifestation that would be observed in this patient? A. Tachycardia, palpitations B. Diarrhea C. Proximal muscle weakness D. Weight loss E. Wide staring gaze and lid lag

618 A. Cardiac manifestations are among the earliest and most consistent features of hyperthyroidism

The following statements characterize Pneumocytes Type II, EXCEPT: A. They are the source of pulmonary surfactant B. They are the main cell type involved in the repair of alveolar epithelium C. They cover 95% of the alveolar epithelium D. A and B E. None of the above

619 C. Alveolar epithelium, which contains a continuous layer of two principal cell types: flattened, platelike type I pneumocytes covering 95% of the alveolar surface and rounded type II pneumocytes. Type II cells are important for at least two reasons: (1) They are the source of pulmonary surfactant, contained in osmiophilic lamellar bodies seen with electron microscopy, and (2) they are the main cell type involved in the repair of alveolar epithelium after destruction of type I cells.

Patients who acquire nodular skin lesions containing foamy macrophages that are filled with numerous acidfast bacilli could be diagnosed to have: A. Tuberculoid leprosy B. Borderline leprosy C. Lepromatous leprosy D. Borderline tuberculoid leprosy

620 C. Lepromatous Leprosy (LL) is found in those patients with no or very little immune resistance to the Micobacterium leprae organism. When a biopsy of the affected LL skin is examined pathologically, it shows large collections of macrophages with foamy cytoplasm, called “Foam Cellsâ€. Tuberculoid (TT) leprosy - a “Paucibacilliary†type. Tuberculoid leprosy pressure atrophy is due to a large collection of lymphocytes and epithelioid cells, even in the relatively early stages of the disease

A 32 year old female was noted to have a breast mass with axillary lymph node enlargement. Further work ups, revealed that the mass was cancerous. A diagnosis of invasive ductal carcinoma was made. Modified radical mastectomy was done with axillary lymph node dissection. Pathological examination of the axillary lymph nodes will show? A. Follicular Hyperplasia B. Marginal zone B-cell hyperplasia C. Sinus histiocytosis D. Paracortical lymphoid hyperplasia E. Myelodysplastic proliferation

621 C. Sinus histiocytosis (also called reticular hyperplasia refers to the distention and prominence of the lymphatic sinusoids. This particular form of hyperplasia is prominent in lymph nodes draining cancers such as carcinoma of the breast. (SIMILAR TO PREVIOUS BOARD EXAM CONCEPT/PRINCIPLE)

A 32 year old male was admitted due to fatigue, unexplained fever and spontaneous mucosal and cutaneous bleeding lasting for about 2 weeks. Laboratory work ups revelaed anemia, neutropenia and thrombocytopenia. Peripheral blood smear examination revealed a red staining peroxidase-positive structures with abnormal azurophilic granules. What is the possible diagnosis to this case? A. Acute Lymphoblastic Leukemia B. Acute Myelogenous Leukemia C. Chronic Lymphoblastic Leukemia D. Hodgkin's Lymphoma E. Non-Hodgkin's Lymphoma

622 B. This is a case of AML, the presence of a red-staining peroxidase-positive structure (AUER RODS) is specific to AML (SIMILAR TO PREVIOUS BOARD EXAM CONCEPT/PRINCIPLE)

A presence of necrosis with nuclear changes involving basophilia of the chromatin which later on fades or dissolves is called? A. Karyolysis B. Pyknosis C. Karyorrhexis D. Histiocytosis E. Apotosis

623 A. Karyolysis- basophilia of the chromatin which fades or dissolves (SIMILAR TO PREVIOUS BOARD EXAM CONCEPT/PRINCIPLE)

A 58 year old female presented to the emergency department due to severe chest pain and diaphoresis. CK MB was elevated and Trop I was positive. ST segment elevation was noted in the ECG. On the 3rd hospital day, patient went into CHF accompanied by bilateral Grade 2 edema. What is the most probable etiology of the edema of this patient? A. Reduced oncotic pressure B. Increased hydrostatic pressure C. Sodium retention D. Inflammation E. Lymphatic obstruction

624 B. CHF cause increased hydrostatic pressure lcausing edema in this patient.

A 64 year old female was a known case of DM Type II. Futher work ups upon admission revealed elevated Creatinine and BUN. What is/are the expected morphology, if her kidney will be examined pathologically? A. Capillary basement membrane thickening B. Diffuse mesanglial sclerosis C. Nodular Glomerulosclerosis D. Presence of laminated PAS-positive nodules ( Kimmelsteil-Wilson Nodules) E. All of the above

625 E. DM renal morphology can be > Capillary basement membrane thickening > Diffuse mesanglial sclerosis > Nodular Glomerulosclerosis >Presence of laminated PAS-positive nodules ( Kimmelsteil-Wilson Nodules)

A 59 year old man is hospitalized after suffering a severe myocardial infarction. He is initially treated with nitrates, B blockers, and ASA and subsequently undergoues cardiac catheterization with placement of two stents. Following the procedure he is hemodynamically stable without recurrence of chest pain. However, 6 days after admission his heart rate is 134/min, BP is 70/30 mmHg and RR is 29 cpm. PE reveals distant heart sounds and an elevated jugular venous pressure. Which of the following complications is most likely causing this patient's symptoms? A. aneurysm formation B. cardiac arrhythmia C. fibrinous pericarditis D. rupture of the papillary muscle E. rupture of the ventriclar free wall

626 E. This patient is in shock due to cardiac tamponade secondary to rupture of the ventricular wall, which fibrinous pericarditis is common in 3-5 days, this condition usually presents with chest pain, often a pericardial friction rub can be auscultated.

A 5 year old female presents with a new onset hematuria and oliguria after 1 week of experiencing sore throat. Other pertinent findings revealed hypertension, periorbital edema and impaired renal function. A renal biopsy most likely would reveal electron-dense deposits in which of the following sites? A. between basement membrane and endothelial cells of the glomeruli B. between the basement membrane and epithelial cells of the glomeruli C. between the basement membrane and epithelial cells of the proximal tubules D. Within the mesangium of the glomeruli E. Within the juxtaglomerular apparatus

627 B. PSGN- deposits between basement membrane and epithelial cells of the glomeruli

Which of the following colorectal polyps has the highest risk of malignant degeneration? A. Villous adenoma B. Tubular adenoma C. Tubulovillous adenoma D. Hyperplastic polyp E. None of the above

628 A.

For the past year, a 48 year old woman has noted that her menstrual periods have been exceptionally heavy and last 7 - 9 days. She has noted occasional minor intermenstrual bleeding. For the past 3 months, she has been taking supplemental dietary iron for IDA. On pelvic examination, the uterine cervix appears normal, but the uterus is enlarged to twice its normal size. Transvaginal ultrasound reveals the presence of a 9 cm solid mass in the uterus. A hysterectomy is performed, and on gross inspection with sectioning the uterus, a reddish-tan mass is found with fleshy cut surface. Microscopically, the mass is highly cellular, with spindle cells having hyperchromatic nuclei and 10-20 mitoses per high power field. Which of the following is the most likely diagnosis? A. Endometrial polyp B. Adenomyosis C. Atypical hyperplasia D. Leiomyosarcoma E. Endometriosis

629 D. Leiomyosarcoma

This is considered an irreversible histologic manifestation of cellular injury A. Cellular swelling B. Nuclear chromatin clumping C. Nuclear pyknosis D. Ribosomal detachment E. Membrane blebbing

63 C. Nuclear pyknosis, karyorrhesis and karyolysis are considered irreversible changes of cell injury, The rest of the choices are reversible.

A 40 year old male suddenly experienced severe headache. Upon consultation, his BP is noted to be at 200/120. Past history shows normal BP on regular checkups. Anti hypertensive are immediately given. However, BP remains in the range of 200/120-180/100. Nicardipine drip is instituted. Two days later, the patient died becasue of intracerebral hemorrhage. Biopsy of the kidney revealed onion skin, concentric, laminated thickening of the walls of arterioles with progressive narrowing of the lumina consistent with what kind of arteriolosclerosis? A. Hyaline arteriolosclerosis B. Hyperplastic arteriolosclerosis C. Focal segmental nephrosclerosis D. RPGN E. Benign nephrosclerosis

630 B. Hyperplastic arteriolosclerosis

A patient with hemoptysis and renal failure has a renal biopsy which reveals crescentic glomerulonephritis, with linear deposits of IgG and C3 on immunofluorescence. What is the probable diagnosis? A. Acute post-infectious glomerulonephritis B. membranous nephropathy C. Goodpasture’s syndrome D. Minimal change disease E. Wegener's granulomatosis

631 C. Goodpasture's syndrome

A 1 year old patient came in due to fever. On physical examination , you noticed that the patient has epicanthal folds, flat facial profile, macroglossia and presence of simian crease. Further history revealed that the patient was born from a 48 year old mother. What is the most common cause of this chromosal disorder? A. Nondisjunction B. Robertsonian translocation C. Mosaicism D. Chromosomal deletion E. None of the above

632 A. This is a case of trisomy 21, MC is nondisjunction in 95 % of cases.

A 5 year old male developed a demyelinating neuropathy associated with Campylobacter jejuni. He presents clinically with ascending neuromuscular paralysis and areflexia. He was then treated with plasmapharesis, IV immunoglobulins and corticosteroids. The diagnosis to this case belongs to what type of hypersensitivity? A. Type I Hypersensitivity B. Type II Cytotoxic Hypersensitivity C. Type III Immune Complex Hypersensitivity D. Type IV Cell mediated Hypersensitivity E. Any of the above choices

633 D. This is a case of Guillan Barre Syndrome, a type IV Hypersensitivity reaction

What is the most common and the most severe form of LUPUS NEPHRITIS ? A. Minimal mesanglial B. Mesanglial proliferative C. Focal proliferative D. Diffuse proliferative E. Membranous

634 D. Type IV- Diffuse proliferative

A 23 year old female seek consult due to recurrent pyogenic infections caused mainly by staphylococci and streptoccoci. Further work up revealed that she is suffering from an autosomal recessive disease characterized by a failure of phagolysosomal fusion. Moreover, faulty microtubules was found to impair neutrophil chemotaxis. What is the most likely diagnosis? A. Leukocyte Adhesion deficiency B. Chronic Granulomatous Disease C. Chediak-Higashi Syndrome D. Wiskott-Aldrich Syndrome E. Ataxia-Telangiectasia

635 C. Phagocyte disorder= Chediak Higashi Syndrome

Which of the following disease-malignancy association is INCORRECT? A. Down syndrome: ALL, AML B. Actinic keratosis: squamous cell cancer of the skin C. Acanthosis nigricans: visceral malignancy D. Dysplastic nevus: malignant melanoma E. None of the above

636 E. All of the disease-malignancy association are correct

Cachexia is an irreversible catabolic reaction (wasting syndrome) common to those individual suffering from malignancies and tuberculosis. Which of the following substances is correlated to the pathophysiology of cachexia? A. elevated TNF alpha B. elevated VEGF C. elevated IL6 D. elevated Il 3 E. none of the above

637 A. increased TNF alpha is associated with cachexia which suppresses the appetite center and promotes beta oxidation of fatty acids for fuel

Beta HCG is a tumor marker for? A. Hydatidiform mole B. Choriocarcinoma C. Gestational trophoblastic tumor D. All of the above E. None of the above

638 D.

A 65 year old male immigrant from Africa presents to the emergency department after an episode of gross hematuria. He states that he has seen small amounts of blood in his urine from time to time over the past several months. His physical examination is remarkable only for mild hepatosplenomegaly. A urology consult is called, and the urologist performs a bedside cytoscopy. A large fungating mass is seen adherent to the superior part of the bladder. What is the most likely environmental exposure associated with this disease in the patient? A. Cigarette smoking B. Exposure to aniline dyes C. Schistosoma infection D. Infection with papova virus E. Long term indwelling catheter

639 C. diagnosis: squamous cell carcinoma of the bladder, MC cause is chronic irritation of Schistosoma haematobium.

Which statement refers to dystrophic calcification rather than metastatic calcification? A. Diffuse and widespread B. Ca deposition in normal tissues C. Patients are usually not normocalcemic D. Occurs in patients on long term hemodialysis E. Associated with thrombus and infarcts

64 E. Dystrophic calcification is ca deposition in abnormal tissues usually due to necrosis. It is seen in TB, infarcts, thrombus, schistosomiasis, congenital CMV, toxoplasmosis and psamomma bodies. The rest of the choices refers to metastatic calcification.

Glanzmann thrombasthenia is a deficiency of? A. GpIIb-IIIa complex B. GpIb C. Factor IX D. Fibrinogen E. vWF

640 A. GpIIb-IIIa complex- Glanzmann GpIb- bernard soulier

What do you call the systemic reaction including rashes, brochoconstriction and periorbital edema after injecting an unknown antigen to different parts of your body? A. Arthus Reaction B. Serum Sickness C. Delayed hypersensitivity reaction D. Anaphylactic Shock E. None of the above

641 B. Arthus reaction is almost the same as serum sickness but occurs as a local inflammatory reaction. Anaphylactic shock is re-exposure to a known allergen, leading to severe vasodilatation and hemodynamic instability.

Which among the following components of the immune system is NOT included in the pathophysiology of HIV infection? A. CD4 B. CD8 C. Dendritic cells D. Macrophages E. No excemption, all of the above are included in the pathophysiology of HIV infection

642 B. CD8 or the cytotoxic cells are not included in the pathophysiology of HIV infection. It is the CD4 that is affected by HIV.

Which of the following imunoglobulins is/are being secreted by the immune system of the GIT? A. Dimeric IgA B. IgG C. IgM D. B and C only E. All of the above

643 E. As stated in Robbins, the immune system of the GIT secretes dimeric IgA, IgG and IgM.

While on duty as IM resident, a 28 year old female came to ER who presented with th triad of microangiopathic anemia, thrombocytopenia and renal failure. Further history revealed eating half-cooked burgers 2 days PTC. Which among the choices is/are true regarding the etiologic agent? A. Shiga-like toxin B. EHEC C. E. coli 0157:H7 D. B and C only E. All of the above

644 E. this is Hemolytic-Uremic Syndrome. It is caused by shiga-like toxin from E. coli 0157:H7 from poorly cooked burger patties.

A 47 year old male consulted your clinic due to unexplained fever accompanied by weakness. CBC revealed leukocytosis of 187,000/mm3. Suspecting leukemia, you requested bone marrow biopsy which revealed scattered macrophages with abundant wrinkled, green - blue cytoplasm. What is the diagnosis? A. AML B. CML C. ALL D. CLL E. Acute Promyelocytic Leukemia

645 B. the scattered macrophages with abundant wrinkled, green--blue cytoplasm is the "sea-blue histiocytes" which is the pathognomonic finding for Chronic Myelogenous Leukemia

Which among the following statements is/are true regarding thymoma? A. Associated with Pure Red Cell aplasia B. Associated with Myasthenia Gravis C. Histopath findings may show swirling pattern of sphindle-shaped cells D. B and C only E. All of the above

647 E. topnotch notes

This is the most common complication of Chronic Otitis Media, characterized by cystic lesions filled with amorphous debris. A. Chloroma B. Branchial pouch cyst C. Cholesteatoma D. A and C only E. None of the above

648 C. Cholesteatomas are mainly deriveds from desquamated epithelium resulted from chronic otitis media.

A 54 year old female presented with painless palpable mass on upper-outer quadrant of her left breast. History revealed previous surgical removal of a mass (2 months PTC) of which histopath showed fibroadenoma. If you are about to request a biopsy of the new mass, what findings do you expect? A. localized area of inflammation which may progress to abscess formation B. ill-defined, firm, gray-white nodules C. chalky-white foci with or without hemorrhagic debris D. A and B only E. B and C only

649 E. this is a case of Traumatic Fat necrosis. Usual clinical picture is a patient with history of trauma or prior surgery who presented with painless palpable mass. Choices B and C pertains to histopath findings of this condition.

Which of the ff mechanisms regarding leukocyte extravasatation is incorrectly paired? A. Margination and rolling- Selectins B. Margination and rolling- GlyCAM-1 C. Tight-binding- ICAM-1 D. Diapededis- VCAM-1 E. Migration- IL-8

65 D. Diapedesis uses PECAM-1 not VCAM (utilized in tight binding together with ICAM). All the other choices are correct

This is the most common type of breast cancer with characteristic grating sound (like cutting a water chestnut) when cut. A. DCIS B. LCIS C. Invasive ductal CA D. Invasive lobular CA E. Medullary CA

650 C. About 70% of diagnosed breast cancers are invasive ductal CA's. The grating sound is due to small, central pinpoint focior streaks of chalky-white elastotic stroma

Psoriasis is a skin disease with predilection to elbows, knees, scalp, and lumbosacral areas. Which among the following lesions is/are related to the said disease? A. Test tubes in rack appearance B. Suprapapillary plates wuth dilated, tortuos blood vessels C. spongiform pustules of Kajog D. A and C only E. All of the above

651 E. All of the choices given pertainsto the series of histopath findings can be found on psoriasis.

A 12 year old male was brought to your clinic due to pruritic, pink to skin-colored papules with central umbilication. What is your primary differential diagnosis? A. Acne vulgaris B. Verrucae C. Molluscum Contagiosum D. Impetigo E. Phemphigus vulgaris

652 C. this is a typical description of Molluscum contagiosum. Pathognomic finding is the central umbilication

Retinoblastoma is the most common primary intra-ocular malignancy of children. What do you call its pathognominic morphology which is described as arrangements of a single layer of tumor cells around an apparent lumen? A. Homer Wright Rosettes B. Homer Wright Pseudorosettes C. Flexner-Wintersteiner rosettes D. Mutton-Fat precipitates E. Hollenhorst plaques

653 C. Homer Wright Pseudo rosettes - seen in neuroblastoma, medulloblastoma. True rosettes are the flexner wintersteiner rosettes.

This is a sequelae of chronic Escherichia coli infection, characterized by histopathologic findings of large, foamy macrophages with laminatedmineralized concretions. A. Hemorrhagic cystitis B. Interstitial cystitis C. Malacoplakia D. Papillary Urothelial CA E. Papillary urothelial neoplasms of Low malignant potential

654 C. the histopath findings described is called Michaelis- Guttman bodies which is the pathognomonic finding for Malacoplakia

A 16 month old boy was brought to your clinic due to history of 2 days fever. Being so busy, you just requested urinalysis which showed PMN of 15-19 cells per HPF. You were told that the patient had 4 episodes of UTI for the past year. That is the only time you decided to examine his genitalia. You noticed that the orifice of his penile prepuce is too small. What is your diagnosis? A. Hypospadia B. Epispadia C. Phimosis D. Balanitis E. Balanoposthitis

655 C. Hypospadia is the condition where the meatus is located ventrally, epispadia if dorsally. Balanitis is the inflamation of glans penis.

A 49 year old female consulted to your clinic due to progressively enlarging abdomen. Upon PE, you noted a non-painful palpable mass at the left lower quadrant. Further studies showed elevated serum AFP, normal beta hCG and inhibin levels. You referred the patient to an obstetrician for operation. You are expecting to find Schiller-Duval bodies on histopath because you know this is a case of? A. Leydig cell tumor B. Sertoli cell tumor C. Yolk sac tumor D. Choriocarcinoma E. Teratoma

656 C. On histopath, Yolk sac tumor will show lacelike reticular cell network of structures resembling endodermal sinuses called Schiller-Duval bodies.

Which of the following statements is/are true regarding Guillain-Barre Syndrome? A. Immune-mediated demyelinating neuropathy B. Associated with Campylobacter jejuni infection C. Ascending motor paralysis D. A and C only E. All of the above

657 E.

This is a bone disease characterized by mosaic pattern or jigsaw puzzle appearance on microscopy. A. Osteitis Fibrosa Cystica B. Von Recklinghausen disease of the bone C. Paget's disease D. Osteogenesis imperfecta E. Osteopetrosis

658 C. choices A and B were the same. OI is a collagen type I problem. OP is the marble bone disease.

A 28 year-old construction worker fell from a 3- storey building and hit his head on the pavement. Patient came in conscious but after 20 minutes, became stupurous and eventually GCS 3. You are entertaining Epidural hematoma. Which among the following is/are associated with Epidural hematoma? A. fractured Pterion B. Lucid interval C. Lenticular lesion on CT scan D. B and C only E. All of the above

659 E.

Cigarette smoking is carcinogenic to the ff organs EXCEPT? A. Colon B. Bladder C. Pancreas D. Kidney E. Cervix

66 A. SIMILAR TO PREVIOUS BOARD EXAM CONCEPT/PRINCIPLE. Cigarrete smoking is assocated with maligancies in the ff organs: bladder, cervix, esophagus, kidne, larynx, lung, pancreas. Colon and breast malignancies usually does not have smoking as a risk factor/

The triad of Graves disease includes? A. hyperthyroidism B. ophthalmopathy C. Pretibial myxedema D. A and B only E. All of the above

660 E.

Type of cellular adaptation exhibited by genital warts? A. hyperplasia B. Hypertrophy C. metaplasia D. dysplasia E. None of the above

661 A. Number of cells in organ increase, the process is reversible when stimulus stops

Identify the mechanism of injury from cellular hypoxia? A. ATP depletion B. Free radical injury C. A & B D. None E. Hydrolysis

662 C. Hypoxia causes injury by depleting ATP sources (no ETC and Krebs cycle) and free radical injury

During the process of inflammation, laukocytes adhers to integrins found on endothelial surfaces thru this athesion molecules? A. E-selection (rolling) B. PECAM (diapedesis) C. Sialyl lewis (rolling) D. ICAM - I E. None of the above

663 D. E-selection (rolling) PECAM (diapedesis) Sialyl lewis (rolling)

True of the complement system? A. Consists of plasma proteins which participates in immune lysis of cells but does not play a role in inflammation B. C3A is chemotactic (C5A) C. C5A is an anaphyiotoxin (C5A & C3A) D. C3B is part of the MAC (C5B-A) E. None of the above

664 C. Consists of plasma proteins which participates in immune lysis of cells AND PLAYS a role in inflammation C3A is chemotactic - (C5A) C5A is an anaphyiotoxin (C5A & C3A) C3B is part of the MAC (C5B-A)

Observed in deep-sea divers who return to the surface too rapidly, resulting in musculoskeleteal pain and infarcts in the CNS? A. Embolism Syndrome B. Decompression Sickness C. Fluid Emboli D. Paradoxical Emboli E. None of the above

665 B. Decompression sickness (DCS; also known as divers' disease, the bends or caisson disease) describes a condition arising from dissolved gases coming out of solution into bubbles inside the body on depressurisation.

Newborn presents with a prominent acciput, micrognathia, low set ears and rodenbottom feet genetic d/o caused by nondisjunction is called? A. Down Syndrome B. DiGeorge Syndrome C. Edward's Syndrom D. Patau Syndrome E. None of the above

666 C. Edwards syndrome (also known as Trisomy 18 (T18)) is a genetic disorder caused by the presence of all or part of an extra 18th chromosome. This genetic condition almost always results from nondisjunction during meiosis.

Type of hypersensitivity reaction manifested by a patient with weight loss, taxhycardia, exophthalmos and elevated free thyroid hormones? A. Type I Anaphylaxis B. Type II Cytotoxic C. Type III Immune complex D. Type IV Delayed/Cell Mediated E. None of the above

667 B. Type I Anaphylaxis Type II Cytotoxic - Graves Disease Type III Immune complex Type IV Delayed/Cell Mediated

A patient whose mother was exposed to DES during pregnancy is prone to developing? A. VSD B. Limb Anomalies C. Acute Leukemia D. Clear Cell CA of Vagina E. None of the above

668 D. In 1971, DES was shown to cause a rare vaginal tumor in girls and women who had been exposed to this drug in utero.

Which of the following is the most toxic vitamin? A. Vitamin A B. Vitamin D C. Vitamin E D. Vitamin K E. None of the above

669 D. Vitamin D is a group of fat- soluble secosteroids responsible for intestinal absorption of calcium and phosphate.The threshold for vitamin D toxicity has not been established, however, the UL is 4000 IU/day for ages 9â€"71.

Which of the ff statements regarding neoplasia is INCORRECT? A. Tumor is said to be benign when its gross and microscopic appearances are considered relatively innocent B. Environmental influences not genetic factors appear to be the dominant risk factors for most cancers. C. Hamartomas are disorganized but benign masses composed of cells indigenous to the involved site. D. Benign neoplasms and poorly differentiated carcinomas of endocrine glands frequently secrete hormones characteristic of their origin E. Once the tumor cells breach the basement membrane, the tumor is said to be invasive.

67 D. Benign neoplasms and well-differentiated carcinomas of endocrine glands frequently secrete hormones character tic of their origin. Well-differentiated squamous cell carcinomas of the epidermis synthesize keratin, and well-differentiated hepatocellular carcinomas elaborate bile. All other statements are correct

A 5-Year old boy came to you with complaints of fever, headache and joint pains on physical examination you note erythematons papules and plaques appearing in crops lower abdomen and buttocks. History revealed that he had cough and colds two weeks prior to consult. What is the diagnosis? A. Polyartentis Nodosa B. Buerqer Disease C. Serum Sickness D. Henoch-Schonlein Purpura E. None of the above

670 D. HSP is a disease of the skin and other organs that most commonly affects children. In the skin, the disease causes palpable purpura (small hemorrhages); often with joint and abdominal pain.

A 40 year old male presented with crushing chest pain radiationg to his left arm, associated with dizziness and dyspnea. ECG was done at the ER showing chair-pattern tracing at chest leods V2, V3 and aVf. What is the most common complication 7 days after the incident? A. Arythmia B. Myocardial Rupture C. Ventricular Aneurysm D. Dressler Syndrome E. None of the above

671 B. Arythmia - 24 hours Myocardial Rupture - 4 - 7 days Ventricular Aneurysm - > 7 weeks Dressler Syndrome - pericarditis post - MI

A 21-year old male who has a history of recurrent throat infection develops fever and joint pains. On physical examination friction rub is heard at the latoral border of the sternum, with rapid pulse. What is the pathognomynic sign in this patient? A. Verrucae B. Anitschow Myocytes C. Aschoff Body D. Heberden's Node E. None of the above

672 C. Aschoff bodies are nodules found in the hearts of individuals with rheumatic fever. They result from inflammation in the heart muscle and are characteristic of rheumatic heart disease.

A 30 year old female with a history of oral contraceptive use presented with fatigue, headache, palpitations, sore tongue and generalized pallor, she was diagnosed to have filate dificiency. The following is seen in the PBS of patients with FA deficiency except? A. Hyposegmentad neutrophils B. Pancytopenia C. Macrocytosis D. a and b E. none

673 A. Folate deficiency is a lack of folic acid in the diet and the signs are often subtle. Folate deficiency anemia is the medical name given for the condition. PBS show hypersegmented nuclei, pancytopenia and macrocytosis.

Characteristic histopathologic findings in multiple myeloma? A. Fiery red cytoplasm B. Multiple nuclei with prominent nucleoli and cytoplasmic droplets containing immunoglobulin C. Pink gobluar cytoplasmic inclusions D. AOTA E. None of the above

674 D. In multiple myeloma, collections of abnormal plasma cells accumulate in the bone marrow, where they interfere with the production of normal blood cells. Most cases of myeloma also feature the production of a paraproteinâ€"an abnormal antibody which can cause kidney problems. Bone lesions and hypercalcemia (high calcium levels) are also often encountered

A 35-year old male came in due to fever, weight loss, night sweats and easy bruising. On physical examination, splenomegaly and swollen gums are present. CBC shows leukocytosis with predominance of blasts. PBS shows distinctive needle-like azurophilic granules in myeloblasts. What is the diagnosis? A. ALL B. AML C. CLL D. CML E. None of the above

675 B. AML is a cancer of the myeloid line of blood cells, characterized by the rapid growth of abnormal white blood cells that accumulate in the bone marrow and interfere with the production of normal blood cells. AML is the most common acute leukemia affecting adults, and its incidence increases with age.

In Von Willebrand disease, deficiency of VWF leads to dysfunctional platelet adhesion. Laboratory findings diagnostic of VWF includes? A. Inc. Bleeding time, Dec. Platelet count, Inc. PT, Inc. PTT B. Inc. Bleeding time, Dec. Platelet Count, Normal PT, PTT C. Inc. Bleeding Time, Normal Platelet Count, Normal PT, Inc. PTT D. Normal Bleeding Time, Normal Platelet Count, Normal PT, Inc. PTT E. None of the above

676 C. Inc. Bleeding time, Dec. Platelet count, Inc. PT, Inc. PTT (DIC) Inc. Bleeding time, Dec. Platelet Count, Normal PT, PTT (Thrombocytopenia) Normal Bleeding Time, Normal Platelet Count, Normal PT, Inc. PTT (Hemophilia)

Deficiency of alpha-1 antitrypsin leads to this type of emphysema? A. Centrilobular B. Panacinar C. Paraseptal D. Irregular E. None of the above

677 B. Severe A1AT deficiency causes panacinar emphysema or COPD in adult life in many people with the condition (especially if they are exposed to cigarette smoke), as well as various liver diseases in a minority of children and adults, and occasionally more unusual problems

Most common type of cancer arising from the distal 1/3 of esophagus? A. Squamous cell CA B. Barrett's esophagus C. Adenocareinoma D. Netastasis E. None of the above

678 C. Adenocarcinoma arises from glandular cells that are present at the junction of the esophagus and stomach.

Most common pituitary tumor? A. Prolactinoma B. Somatotropic Adenoma C. Acidophilic adenoma D. Corticotrophic Adenoma E. None of the above

679 A. A prolactinoma is a benign tumor (adenoma) of the pituitary gland that produces a hormone called prolactin. It is the most common type of pituitary tumor. Symptoms of prolactinoma are caused by too much prolactin in the blood (hyperprolactinemia) or by pressure of the tumor on surrounding tissues.

Overall, what is the most commonly mutated proto-oncogene seen in approximately 15% to 20% of all human tumors? A. RET B. BCL-2 C. RAS D. p53 E. Rb

68 C. Point mutations of RAS family genes constitute the most common type of abnormality involving proto- oncogenes in human tumors. Approximately 15% to 20% of all human tumors express mutated RAS proteins, but in some types of cancers the frequency of RAS mutations is much higher. p53 and Rb are tumor suprresor genes not proto-oncogene

A 20-year old female presented with malar rash, oral ulcers, protosensitivity and proteinuria with granules or casts, kidney biopsy was done showing a "wire loop abnormality" on light microscope with marked subendothelial immune complex deposition on electron microscope. What is the diagnosis? A. IqA nephropathy B. Goodpasture syndrome C. Focal segmental glomerulosclerosis D. Lupus nephropathy E. None of the above

680 D. A wire-loop lesion may be present in stage III and IV. This is a glomerular capillary loop with subendothelial immune complex deposition that is circumferential around the loop.

What endothelial-leukocyte adhesion molecule has a major role in adhesion, arrest and transmigration of neutrophils, monocytes and lymphocytes? A. P-selectin B. E-selectin C. VCAM-1 D. ICAM-1

681 D. Answer: D. ICAM-1 (Table 2-1, Robbins and Cotran Pathologic Basis of Disease, 8th ed.) *SIMILAR TO PREVIOUS BOARD EXAM CONCEPT/PRINCIPLE

Mrs. Kaka Awa, 32 yo presented to the ED because she passed blood instead of urine morning PTA after aerobics the night before. She also had 3-day history of recurrent attacks of tolerable abdominal pain and vomiting. Physical examination revealed mild jaundice, abdominal distention, and bilateral lower extremity edema. Laboratory evaluation revealed hemoglobin level of 7.0 g/dL, pancytopenia, total serum bilirubun level 4.4 mg/dL, ALT = 51 U/L, AST = 17 U/L, serum creatinine normal. Initial screening with sugar water hemolysis test was positive. Findings at abdominal radiography showed a small amount of bowel gas but disclosed no other abnormalities. US and CT findings included inferior vena caval, hepatic venous, and portal venous thrombosis, with an associated Budd- Chiari Syndrome. Further studies revealed acquired mutations in the phosphatidylinositol glycan complementation group A gene (PIGA). In the absence of liver disease, this raises the suspicion of what condition? A. Paroxysmal Cold Hemoglobinuria B. March Hemoglobinuria C. Hereditary Spherocytosis D. Marchiafava-Micheli Syndrome

682 D. Answer: D. MarChiafava-MiCheli Syndrome Notes: Paroxysmal NOCTURNAL Hemoglobinuria (PNH), sometimes referred to as Marchiafava-Micheli Syndrome, is a disease that results from acquired mutations in the PIGA gene, an enzyme that is essential for the synthesis of certain cell surface proteins. PNH triad: hemolysis, pancytopenia and distinct tendency to venous thrombosis. Screening test: Sugar water hemolysis test, confirmatory test: Ham’s test. The only form of treatment that currently can provide a definitive cure for PNH is allogeneic bone marrow transplantation (BMT).

After a week in the hospital for treatment of an upper respiratory infection complicated by pneumonia, a 43 yo female develops skin lesions that are 2 to 4 mm in diameter. These lesions are red, papulovesicular, oozing, and crusted and are located on her trunk and extremities. The lesions begin to disappear after she is discharged from the hospital a week later. What is the most likely pathogenesis for her skin lesions? A. Type I hypersensitivity B. Drug reaction C. Bacterial septicemia D. Photosensitivity

684 B. Answer: B. Drug reaCtion Notes: The time course fits best with a drug reaction producing an acute erythematous dermatitis. Urticaria from type I hypersensitivity is not as severe or as long lasting. Sepsis rarely involves the skin with an erythematous dermatitis. Photosensitivity may be enhanced by drugs, but UV light is the key component in light that produces photodermatitis.

A 45 yo female complained of headaches for about a month. She then suffered a generalized seizure and became obtunded. Her serum calcium concentration was found to be markedly elevated at 15.4 mg/dL, with a serum phosphorus level of only 1.9 mg/dL. The serum albumin level was 4.2 g/dL. A chest radiograph showed multiple lung masses, and there appeared to be lytic lesions of the vertebral column. Which of the following conditions best accounts for these findings? A. Parathyroid carcinoma B. Metastatic breast cancer C. Tuberculosis D. Vitamin D toxicity

685 B. Answer: B. MetastatiC breast CanCer Notes: The most common cause for clinically significant hypercalcemia in adults is a malignancy. Metastatic disease from common primaries such as breast, lung, and kidney tumors is much more frequent than parathyroid carcinoma, which tends to be local but aggressive.

In the third trimester of pregnancy, a 28 yo woman discovers a lump in her right breast. Her physician palpates a 2-cm, discrete, freely movable mass beneath the nipple. After delivery of a term infant, the mass appears to decrease slightly in size. The infant breast-feeds without difficulty. This breast lesion is most likely to be a (an) A. Intraductal papilloma B. Phyllodes tumor C. Lobular carcinoma in situ D. Fibroadenoma

686 D. Answer: D. Fibroadenoma Notes: Fibroadenomas are common, and they may enlarge in pregnancy or late in menstrual cycle. Most intraductal papillomas are smaller than 1 cm, and they are not influenced by hormonal changes. Phyllodes tumors are uncommon, and they tend to be larger than 4 cm. Lobular carcinoma in situ (LCIS) is typically an ill-defined lesion without a mass effect.

An otherwise healthy 72 yo male has increasing difficulty with urination. He has to get up several times each night because of a feeling of urgency, but each time, the urine volume is not great. He has difficulty starting and stopping urination. This problem has gotten worse over the last few years. His serum PSA level is slightly increased but stable over this time. A biopsy of the prostate is most likely to reveal which of the following? A. Hyperplastic nodules of stroma and glands lined by two layers of epithelium B. Poorly differentiated glands lined by a single layer of epithelium and packed back to back C. Foci of chronic inflammatory cells in the stroma and in normal-appearing glands D. Areas of liquefactive necrosis filled with neutrophils

687 A. Answer: A. HyperplastiC nodules of stroma and glands lined by two layers of epithelium Notes: The clinical features are typical of nodular hyperplasia of prostate. Mild elevation of the PSA level can occur with nodular hyperplasia. The area of the prostate that is most often involved with nodular hyperplasia to produce significant obstruction is in the inner (transitional and periurethral) zone.

This term refers to an increase in white blood cells in cerebrospinal fluid A. Cylindruria B. Pleocytosis C. Leukocytosis D. Xanthochromia

688 B. Answer: B. PleoCytosis

A 60 yo male who has terminal carcinoma of the colon develops widespread ecchymoses over his skin surface. The PT is 3 seconds, PTT is 55 seconds, platelet count is 15,200/uL, fibrinogen level is 75mg/dL, and fibrin split products levels are very elevated. Which of the following morphologic findings would you most expect to find on examination of the PBS? A. Howell-Jolly bodies B. Tear-drop cells C. Macro-ovalocytes D. Schistocytes E. Target cells

689 D. Answer: D. SChistoCytes Notes: This is an example of a DIC with associated microangiopathic haemolytic anemia. The DIC developed the setting of a mucin-secreting adenocarcinoma. Schistocytes are fragmented RBCs.

Which of these statements regarding atherosclerosis is correct? A. It literally means “hardening of the arteries" B. Fatt streaks are present in virtually all adolescents even without risk factors C. Age is the most important independent risk factor for atherosclerosis. D. Because the modified lipoproteins cannot be completely degraded, chronic ingestion leads to the formation of lipid-filled macro- phages called fatty streaks E. Fibrinogen is the major structural component of the fibrous cap, and accounts for its mechanical strength and stability

69 B. True enough, aortas of infants can exhibit fatty streaks, and such lesions are present in virtually all adolescents, even those without known risk factors. Arteriosclerosis literally means “hardening of the arteriesâ€. Family history and not age is the most important independent risk factor for atherosclerosis. lipid-filled macrophages are called foam cells. Collagen is the major structural component of the fibrous cap, and accounts for its mechanical strength and stability.

Mutya, a 3 yo girl presents with dark precipitates along gingival margins, radiopaque deposits in the epiphyses of her bones, and urinary excretion of delta-aminolevulinic acid (delta-ALA). Her father states that they live in an old house that has chipped paint. The child’s blood would most likely have which of the following? A. Schistocytes and helmet cells B. Basophilic stippling of erythrocytes C. Increased osmotic fragility of erythrocytes D. Clumping of erythrocytes at temperatures below 30OC

690 B. Answer: B. BasophiliC stippling of erythroCytes Notes: Classic features of lead poisoning forming a gingival lead line (composed of precipitated lead sulfide), radiopaque deposits in epiphyses, basophilic stippling of erythrocytes, increased delta-ALA, and peripheral neuropathy and other CNS changes.

A 35 yr-old woman who has been taking oral contraceptives for many years presents with acute abdominal pain and fullness. Paracentesis harvests 200 ml of bloody fluid. Imaging studies show a 6-cm mass in the liver that is subsequently resected. Histologic exam of this specimen would most likely reveal this to be which of the following? A. Angiosarcoma B. Cholangiosarcoma C. Focal nodular hyperplasia D. Hepatocellular carcinoma E. Liver cell adenoma

691 E. Answer: E Liver cell adenomas may occur after several years of taking oral contraceptives but the actual mechanism of tumor formation is unknown.

The alterations in the hemodynamic, metabolic and immune responses evident in stressed patients are orchestrated by endogenous polypeptides known as cytokines. They are produced by immune cells in direct response to injury, with levels correlating with the degree of tissue damage. Despite considerable overlap in bioactivity among cytokines, they are commonly classified by their predominant effect as proinflammatory or anti- inflammatory. Which is not considered as the former? A. IL-1 B. IL-4 C. IL-6 D. IFN-y

692 B. Answer: B IL-4, IL-10, IL-3 and TGF-B are all anti-inflammatory.

Malignancy is second only to trauma as the leading cause of death in children. In infants, it is the most frequent cause of death after prematurity and congenital anomalies. Approximately 40% of childhood anomalies are A. Lymphoma B. Leukemia C. Neuroblastoma D. Rhabdomyosarcoma

693 B. Answer: B Solid tumor for < 2 y.o = neuroblastoma, > 2 y.o= Wilms tumor

Upon investigation, a farmer was diagnosed to be infected with actinomycetes. His lung was noted to have local, subacute hypersensitivity reaction characterized by edema and necrosis along with complement activation. Your impression is : A. Serum Sickness B. Arthus reaction C. Graft-versus-host disease D. Goodpasture syndrome

694 B. Answer: B Serum sickness- type III systemic; Goodpasture- type II; GVHD â€" type IV

A 70 y.o man with sepsis has a pH of 7.18. which of the following statements is true regarding his metabolic acidosis? A. Tissue hypoxia leads to increased oxidative metabolism. B. Acute compensation for metabolic acidosis is primarily renal. C. Metabolic acidosis results from loss of bicarbonate or gain of fixed acids. D. Restoration of blood pressure with vsopressors corrects the acidosis associated with circulatory failure.

695 C. Answer: C Metabolic acidosis- initial compensation is respiratory. Hypoxia leads to anaerobic metabolism; volume replacement & not vasopressors will correct underperfusion.

In differentiating obstructive from restrictive lung diseases, this spirometry criteria characterizes the former: A. Decreased TLC B. Decreased FEV1/FVC ratio C. Decreased FRC/FEV ratio D. Decreased RV

696 B. Answer: B Obstructive: Inc.TLC, FRC,RV but FEV1 is more dramatically reduced = dec.FEV1/FVC ratio

An 85 y.o male patient was recently found to have an apical lung mass. A few months after, patient was noted to have episodes of drooping of eyelids, pupillary constriction and anhidrosis. This condition is due to A. Neoplasm of neuroendocrine Kulchitsky cells B. Lambert-Eaton syndrome manifested as muscle weakness C. Ectopic secretion of serotonin known as carcinoid syndrome D. Compression of cervical sympathetic plexus

697 D. Answer: D Pancoast tumor- cause Horner’s syndrome (triad of ptosis, miosis, anhidrosis)

The least important independent risk factor associated with increased risk for DVT: A. Obesity B. Central venous catheter C. Hospitalization with recent surgery D. Previous DVT

698 A. Answer: A Venostasis of lower extremities is asso.with prolonged bed rest, standing or sitting, my immobilization and ,muscular paralysis asso.with trauma and gen.& spinal anesthesia.

Most tumors tend to metastasize via the lymphatics or blood vessels. Which among these deviate from the more common pattern of spread and metastasize over the surface of viscera or body cavities? A. Colon CA B. Gastric CA C. Mesothelioma D. Hepatoblastoma

699 C. Answer: C Pattern of spread for mesothelioma is distinctive, it can even be diagnosed radiologically by a thick rind of tumor tissue it characteristically produces over the surface of involved lung.

Which of the following findings is an unlikely finding in malignant hypertension? A. Multiple punctate hemorrhage on the surface of both kidneys B. Trea-bark appearance of the ascending aorta C. Fibrinoid necrosis of arterioles D. Onion skinning of arterioles E. None of the above

7 B. Trea-bark appearance of the ascending aorta is a characteristic of tertiary syphilis. Source: Robbins and Cotran Pathologic Basis of Disease 8th ed p. 950

Pertaining to the previous question, where is the most common site of atherosclerosis? A. Abdominal aorta B. Coronorary artery C. Internal carotid artery D. Thoracic aorta E. Popliteal artery

70 A. In descending order, the most extensively involved vessels are the lower abdominal aorta, the coronary arteries, the popliteal arteries, the internal carotid arteries, and the vessels of the circle of Willis.

A 30 yr-old woman presents to her physician complaining of a recent nodular growth on her left neck. Further history reveals a 16-lbs weight loss and intermittent fevers over the past3-6 months. Physical examination reveals the presence of a firm, mobile, non-tender, 2.5 cm lower cervical lymph node, left neck. A biopsy is performed and pathology report indicates the presence of large cells with multilobate nuclei, abundant pale cytoplasm and significant band of sclerosis. Immunohistochemistry is positive for the presence of CD15 and CD30 cell markers. These findings are most consistent with which of the following neoplasms? A. Acute lymphoblastic leukemia B. HodgkinLymphoma C. Lymphoplasmacytic lymphoma D. Mantle zone lymphoma E. Mycosis fungoides

700 B. Answer: B Findings are consistent with nodular sclerosing HL, most common type (65-75%); cells described pertain to Reed-Sternberg cells.

A 70 yr old male had severe crushing chest pain and died 3 hours later before he was brought to the ER. Cause of death is myocardial infarction. What do you expect to see on light microscopy on time of death? A. Early coagulation necrosis B. Marginal contraction band necrosis C. Myocyte hypereosinophilia D. Waviness of fibers E. Pyknosis of nuclei

71 D. SIMILAR TO PREVIOUS BOARD EXAM CONCEPT/PRINCIPLE. The gross and microscopic appearance of an infarct depends on the duration of survival of the patient following the MI. Light microcopy findings: Less than 30 mins- None; 30 mins-4 hours- Usually none but variable waviness of fibers at border; 4-12 hours- Early coag necrosis; 12- 24 hrs- pyknosis; myocyte hypereosniphilia and marginal contraction band necrosis

A 50 yr old patient had routine checkup and the only finding is a midsystolic click on auscultation. You are suspecting a valvular defect. Which of the following pathologic changes is most likely present in the valve? A. Destructive vegetations B Dystrophic calcification C Fibrinoid necrosis D Myxomatous degeneration E Rheumatic fibrosis

72 D. The case points to possible mitral valve prolapse wherein the underlying pathology is myxomatous degeneration. The chordae tendineae become elongated and can rupture to produce acute valvular incompetence.

A patient came to you complaining of chronic back pain. Radiagraphic exams shows multple punched out lytic bone lesions. Which lab abnomrality is most likely? A. Serum IgM M protein B. Hypocalemia C. Increased serum alkaline phpsphatase D. Marked splenomegaly E. Polyclonal urinary light chains

73 A. Answer is A. Typo error. Choice should have been Serum IgA. This is highly suggestive of multitple myeloma. IgG or IgA M proteins are almaot always found in multiple myeloma. Additionaly lab criteria is is hypercalcemia, monocloonal urinary light chains and rolouex formation.

This pneumoconioses is seen as radiographically as an eggshell calcification of hilar lymph nodes described as stained glass in appearance? A. Asbestosis B. Berylloisis C. Coal worker's pneumoconioses D. Silicosis E. Hypersenstivity pneumonitis

74 D. Eggshell calcification of hilar lymph nodes is suggestive of silicosis.

This statement is correct regarding esophageal cancer. A. Adenocarcinoma is more common worldwide B. Half of squamous cell carcinomas occur in the lower third of the esophagus C. Reduced rates of Helicobacter pylori infection may be a factor in the increasing incidence of esophageal adenocarcinoma D. Squamous cell CA most commonly produce mucin and form glands often with intestinal type morphology E. Esophageal adenocarcinoma occurs most frequently in Caucasians and is equally distributed among genders

75 C. Some serotypes of Helicobacter pylori are associated with decreased risk of esophageal adenocarcinoma, because they cause gastric atrophy, which in turn leads to reduced acid secretion and reflux, and reduced incidence of Barrett esophagus. Squamous CA is most common and occurs at middle third of esophagus. AdenoCA produces mucin and is often the intestinal type morphology. It is 7 times more common in males than females.

On endoscopy of a 14 year old boy complaining of bleeding per rectum, a colonic mass was seen. Histologic examination demonstrates a characteristic arborizing network of connective tissue, smooth muscle, lamina propria, and glands lined by normal-appearing intestinal epithelium. What is the diagnosis? A. Familial adenomatous polyposis B. Peutz-Jeghers syndrome C. Juvenile polyposis D. Lynch syndrome E. Colon Adenocarcinoma

76 B. Grossly, the polyps are large and pedunculated with a lobulated contour. Histologic examination demonstrates a characteristic arborizing network of connective tissue, smooth muscle, lamina propria, and glands lined by normal-appearing intestinal epithelium. The arborization and presence of smooth muscle intermixed with lamina propria are helpful in distinguishing polyps of Peutz-Jeghers syndrome from juvenile polyps.

What is the most important characteristic of adenomatous polyps that best correlates with risk of malignancy? A. Morphology (tubular or villous) B. Degree of differentiation C. Size D. Presence of stalk (pedunculated or sessile) E. APC proto-oncogene mutation

77 C. Size is the most important characteristic that correlates with risk of malignancy. Although villous adenomas contain foci of invasion more frequently than tubular adenomas, villous architecture alone does not increase cancer risk when polyp size is considered.

A patient is suspected of having alcholic liver disease possible alcohol hepatitis. You expect to see what finding in liver biopsy? A. Clumped, amorphous, eosinophilic material in ballooned hepatocytes made up of tangled skeins of intermediate filaments B. Plasma cell predominance in the mononuclear inflammatory infiltrates C. Ground-glass hepatocytes D. Extensive feathery degeneration of periportal hepatocytes E. Circumferential “onion skin†fibrosis around an increasingly atrophic duct lumen

78 A. SIMILAR TO PREVIOUS BOARD EXAM CONCEPT/PRINCIPLE. Do not rely on buzzwords. Understand the morhpology. Choice a refers to Mallory bodies and is characertistic of Alcoholic hepatitis. Choice B refrs to autoimmune hepatitis. Choice C refers to chronic HepB infection. Choice D refers to cholestatis. Choice E refers to Primary sclerosing cholangitis

A child was brought to your clinic complaining of hematuria. He had sore throat 5 days prior to consult. What do you expect to find on electron microscopy? A. discrete, amorphous, electron-dense deposits on the epithelial side of the membrane, often having the appearance of “humps B. uniform and diffuse effacement of foot processes C. double contour†or “tram-track†appearance D. presence of electron-dense deposits predominantly in the mesangium E. proliferation of parietal cells forming crescents

79 D. SIMILAR TO PREVIOUS BOARD EXAM CONCEPT/PRINCIPLE.The case most likely points to IgA nephropathy. PSGN usually appears 1 to 4 weeks after a streptococcal infection of the pharynx or skin (impetigo). Choice A refers to PSGN, Choice B is miniman change disease. Choice C refers to MPGN Type 1. Choice E is RPGN

A 65 year old female presents with fever, headache and diplopia. On palpation, tenderness is noted along the course of the temporal artery. A biopsy would most likely confirm which of the following diagnosis? A. Takayasu arteritis B. Retinoblastoma C. Giant cell arteritis D. Kaposi sarcoma E. Katayama disease

8 C. This is a case of Giant cell arteritis. Along with involvement of the branches of the carotid artery, there is a close association with polymyalgia rheumatica. Takayasu arteritis is pulseless disease. It is more common in women of childbearing age. Initial symptoms are usually nonspecific, including fatigue, weight loss, and fever. With progression, vascular symptoms appear and dominate the clinical picture, including reduced blood pressure and weaker pulses in the upper extremities; ocular disturbances, including visual defects, retinal hemorrhages, and total blindness; and neurologic deficits. Involvement of the more distal aorta may lead to claudication of the legs; pulmonary artery involvement may cause pulmonary hypertension. Narrowing of the coronary ostia may lead to myocardial infarction, and involvement of the renal arteries leads to systemic hypertension in roughly half of patients. Source: Robbins and Cotran Pathologic Basis of Disease 8th ed p. 512

What is the microscopic finding in diabetic glomerulonephropathy? A. Mesangial expansion B. GBM thickening C. Eosinophilic nodular glomerulosclerosiss D. Renal atherosclerosis E. All of the above

80 E. SIMILAR TO PREVIOUS BOARD EXAM CONCEPT/PRINCIPLE. The most important glomerular lesions are capillary basement membrane thickening, diffuse mesangial sclerosis, and nodular glomerulosclerosis aka Kimmelstiel-Wilson disease. Renal atherosclerosis and arteriolosclerosis constitute part of the macrovascular disease in diabetics.

1. A 58 year old male presented with left sided weakness associated with headache and vomiting 4 hours prior to admission. Patient is a known hypertensive and currently takes Losartan. What is the most probable pattern of tissue necrosis seen in this patient? a. gangrenous necrosis b. coagulative necrosis c. liquefactive necrosis d. fat necrosis e. none of the above

81 B. liquefactive necrosis is characterized by digestion of dead cells resulting in transmoration of tissue into a liquid viscous mass. for unknown reasons, hypoxic death of cells within CNS often manifests as such. It is also seen in bacterial and fungal infections. Coagulative necrosis - architecture of dead tissues is preserved for a span of at least some days. Gangrenous necrosis - usually applied to a limb that has lost its blood supply and has undergone typically coagulative necrosis involving multiple tissue planes. Fat necrosis refers to focal areas of fat destruction from release of activated pancreatic lipases into pancreas and peritoneal cavity. (robbins 8th edition p.16)

2. A 20 year old male was bitten by a red ant on the dorsum of his foot. After 30 minutes, he noticed his foot was swollen, erythematous, and itchy. What is the most likely mechanism for the symptoms? a. increased transcytosis b. endothelial injury c. retraction of endothelial cells d. leukocyte mediated vascular injury e. none of the above

82 C. contraction of endothelial cells resulting in increased endothelial spaces is the most common mechanism of vascular leakage and is elicited by histamine, bradykinin, leukotrienes, substance P, etc. It is called the immediate transient response because it occurs rapidly after exposure to the mediator. (Robbins 8th edition p. 47)

3. After 1 day, patient noted increased pruritus, swelling, and erythema over the dorsum of his foot after repeatedly scratching it. What is now the most likely mechanism for the symptoms? a. increased transcytosis b. endothelial injury c. retraction of endothelial cells d. leukocyte mediated vascular injury e. none of the above

83 C. contraction of endothelial cells resulting in increased endothelial spaces is the most common mechanism of vascular leakage and is elicited by histamine, bradykinin, leukotrienes, substance P, etc. It is called the immediate transient response because it occurs rapidly after exposure to the mediator. (Robbins 8th edition p. 47)

4. A 28 year old female, CRB, presents 1 month history of easy fatigability, knee pain, mouth sores, and tea colored urine. Past medical history and family history was unremarkable. On workup, the physician noted a positive ANA and anti - dsDNA. Which of the following patterns of nuclear fluorescence most likely suggests the type of antibody present in the patient? a. homogeneous b. rim c. speckled d. nucleolar e. none of the above

84 B. rim or peripheral staining pattern are most indicative of antibodies to dsDNA. Homogeneous or diffuse usually reflects antibodies to chromatin, histones, and occasionally dsDNA. Speckled pattern is least specific and is the most commonly observed. This includes Sm antigen, RNP, SS-A, SS-B. (RObbins 8th edition p. 214)

5. Patient CRB was then started on steroids. Creatinine was noted to be slightly elevated on follow up. Which of the following is the hallmark of her disease? a. malar rash b. glomerulonephritis c. production of autoantibodies d. normocytic normochromic anemia e. none of the above

85 C. robbins 8th edition p. 213

6. A 2 year old female presented with 12 week history of recurrent, fever, cough, 15% weight loss, rash, and generalized lymphadenopathy. Patient was started with antibiotics without relief of symptoms. Biopsy of the lymph node was done which showed follicles depleted of cells and the organized network of follicular dendritic cells is disrupted. What is the most likely diagnosis? a. hodgkin's lymphoma b. AIDS c. ALL d. Miliary TB e. none of the above

86 B. robbins 8th edition p. 249

7. A 56 year old farmer comes to you with complaints of a painful chronic skin ulcer. Biopsy was done which revealed squamous cell carcinoma. On further workup, patient also has pulmonary nodules on the right middle lobe and left upper lobe. Which of the following agents is most likely responsible? a. benzene b. beryllium c. chromium d. arsenic e. none of the above

87D. benzene - leukemia, hodgkins lymphoma (principal component of light oil); beryllium - lung cancer (missile fuel and space vehicles); chromium - lung cancer (metal alloys, paints, pigments, preservatives) Robbins 8th edition p. 274

8. JCT, 5 year old child presents with fever, neck mass, and decreased level of consciousness. Past medical history includes pneumonia at 8 months old and German measles at 2 years old. Biopsy of neck mass showed a edematous interstitium diffusely infiltrated with macrophages, lymphocytes, and plasma cells. Which of the following is the most likely etiologic agent? a. SSPE b. mumps c. polio d. EBV e. none of the above

88 B. Mumps encephalitis causes perivenous demyelination and perivascular mononuclear cuffing. Aseptic meningitis is the most common extrasalivary gland complication of mumps infection occuring in 10% of cases. Robbins 8th edition p. 250

9. A 72 year old male chronic smoker presented with abdominal pain and bone pains at night. Serum calcium was noted to be elevated. Which of the following is the organ specific carcinogen in tobacco smoke? a. tar b. formaldehyde c. polycyclic aromatic hydrocarbons d. nicotine e. none of the above

89 C. Components of cigarette smoke particularly polycyclic hydrocarbons and nitrosamines are potent carcinogens in animals and likely to be directly involved in the development of lung cancer in humans. Robbins 8th edition p. 411-412

A 50 year old male was recently diagnosed with PTB. On physical examination, he appears emaciated. Which of the following is also called cachectin and is responsible for the weight loss and wasting noted in the patient? A. IL1 B. IL2 C. PAF D. TGF-beta E. TNF

9 E. Tumor necrosis factor or cachectin is secreted by activated macrophages and is also responsible for the cachexia in cancer patients. Source: Robbins and Cotran Pathologic Basis of Disease 8th ed p. 320

11. Which of the following refers to a plaque witha superficial fibrous cap composed of smooth muscle cells and relaively dense collagen with a lipid core? a. atherosclerotic plaque b. fatty streak c. all of the above d. none of the above e. none of the above

91 A. Atherosclerotic plaques have 3 principal components: 1) cells 2) ECM 3)intracellular and extracellular lipid. Robbins 8th edition p. 502

12. A 58 year old retired Chinese teacher came in for multiple injuries secondary to fall in the bathroom. X ray was done and showed a femoral fracture. ORIF was done and 48 hours post -op, patient was noted to be dyspneic, tachycardic with O2 sats 68%. Despite resuscitative efforts, patient died. What is the most likely autopsy findings of the heart? a. thickened right ventricular wall b. dilated right ventricular wall c. Disappearance of fat in the ventricular wall d. regurgitation and fibrous thickening of tricuspid valve e. none of the above

92 B. In acute cor pulmonale, there is marked dilation of right ventricle without hypertrophy. The rest of the choices refer to chronic cor pulmonale. Robbins 8th edition p. 560

13. An 8 year old male came for complaints of exertional dyspnea, recurrent high grade fever, subcutaneous nodules in the pulps of the digits, and tea colored urine. A year ago, patient was noted to have recurrent sore throat however no consult was done and no medications were taken. Which of the following is the hallmark seen in this case? a. friable bulky destructive vegetations b. small warty vegetations along the lines of closure c. small bland vegetations along the lines of closure d. medium sized vegetations on either sides of the valve leaflets e. none of the above

93 A. B - RHD, C - NBTE, D - Libmann sacks endocarditis robbins 8th edition p. 567

16. A 13 year old female presented with headache and decreased urine output after a 4 week history of skin infection. Which of the following is the most likely morphology of the kidney? a. linear GBM fluorescence for Ig and complement b. discrete subendothelial electron dense deposits c. granular deposits of IgG, IgM, and C3 in the mesangium and along the GBM d. GBM transformed into an irregular ribbon like, electron dense structure e. mesangial deposition of IgA

96 C. A - goodpasture's syndrome; B - MPGN type 1; D - MPGN type 2; E - IgA nephropathy Robbins 8th edition p.921

17. An 17 year old female presented with elevated blood pressure, tea colored urine, and pallor. Kidney biopsy was done which revealed glomeruli being large, hypercellular, and having an accentuated "lobular appearance". Which of the following is the most likely diagnosis? a. membranoproliferative glomerulonephritis b. membranous nephropathy c. IgA nephropathy d. rapid crescentic glomerulonephritis e. focal segmental glomerulosclerosis

97 A. B - uniform diffuse thickening of the glomerular capillary wall C - glomeruli show mesangial widening and endocapillary proliferation D - crescents which are proliferation of parietal cells and migration of monocytes and macrophages into the urinary space E - collapse of capillary loops, increase in matrix, and segmental deposition of plasma proteins aong capillary wall (hyalinosis) Robbins 8th edition p. 929

18. A 63 year old female, ECT, came in for complaints of bilateral knee pain. ECT has been self medicating Ibuprofen for 5 years. For the past week, ECT has been having recurrent headaches, vague abdominal pain, and easy fatigability. Which of the following occurs first in renal damage in analgesic nephropathy? a. cortical tubulointerstitial nephritis b. papillary necrosis c. ischemic kidney injury d. toxic kidney injury e. none of the above

98 B. Robbins 8th edition p. 945

19. Which of the following nipple discharges would most likely signify malignancy? a. bloody b. serous c. milky d. both a and b e. none of the above

99 D. robbins 8th edition p. 1068


Ensembles d'études connexes

States and Capitals (State/Capital) USA

View Set

Practice Quiz History and Layers

View Set

Test 5 - CH 51, 18, 19, 26, 55, 49, 52

View Set

Ch. 20: The Industrial Revolution Vocab & Questions

View Set

Spinal Cord and Subarachnoid Space

View Set

Compound Fracture (Preschooler) HESI Case Study

View Set

Georgia State Insurance Test Practice

View Set